Vous êtes sur la page 1sur 196

M.L.Krasnov G.I.

MaKarenKo
A.I. Kiseliov
;

CALCULO
VARIACIONAL
(ejemplos y problemas)

Editorial MIR

Los autores de este libru sen


Mijafl i<tasnov, Gri5tori Makn-~,ko, candidatos 8 Doctores en
Ciencias Usico-matel~tieas y docentes cl~l Instituto Ene~tico
de Mosc, y Alexandr Kiseliov,
colaborador- cientfficu superior
del Instituto Unleade de Investigaciones Nucieares de la
ciudad de Duhna.
Este compendio contiene problemas y ejercicios dedieedos 11
ilustrar 108 diferentes principiffl
de la !eoria y los mtodos de
resolucin de las ecuaciones por
el clculo de variaciones.

M. L. Krasnov, G. I. Makarenko,
A. l. Kiseliov

CLCULO
VARIACIONAL
(ejemplos y problemas)

Traducido del ruso por Carlos Vega)


candidato a doctor en ciencias fsico-matemticas

~
\~

Editorial

MI R

I.S.B.N.: 84-604-1605-4
D.L: M-$299-1992
@Editorial MIR- 1992
Impresin: Grficas Zenit - Madrid

Prefacio

a la edicin

Observaciones

espaola

preliminares

. . . . .

. . . . .
.

1. Extremo de funciones de varias variables


l. E xtrerno incondicionado
2. Extremo condictcnaoo
. . . . . . . . .

7
9

Captulo

Captulo 11. E(tremo de funcionales


3. Funcional. Variacin de una funcional 'i sus propiedades
Ecua 4. Problema elemental del Clculo var iacronal.
cin de Euler . . . . . . . . . . . . . . . .,
5. Generalizaciones del problema elemental del Clculo
variacional
. . . . . . . . . . . . . . . . ..
de la ecuacin de Euler .
6. lnvarianca
7. Campo de extremales . . . _ . . . . . . . . ..
~ 8. Condiciones suficientes de extremo de una funcional
9. Extremo condicionado
con fronteras mviles ..
,
10. Problemas varacionales
discontinuos.
Variaciones
Unilaterales
Ii. Problemas
12. Teora de H amil ton _. J acobi. Princip OS var lacionales
de la Mecnica . . . . . . . . . . _ . . . . ..
Captulo 1JI. Mtodos directos en el Clculo vartaclonal
13. Mtodo de diferencias finitas de Eul er . .
14. Mtodo de R. Hz. Mtodo de Kantorvich
.
15. Mtodos variaclonales para la determinacin de Jos
valores y de las funciones propios . . . . . .,
Respuestas e indicaciones

11

Hl

26
50
68
77
80
D1
106
121
134
143
15B
160
167
180

BIBLIOGRAFJA
l. Courant R. y D. Hilbert, Methoden der Mathernatischen Physik
(Mtodos de la Fsica Matemtica), vol. 11, Spr inger, Berl n, 1937.
2. L. Elsgoltz, Ecuaciones diferenciales y clculo vartacional, Editoral MIR, Mosc, 1969.
3. 11. M. FellbfjJal.() u C. B. f/JOAIUH., BapH8l(HOHHOe JtC'IHCJleHHe,
<l>l:I3M3TfH3, 1961 (/. M. Guelland y S. V. Fomin, Clculo variacional).
4. E. Goursat, Cours d'analyse
mathrnat lque (Curso de Anlisis
Matemtico),
vol. !JI, 58 ed., Gauth er-Villars. Pars. 1942.
5. M. A. Jlaepenmuee u Jl . A. Jliocmepnux, Kypc aap aauaonaoro
HC'IHCJH'RHSI,rOcTexH3J(3T, 1950 (M. A. Laurientieo y L. A. Lusternik, Curso de clculo v artacional).
6. J. Rey Pastor, P. Pi caun, y A. C. Treio, Anlisis matemtico,
vol. UI, 211 ed., Kapelusz, Buenos Aires, 1961.
7. L. C. Young, Lectures on the calculus of varratrons and optimal
control tneory (Lecciones sobre el clculo var laconal y la teora
de control ptimo), Ph il adelph ia, London, Toronto, 1969.

PREFACIO

A LA EDICiN

ESPANOLA

Hoy da todo ingeniero tropieza frecuentemente


con
problemas que requieren buenos conocimientos matemticos
y pericia en la aplicacin de distintos mtodos matemticos.
Se puede afirmar que la elevacin de la cultura matemtica
de los ingenieros contribuye a nuevos logros en la Tcnica
El Clculo varlaclonal es uno de los captulos del Anl sis
Matemtico clsico ms importante para las aplicaciones.
Actualmente,
en varios Institutos politcnicos el Clculo
variacional se incluye en el programa obligatorio del curso
de Matemticas superiores. Existen valiosos libros sobre el
Clculo variacional corno, por ejemplo, Jos libros de L. Elsgoltz 121, de E. Goursat (4), de 1. C. Young [7J, etc. En cuanto
a los problemas, muchos de ellos aparecen diseminados en
numerosos textos o artculos cientficos especiales dedicados
a este tema. Pero, por lo que conocernos, no existe en la
literatura correspondiente ningn libro de problemas dedicado especialmente al Clculo variacional. Los autores se
han planteado la tarea de preparar un cierto minlme de
problemas referentes a los cartulos principales del Clculo
variaclonal clsico (sin tocar as cuestiones relacionadas con
la Teora de direccin ptima).
El libro est escrito de forma que al principio de cada
pargrafo se dan los elementos tericos ind ispensables (definiciones, teoremas y frmulas) y se analizan detalladamente
ejemplos t picos.
El libro contiene ms de 100 ejemplos resueltos y 230
problemas para el trabajo individual. Al final se dan las
respuestas a todos problemas y, en algunos casos, las indicaciones correspondientes. Por eso, el libro puede servir tan to
para estudiar asignatura individualmente como para profundizar en el material que se expone en las lecciones.

PREFACIO

Consideramos deber nuestro agradecer al traductor Carlos


Vega, candidato a doctor en ciencias fsico-matemticas, por
el gran trabajo que se ha realizado al revisar los problemas
y por las tiles observaciones que ha hecho contribuyendo
al mejoramiento de) libro.
M. L. Krasnou
O. l. Makarenko
A. l. Kiselioo
,~\O~Ct

30 de Octubre de 1974.

OBSERVACIONES PRELIMINARES
l. Si A es un conjunto cualquiera de elementos, la proposrcron
el elemento a pertenece al conjunto A, se representa simblicamente as:
a E A.
Si se escribe a E A (o bien a E A). ello significa que el elemento a
no pertenece al conjunto A.
Siendo A y 8 dos conjuntos. la proposicin A es un subconjunto
del conjunto B (A c: B, simblicamente) significa que todo elemento x
del conjunto A tambin pertenece al conjunto B.
2. La unin y la interseccin de dos conjuntos A ':1 8 se definen
del modo siguiente:
la unin A U B = {x I x E A o x E 8} es la totalidad de los
elementos x que pertenecen por lo menos a uno de los conjuntos A o B:
la interseccin A n B = {x I x E A. x E 8} es la totalidad de los
elementos x que pertenecen tanto a A como a 8.
3. Si A es un conjunto Formado por nmeros reales, se denomina
cota superior (cota superior exacta) de A el menor nmero real M tal
que a ~ M para todo a E A. En otras palabras, M es la cota superior
de A si para todo a E A es a ~ M Y si para cualquier e > 0, por pequeo que sea. existe como minimo un elemento b E A tal que M - e < b.
Si no existe tal nmero. convendremos en decir Que la cola superior
de A es
En ambos casos designaremos la cota superior del conjunto A
por sup A.
Anlogamente se define la cota inferior del conjunto A que se represen la por inf A.
4. Se denomina espacio lineat todo conjunto R de elementos
x, y, z... de naturaleza arbitrarla para los cuales estn definidas dos
operaciones. de adicin y de multiplicacin por nmeros. que cumplen
los axiomas siguientes:
l~ x
y = !I
x;
2 (x
y)
z = x + (y
z) i
8 existe un elemento O (elemento nulo) tal que x
O = x para
todo x E R:
4) para todo x E R existe un elemento -x (elemento opuesto)
tal que x
(-x) = O;
5) lJt = x;

+00.

++ + +
+

6) a (Jlx)

(a~) x;

+ ~)x = ax + ~x;
+ ti) = + ay.

7) (ex.
8) ex. (x

(X.X

10

OI3SEI~VACJONES

PRELlMINARP.S

5. Vil espacio lineal R se llama normado si a todo elemento x E R


le corresponde un nmero real no negativo 11 x tI. llamado norma de
este elemento, con la particularidad
de Que
1) 11 x 11 == O slo si i(".." O;
2) Ctx 11 = I Ct , ti .e 11;
3) x -1- y 11 ~ 1I x 11
11 y 11 (axioma
triangular
para las norni

a s);

6. Un conjunto M de elementos x, y, z..


de naturaleza arbitraria se denomina espacio mtrico si a todo par de elementos x, y de M
le corresponde un nmero real no negativo p (x, y) de modo que
1) p (x, y) = o si, y slo si, x = y (axioma de identidad);
2) p (x, y) = P (1/. x) (axioma de simetra);
3) (l (x, y) + p (y, z) ~ p (x, z) (axioma triangular).
El nmero p (x, y) lleva el nombre de distancia entre los elementos
x e y.
TOCio espado lineal norrnado es mtrico: basta tomar p (x, y) =
= 11 x-ull
7. El espacio e la, b] es <.>1espacio formado por todas las funciones
.r/ (x) continuas en {a. b] donde
11

110 =

mx

Il~x::::b

y (x

l.

el

El espacio
[a, b] es el espacio formado por todas las funciones
y (x) Que, a parte de ser continuas, tienen derivada primera continua

en {a.

bl donde
11

IICl

rnx

a~;;<b

I y (x)

+ a~x<b
mx

I y'

(x)

en

El espacio
la, b] es el espacio formado por todas las unclones
y (x) Que, a parte de ser continuas, tienen en la, bl derivadas continuas
hasta de orden n-simo inclusive (fl es un nmero natural fijo) donde
7\

11 y /len

=~

mx I 11(Ir.) (x)

l.

k=O a~x~b

A veces

la norma del elemento y (x) en en la. b) se define as:


IIIJ lIen = mx {I y (x) J. ,y' (x) 1, ... 1yen) (x) I }.
a:!'i%~b

Capitulo

EXTREMO DE FUNCIONES DE VAR,IAS VARIABLES

l. Extremo incendicionado
Sea f {Xl' X2, Xn). o brevemente f (x), una Iuncin definida
en un recinto D del espacio eucldeo En de n dimensiones.
Diremos que la funcin f (x) alcanza su valor mximo (mnimo)
en el punto Xo E D si
f (x) ~ f (xo) (1 (x) ~ f (xo)
cualquiera

que

SE.'3

el punto

x E D.

Toda funcin continua en


un recinto acotado cerrado alcanza en l sus valores mximo y mnimo.
DEF1NICJON J. Sea f(x) una funcin definlda en un recinto De ECnj.
Diremos que el punto xCO) = (x y, ...
4) E D es un punto de mximo
estricto (un punto de mnimo estricto, respectivamente)
de la funcin
f (x) si existe una vecindad Q (xtO del punto Xl O) lal que la desigualdad
f (x) < f (x(O (1a desigualdad f (x) > f (xIO, respectivamente) se
cumple para todos los puntos x E g (x(o n D. x =t= x(o). Es decir, lo
que caracteriza el punto de mximo estricto (el punto de mnimo estricto, respectivamente)
es que
f = f (x) - f (x<o < O (f > O. respectivamente)
para todo x E Q (xlO n D. x =F x(o).
En cambio, si para el punto xc!~ existe una vecindad Q (x(O)
tal que para todos los puntos x E Q (xIO)
D se cumple la desigualdad
f (x) ~ f (XCO (la desigualdad f (x) ~ f (x<O. respectivamente). se
dice simplemente que el punto xlO) es un punto de mximo (un punto
de minimo, respectivamente).
DEFINICION 2. Los puntos de mximo y de mnimo da la funcin
f (x) se denominan puntos de extremo de la misma.
TEOREMA DE BOLZANO- WEIERSTRASS.

J. Basndose en la definicin, hallar Jos puntos de extremo de las funciones


a) {(XII X2)=X~+X~;

b) (XI> xz)=

e) f (XI'

x +xi

si

si

.\'"2) = xi

en el recinto D {x:

- x~

x~ ~ I}.

Xi+X~*

0,

xr+ x~=O;

CAP. 1, EXT~f;MO

12

DE FUNCIONES

TEOREMAI (condicin
necesaria de extremo). Sea
(x), x =
, Xn). Una funcin definida en una vecinda
del punto
x,o> = (xr, xg, ... , xg). Si este punto es un punto de extremo de la
=

(Xl> XZ, ...

funcin

(xl y .si en l existen las derivadas

;~f

vXj

ellas son iguales a cero:

al

(xIO
i)Xj

=o

(J'='
. I

(i = 1, '2...

, n),

todas

2, .. n.)

Si la funcin f (x) es diferenclable en el punto de extremo xlOl su


diferencial en este punto es igual a cero: dI (X\o,) = O.
EJEMPLO l. Hallar los puntos de extremo de la luncin z = x2
y2.
SOLUCION. Los puntos de extremo estn entre los puntos para los
cuales dz = O. En nuestro caso. dz = 2x dx
2y dy. La condicin
dz = O se cumple en el punto x = 0, y = O solamente. En efecto, s
x = y = 0, tenemos dz = O. Recprocamente, sea dz = O; basndonos
en que dx y dy son arbitrarios, tomemos dy = O de modo que 0= dz =
= 2x dx de donde. puesto que ax es arbitrario,
resulta que x = O;
anlogamente encontramos que tambin y = O. En el punto (O, Q)
tenemos z = O; en todos los dems puntos tenemos z = :(,1.
yZ > O.
Por eso, el punto (O. O) es un punto de minimo estricto de la funcin

= XZ + y2.

Si se am p1a 1a elase de f unciones en 1a que se busca el ex tremo


incluyendo en ella las funciones no dllerenciables en algunos puntos,
se llega a la siguiente condicin necesaria de extremo.
Si x(o) es un punto de extremo de la funcin f (Xl. XZ, , xn).
cada una de las derivadas parciales

o no existe en dicbo punto,


=

(i

1, 2, ...

, n) es igual a cero

EJEMPLO 2. Consideremos la parte superior z ;?- O del cono z~ =


.yz. Es obvio que la funcin z tiene. mnimo en el punto (O, O).

X2

Pero

:'

Vx

las derivadas ::
DEFINICIN

y:;

no

existen en este punto.

Los puntos en los que se cumple la condicin necesade la [uncin f (x) se denominan puntos crticos de la
3.

ria de extremo
misma.
Los puntos x(o) en los que dI (X,Ol) = O se denominan puntos estacionarios de la funcin f (x).
La condicin dI (x(O = O es equivalente a la condicin

al (.~IO)
aX

(i=I.2,

... ,n).

La existencia de punto cr itlco no garantiza an la existencia del


extremo de una funcin. Por ejemplo, el punto (0, O) es un punto estacionario de la funcin z = x - y'J. y, sin embargo, la funcin z no
tiene extremo en l: en cualquier vecindad del punto (O. O), por pequea
que sea. la funcin toma valores tanto positivos como negativos.

EXTREMO

13

INCONDICIONADO

10 Condiciones suficientes de extremo estricto


DE!" NICION 4. Se dice que la forma cuadr Iica
n

A (x)

_;

= A (Xl> X2, ... , xn) =


i,
0j"""

aj;

i,

oJxXj;

;-t

i = 1, 2, ... ,

TI;

positiva (definda negativa, respectivamente)


si A (x) > O
(A (x) < O respectivamente)
para todo punto x E En, x:::fo O. y se
anula slo para x = O, o sea, para Xl = X9 = ... = xn_ = O.
La forma cuadrtica S~ denomina no negativa si jams toma valores
negativos. Por ejemplo, las formas
es definida

son ambas no negativas. La primera es definida positiva ya que se


anula slo para Xl = Xz = ... = Xn = O; en carnhlo la segunda no lo
es ya que se anula, por ejemplo, para Xl = l. x2 = -1, x. = x. =
= " . = xn = O.
Una forma cuadrtica definida positiva o definida negativa se
denomina forma cuadrtica definida.
Una forma cuadrtica que toma valores tanto positivos como
negativos se denomina indefinida.
TEOREMA 2. (condiciones. suficientes
de; extremo estricto). Sea

(x)

una

= (xY, x],

x(O)

funcin

...

definida

en

una

vecindad

del

punto

X,Ol

, x~) en la que son continuos sus segundas derivadas y sea

un punto estacionario de la funcin

(x).

Si

la forma cuadrtica

( 1)

de la funcin f en el punto X<Ol. es definida


positiva (definida negativa, respectioamentes,
el punto x<O) es punto de
mnimo estricto (punto de mdximo estricto, respectivamente): si la forma
cuadrtica (1) es indefinida, no hay extremo en el punto xto).

o sea, la segunda diferencial

CRITERIO
DE snVESTER
POSITIVAS.
Condicin necesaria

DE FORMAS CUADRATICAS
DEFINIDAS
para que la forma cuadrtica

y suiiaente

(2)

CAr

14
eOIl aH

1. ux IIH::MO

aJj; i, ; =- 1, 2, ...
u > O.

DE FUNCIONL':>

, Il; sea definida positiva es que se cumpla

a121>
021 a22
aH

aH 012 a13

O.

a21 022 a23

>0,

...

a31 a32 U:\3

... a'

all 012

a21 (122

211

.. ~,

>0 .

Condict necesaria y suficiente para que la forma cuadrtica


sea definida negativa es que se cumpla

tll\<'O.

(2)

all al2 al3


11

(llzl>o.

a21 U22 a23

(12.\ D22

< 0, ...

a31 a32 a33

au

al:!

a21

azz ...

Dln

..

a2n

(-w>o.

CASOn=2. Sea f (x, y) una funcin definida en una vecindad del


punto (xo. Yo) en la que son continuas sus derivadas parciales de segundo
orden y sea (xo, Yo) un punto estacionario. es decir, sea

f~ (xo, Uo) =
Entonces, si en el punto (:eo. Yo)
I'!c:r.f~y -

(xo. Yo)

(f;'v)~

=.

O.

> O.

hay extremo en este punto: a saber. mximo si en l


y mnimo si en l
Si en el punto

(%0, Yo)

<O
n, > O
';ex

< O)
(!yy > O).
(f~1J

<

t;xfll - (f~y)"
O,
no hay extremo en E.'Ipunto (xo. Yo). Por ltimo, si en el punto (:eo. Yo)
f';.:.:lvy - (f';,1I)2 = O,

15

EXTREMO INCONDICIONADO

en dicho punto puede haber extremo y puede no haberlo; este caso


requiere un estudio complementara.
. EJEMPLO3. Consi derernos las funciones z = xf
y4, z = -x' - y'
y z = xf - y'. El punto (O, O) es un punto estacionario de las tres y
en l se tiene z:~:xZy- (Z;y)2. = O para cada una de las funciones.
Es fcil ver que el punto (0, O) es un punto de mnimo de la primera funcin, un punto de. mxlrrio de la segunda y no es punto de
extremo de la tercera. Efectivamente,
en los tres casos tenemos
2 (O, O) = O; sin embargo, en cualquier
vecindad del punto (0, O),
a excepcin del propio punto, los valores de la funcin son positivos
en el primer caso y negativos en el segundo mientras que en el tercer
caso la funcin z = x' - y' toma, en cualquier vecindad del origen
d~'coor denadas, valores tanto positivos (por ejemplo, si x ::fo Oe y = O)
como negativos (por ejemplo, si x = O e y ::fo O).
tiJEMPLO 4. Hallar el extremo de la funcin de tres variables
f = x20
y2
zia _ xy
x - 2z.
SOLUCIN. Determinamos los puntos estacionarios de la funcin f.
Consideremos con este fin el sistema de ecuaciones

+ +

:~ =2x-y+ I=0, )~
i)f

oy =2y-x= 0,
af

oz

Xo=

resolvindolo, encontramos
Consideremos

--t, 1).

JI

=2z-2=O;
2

-3' Yo = - 3'

11I Iorrna cuadrtica

(1)

y' Zo =: l.

en el punto

Tenemos
2,

)';;;0:=

flx=-I,

t;x=

O,
En el punto Po encontramos
an = 2,
an = -1,
a81 = 0,
de modo que

au

> 0,

';,,=-1,

au

2 -1

-1

-1,

a" = 2,
aa, = 0,

1_1

012
al1
G21 a22

f;z=O,

tv" = 2, ~rl.= O,
;"= o. f;z =2.

O
O 2

1-

2 - I
3 -.., O
l
2 - ~. ,

2 O =6::>0.
O

0,
= O,
a~3 = 2,
a13

a23

Po ( _. ~ ,

16

G,\P.

1. EXTREMO

DE FUNCIONES

Basndose en el crrerto de Sylvester. llegamos a la conclusin de que


la orma cuadrtica es definida positiva; por Jo tanto, en virtud del
teorema 2, el punto Po es un punto de mnimo estricto siendo f (Pu) :::::
4

=-"3'

EJEMPLO 5. Hallar

el extremo

de la funcin

de dos variables

z= X'Jy2 (6-x-y).1
Determinamos

SOl.UCION.

los puntos

z~:-.'

J8X2y2

Zy'=

12x'ly -

4xay2 _

2~y

estacionarios;
3X2y3

3x3y2

O, }
O,

de donde XI -=. O, Y1
O y X2 = 3, yz = 2. Hemos obtenido dos puntos
estacionarlos Pi (O, O) Y P, (3, 2).
Calculemos las segundas dertvadas de la funcin:
-r :

z!J

=
=

Z;y

z~x

36xyz - 12x'Jy?' - 6xyS,


12.,,~- 2xt - 6x'Jy
3Gx2y - 8x3y _ 9x:!y2.

En el punto PI tenernos z;x = z" y .= z;., = O de modo que


z~:x:zZY - (~)2 = O y queda pendiente eY probleIna sobre la existencia
de extremo en este punto; para resolverlo habr que recurrir a las
derivadas superiores.
En el punto Pa tenemosz;'x = -144, Zyy = -162 Y z;;1I = -108.
Queda claro que ~xZY. - (zXI/)2
O Y como z;'x < O, en el punto
P" l3, 2) hay maxrmo siendo zmx = 108.

>

Hallar tos mximos y los mnimos de las funciones:

f = (x - 1)2 - 2y2.
3. f = X4 + y4 - 2Xll + 4xy - 2y'l.
4. f = (x2 + !f2) e-(x2+y~}.
2.

5.

I+x-y

VI +x2+y2.
y2

Z2

6. f=x+--x+y-+-z(x>O,
7.

f
f

x1. -

xy

+ y2 -

2x

sen x sen y sen (x


9. f """"XIX~ x~ (l 8.

(Xl>

0,

X2>

y>O, z>O}.

+ y.

+ y)

(O ~ x ..::;;;n, O ~ y ~

Xl -

2X2 -

O, ... ,

XII

. . . -

> O).

nxn)

;1)

EXTREMO

1.

INCONDICIONADO

J7

10. Demostrar que la funcin z = (J + e/l) cos x - yeY


tiene una cantidad infnita de mximos y no tiene mnimos.
11. Ser condicin suficiente para que la funcin f (x, y)
tenga mnimo en el punto Mo (xo, Yo) el que esta funcin
tenga mnimo a Jo largo de cualquier recta que pase por el
punto MI)? Considerar la funcin f (x, y) = (x - y2) X
X (2x _ y2).

12. Demostrar que (a diferencia de las funciones de una


variable) incluso para las funciones de dos variables la existencia de un extremo nico - mximo o mnimo - en un
recinto D no significa aun que este extremo represente el
valor mximo o mnimo de la uncin respecto a todo el
recinto. Considerar los ejemplos:
a) z = X2 - yZ + 2e-.xa,
-00

b) z = XS -

4Xll

<x<+

00, -00

<y <+

00;

2xy _ y2,

D {-5 ~ x ~ 5; -1 ~

y ~

1}.

13. Sea f (x) una funcin peridica con perodo 2:rt. Entre
todos los polinomios trigonomtricos de grado n
n

a;

+~

(ah COS kx

+ ~hsen kx)

h=1

a"

determinar, escogiendo convenientemente los coeficientes


y ~k, aquel que ofrece el valor mnimo para el error cuadrtico definido por la igualdad

!=

2~

re

-n

k=1

1 [f(x)- i -~(akcoskx+~ltsenkx)ydx.

2. Mtodo del gradiente. Supongamos que es preciso hallar el


mfnimo de la funcin f (x). donde x = (Xl' X2
xm). Tomemos
un punto f) = (xf, xU... , xg.) y calculemos el gradiente de la luncn
f (x) en este punto
m
grad

f (XO) = ~ a~~~} el.


i-l

donde eh ez... , en es una base ortonorrnal


S se tiene gr ad f (xO) :;= O. ponemos
xl = xi - h1 (grad t (f). eh) (k
2-01387

del espacio R",

1, 2, ...

m),

18

GAP. 1. EXTREMO

donde h1 > O es suficientemente


ponemos

DE FUNCIONES

pequeo. Si se tiene grad

en general, si se tiene grad f (xn-l) *0,


x~ = X~-l - h,. (grad

t (~..1)*O,

ponemos
(xn-l), eJt)

(k = 1, 2, .. , m; hn > O).
As obtenemos, si se cumplen determinadas condiciones, una su-

cesin montona decreciente {I (xn}). Si xn - X y


es un punto de
mnimo de la funcin f (x), se tiene grad f (xn) -+ O cuando n -+ oo.
JEMPLO 6. Hallar el punto de mlntrno de. la funcin f (x) = x'.
SOLUCIN. Tomemos, por ejemplo. el punto xO = 1. Tenemos

gr ad f (xO} = 2xOI = 21
O.
Por eso, ponemos
xl- = xO - h 2 = 1 - 211. donde h
Tenemos ahora
grad f (Xl) = 2 (1 - 211) l.

* !'

Si h

es grad
x?

Continuando

f (Xl)

O.

y ponemos

2h (1 -

Xl -

>

2h)

(1 -

2h)2.

este proceso. encontramos


xn

Es claro que, siendo 0<

(1 _ 2h)n.

h < 1, se tiene

xn - O para Il_ oo. El


f (x) = XS.Si" = -}.

punto x = O es el punto de mnimo de la funcin

es Xl = O y gead f (xt) = y obtenemos la sucesin estacionaria {O}


cuyo lmite es el cero.
EJEMPLO 7. Hallar el punto de mlnrno de la funcin f (x, y) =

= x:A
XO

+ y2.

SOLUCIN. Tomemos. por ejemplo, el punto (l. 1), o sea, tomemos


1 e /jo = 1. Tenemos

grad

(1, 1) = 21

+ 2j.

Puesto que grad T (1. 1) *0,


Xl

yl
Tenemos

= xO _
= !l -

ponemos
1 - 2h,
2!lh
J - 2h.

2xo"

=
=

grad f (xl, yl)=2(1-2h)l+2(1-2h)j=ty. por eso, tomamos


2x1h = (1-- 2h)2,
y2=y1_2ylh=(I_2h)2.
x2 = xl_

(h

>

O)

EXTREMO

2.

19

COND1CIONADO

Continuando

este proceso, encontramos


xn = (1 - 2h)U,
Un = (1 - 2h}n,
de modo que para O < h < 1 obtenemos una sucesin de puntos
Mn (x"", ",11) convergente al punto M (O, O) de rnlnmo de la [uncin
considerada. Es obvio que
grad f (xn, un) = 2 (1 -_ 2h)n 2 (1 - 2h)'n j -+ O cuando n -.. oo ,
Es decir, el punto (O, O) es el punto de mnimo de la luncin { (x, y) =

;:: x~ +

U2

Empleando
de la Iuncln

el mtodo de gradiente,
z

= x2 + y'J. _

2x

hallar el punto de mnimo

+ 4y + 5.

2. Extremo condicionado
Sea z = 1(Xl' X2
Xn) una [uncin de n variables definida de
un recinto D del espacio En.
Supongamos. adems, que las variables Xl. X2> , '"'71 estn
ligadas por (m < n) condiciones complementarias

:1_<X~' .x~':.:' "~~)~~'}


C>m

(1)

(x "2, ... , xn) = 0,

que se denominan

ecuaciones de enlace.
Sea %(0) = (xi, x~... , x~) un punto interior del recinto D.
Se dice que f (Xl. Xs, , xn) tiene mximo condicionado (mnimo
condicionado, respectivamente)
en el pun to (x~, ~ ... , x~) si la desigualdad

(Xl. X2.

Xn) ~

(x~. x~, ...

xi\.)

desigualdad f (Xl. Xa, ... , Xn) ~ f .(xY. X~, ' . X~), respectvarnense cumple en una vecindad del punto (.x~, x~, ' . _, x~) siempre que
puntos (Xl' X2
, Xn) y (X~. X~
, xV.) verifiquen las ecuaciones
enlace (l).
EJEMPLO 1. La funcin z = X2
yo, tiene mnimo incondicionado,
igual a cero. en el punto (O, O). Agreguemos la ecuacin de enlace
x + !I - 1 = O; se trata entonces de determinar el rnnirno de las
Z-coordenadas de los puntos de la superficie z = x~ + y'J. considerando
s6lo aquellos valores de x y de y que satisfacen la ecuacin x
!I - l =
= O. Este mnimo condicionado no se puede alcanzar en el punto
(O. O) pues este ltimo no satisface la ecuacin de enlace. Resolviendo
la ecuacin de enlace x
U - I = O respecto a y e introduciendo la
expresin obtenida y = \ - x en la ecuacin de la superficie, encontramos z = x2.
(I - x)2.. o sea, obtenemos una funcin de una
{la
te)
los
de

variable.

Extremndola,

encontramos xer = ~ y

Zmlll

= ~. A partir
~*

20

CAP. 1. EXTREMO DF. FUNCIONES

Ycr = ;. El punto ( ;,

de la ecuacin de enlace, determinamos

;,

~ )

es el vrtice de la parbola que corresponde a la interseccin del paraboloide z = X2


y'l.. con el plano x
y - 1 = O.
Anlogamente se puede proceder en situaciones ms generales.
Supongamos que se busca el extremo condicionado de la funcin
z
f (x, y) sendo (jl (x, U) = O la ecuacin de enlace. Supongamos que
para los valores considerados de x y de IJ la ecuacin q> (x, y) = O determina !I corno una funcin unvoca diferenciable y = 'P (x). Sustituyendo y por 'i> [x) en la funcin I (x, y), obtenemos una funcin de una
variable x: z = f (x, 'l> (x = F (x). El extremo (incondicionado)
de
la funcin F (x) sera el extremo condicionado buscado de la funcin
f (x, y) con la condicin de enlace (jl (x, y) = O. En la prctica este
mtodo resulta poco cmodo ya que para ap licarlo es preciso resolver
la ecuacin (jl (xo y) = O respecto a una de las variables.
Para hallar los extremos de la funcin z = f (Xl. ;\:2, ... , Xn) con
las condiciones de enlace (1) se emplea el mtodo de los multiplicadores
de Lagrange.
MeTODO DE LOS MULTIPLICADORES
DE LAGRANOE. Supongamos que
1) las derivadas pardales
de primer orden de las Iunciones
f (Xi> x2, . , xn) y epi (Xi, X2, .. "Xn) (1 = 1,2, ... , m) son continuas
en el recin lo D;

2)

< n, srendo

el rango de la matriz

(~:;)

(i

J, 2, ...

, m;

1= 1,2, ... , n) Igual a ni en todo punto del recinto D.


Consideramos una funcin nueva (luncin de Lagrange)
.
<D .=-J+

111

-"':1
k.J

"'[<rio

1.-1

donde "" son factores constan les indeterminados.


Despus analizamos el ex tremo incondicionado
de la uncin
<f) (XJ, xz, ....
xni, o sea. formamos el sistema de ecuaciones
J<I>
o;,tJ
o<l>
-,,- = O, -a = 0, .. , -:;-- = o
(2)
UXi

vXn

'1, a partir de este sistema y de l as m ecuaciones de enlace


(PI = 0,
tpll = O, ..
IPm. = O,
'0

determinamos los valores de los parmetros "lo 1..2 , Am y las coordenadas (Xl. X20 . , xn) de los posibles puntos de extremo.
Las condiciones (2) son condiciones necesarias de extremo tanto
para la funcin de Lagrange como para la [uncin inicial z =
= f (Xl' X2, . , xn)
Si el punto (4 X~, ..
x~) es un punto de extremo condiconado
de la [uncin l {Xl' XZo'
., xn l. ser a la vez un pun lo estacionario
de
0

1a funcin de Lllgr auge, o sea, en este pun to ser


...

, tl). Para analrz ar el punto estacionario

diJ ~

'<i

O (i

(xY. x~, ...

1, 2, ...

XYI) en tanto

2.

EXTR.EMO

CONDICIO"lA.DO

21

que ex tremo condicionado de la Iuncn de Legrange


habr que considerar

(1)(Xi' X2, ..

xl1)

la Iorma cuadrtica
n,.. tn

B(dxt.

, lLl:n-m}= ~ /JIJ,1x dx],


i. ;-1

dX2...

o sea, la segunda diferencial


cuenta las condiciones
01'1
-- dXt
v,\t

oq>'
+T
X2
v

dx,~

(3)

de la funcin de Lagrange, teniendo en


(i= 1, 2, ... , m).

".

Si la lorma cuadrtlca (3) es definida, en el punto (x~. xg, ... , x~)


tendremos extremo condicionado estricto; a saber, mxime condicionado estricto si la forma cuadrtica (3) es definida negativa, y mlnimo
condicionado estricto si la forma cuadrtica l3) es definida positiva.
En cambio si la forma cuadrtica (3) es indefinida, en el punto
(x~, x~, ... , x~) no habr ex tremo condicionado.
Por consiguiente, la existencia en el punto (x.,. x~, ... , x~) de
mximo (mnimo) incondicionado de la funcin de l,agrange (tomada
con los valores encontrados para)...1>Aa, .. " ~'nl)implica la existencia
de este punto mximo (mnimo) condicionado de la funcin z =
f (Xi' X2, , Xn) con las condiciones de enlace
<Pi (Xl. X2 .
XII) = O
(l = l. 2 ..
" m).

La ausencia de extremo incondicionado de la funcin de Lagrange


el> (Xl. Xa ..
, xn) no significa an la ausencia de ex tremo condicionado
de la (uncin f (XI. XZ, Xn):
EJEMPLO .2. Hallar el extremo de la funcin z = xy con la condicin y - x = Q.
SOLUCION. Formamos la funcin de Lagrange

<D

(x, y)

xy

+ '"(y -

los posibles

puntos

x)

para deterrnlnar 'k


de extremo

y el sistema correspondiente

'j

las coordenadas de

~~ =Y-'k=O,}
ay =x+1..=O,
y-x=O.
O<D

La primera ecuacin da A = y. Tenindolo en cuenta, encontramos la


segunda ecuacin x + y = O. Es decir.

x+y= O, }
y-x=O,
de donde x = y = O y. adems, A = O. Por lo tanto. la (uncin correspondiente de Lagrange es <D (x, y) = xy. La funcin <D (x, y) no tene
extremo Incondicionado en el punto (O. O).

CAP. l.

22

EXT"REMO DE PUNCIONES

Sn embargo, existe el extremo condicionado de la runcin z = xy


con la condicin y = x: en efecto, tenemos en este caso z = x', de
donde resulta que hay mfnlmo condicionado en el punto (O, O).
EJEMPLO 3. Hallar el extremo condicionado de la {uncin

f (x, y, z) = xyz
con las condiciones

<PI

(x, !I,

z)=x+y-z-3=O,

<r2 (x, y, z)=x-y-z-8=O.

(4)

Formamos la funcin de Lagrange


ct> (x, y, z) = xyz "'1 (x
y - z - 3) + /..9 (x - y - z y el sistema de ecuaciones para determinar los parmetros At
y Ias coordenadas de los posibles puntos de extremo
SOLUCION.

8)
y Aa

=YZ+/..I+~=O, )

: ~-"+'I-"~O,

(5)

T=XY-Ai-~=O'I
x+y-?-3=0,
x-y-z-8=O

..

Resolviendo el sistema de ecuaciones (5), obtenemos


11

Al ='"32 ,

231

11

~=-32'

la segunda dllerencial de la uncln


(}2(Jl

d2<.D
=--

i}.x'Z

y=-y

x=-;-,

i}zlD
dxz+ --dg2
ayZ

)za.

+-- iJz2

(!>

oz2

iJ2(D

(x, y, z) es ig\lal a

+
~$

+2-ax vy
;, dxd!l+2~dxdz+2-a
<i!l oz

En nuestro caso,
tl2cD = 2z dx dy
2y dx dz
2x dy dz.
Oc las ecuaciones de enlace (4) encontramos

dx+dy-dz=O,
dx-d!l-dz=O,

de donde
obtenemos

dx = dz, dy

= O.

Introduciendo

8 (dx)

~$

'" dydz.

y oz

(6)

estas expresiones en (6),

2y dx2.
- 5 dx2 <O, o sea, en el punto
.
605

En el punto estacionario se tiene B =


11
( T'

11

z=-4'

5
11 )
-'2'
-T
hay mximo, Siendo / mAx= 32.

2.

EXTREMO
EJEMPLO 4. Hallar

CONDICIONADO

de 13 funcin z

el extremo

('(>:;2 X

-1- cos2 !I

con la condicin

y-x=T'
SOLt:C1N. Formamos

de Lagr ange

la funcin

(y-x-:)

tD(x. y)=cos2..-.;+cos2y+]"

y el sistema de ecuaciones para determinar el parametro


deuadas de los posibles puntos de ex tremo
~~ ~~-2cosxsenx-;\-O,
al11

i)y

-v-:

,<

'i la~ coor-

-2 cos y sen v+ ;'=0, ~

y-x--=O,

es decir,
sen 2x= -A,
sen 2y=A.

(7)
(8)

y-x=T'

(9)

De las ecuacloncs (7) y (8) tenemos sen 2x


sen '2y =. 0, o sea,
(10)
2sen (x
y) cos (y - x) = {l.

Debido a (9). tenemos cos (y resulta

sen (x

Resolviendo

+ y) = 0, es
x + y ~ kn,

las ecuaciones

x)

= }~~=fo

y,

JOf

eso, de (10)

decir,
k = O, 1, 2,

(9) y (11), tendremos

Determinamos las se,:ulldas derivadas de la Iuncln IlJ (x, y):


1]2(!l
--=-:--"2

ox = - 2 cos 2x,

i.J2tD
ay2

En los puntos

=-

kn
n
PIi. ( 2-'8'

t)2q)
-;--;;-

ox cJy =O,

2 ces 2y.

krr.

n )

'2+8

se tiene

(11)

CAP. J. EXTPEMO

24

k1f. -

(!);:t(!)~V _(tl>;1/)2 = 4 cos (

DE FUNCIONES

) ces ( kn

+ ~)=
= 2 cos 2kn = 2>

O.

Por consiguiente, hay extremo condicionado en los puntos Pilo Adems.


para k:= 2n es

aiJx2Cl>2
y.

pOC I!SO,

P2n

-V2<O

en los puntos P-an se tiene mximo condicionado

..'"max-_1+V2.
-2para x= 2n+ 1 es

a2Cl>
7fT
X

J)zn+l

siendo

= V-2>0,

o sea. en los puntos Ptn+ 1 tenemos mnimo condlconado siendo

V2

zmln= 1--2-,

En los problemas que siguen hallar el extremo condicionado.

f = xy siendo x'J + .112 = l.


15. f=x2+!l' siendo ~ + ~= l.
16. f = xyz siendo x + y + z = 5 Y xy
17. f = eXY siendo x + y = a.
18. f = 6 - 4x - 3.11 siendo x'J. + .112 =
t 4.

19.

= x -

2.11 + 2z siendo

X2

+ +

sen x sen y sen z siendo x


y> O y z> O.
21. Demostrar la desigualdad
20.

n~

1,

y2

+ yz + zx

= 8.

1.
ZZ

= 9.

+ y + z = ~, x>
x~O

e y~O.

O,

2.

E XTR,EMO

CONDICIONADO

25

22. Hallar el valor mximo del producto xyzt de cuatro


nmeros no negativos x, y, z y t si la suma de los mismos
permanece constante: x + y + z + t = 4c.
23. Hallar la distancia mnima del punto M (1, O) a la
elipse 4x2 + 9y2 = 36.
24. Hallar la distancia de la parbola y = XZ a la recta
x-y = 5.
25. Hallar los lados del rectngulo de rea mxima
inscrito en la circunferencia X2 + y2 = R'J,
26. Inscribir en la esfera de radio R el cilindro de mxima
superficie total.

Captulo JI

EXTREMO DE FUNCIONALES

3. Funcional. Variacin de una funcional y sus


propiedades
1. Definicin de funcional. Proximidad de curvas. Sea M una
clase de funciones y (x). SI a toda {uncin y (x) E M le corresponde,
segn una regla, un nmero determinado J se dice que en la clase M
est definida la funcional J y se escribe J = J ly (x)].
La clase M de funciones y (x) en la que est definida la funcional
J
(x)] se denomina campo de definicin de la funcional.
EJEMPLO 1. Sea M = e [0, Ir el conjunto de todas las [unciones
continuas y (x) definidas en el segmento [0, IJ Y sea

ry

j y(x}dx >.

J ly(x}1=

(1)

o
Entonces J Iy (x)} es una funcional de y (x): a toda [uncin y (x) E
E [0, 1]le corresponde un valor determinado JIu (x)]. Tomando en
(l) funciones concretas en lugar de y (x), obtendremos los valores correspondientes de J r"l. Por ejemplo, si y (x) = 1, tenemos

J[l)=) ldx=!;
O

si y (x) = e, tenemos

J (eX)

=)

eXd.x=e-l

si

y (x) =cos nx,

tenernos
1

J(cosnx)=

Jo

cosnxdx=O.

1) En adelante, al considerar funcionales in tegrales, escribiremos


en el integrando y en lugar de y (x), y' en lugar de y' (x), y etc.

S.

FUNCIONAL.

VA~JACJON

DE

UNA

FUNCIONAL

27

EJEMPLO 2. Sea M = el [a. b) la clase. de funciones y (x) que


tienen derivada continua en el segmento [a, b) y sea
J [y (x)l = /j' (xo) ,
donde Xo E la, b.
Queda claro que J [y (x)J es una funcional deinl da en la clase de
funciones sealada: a toda funcin de esta clase le corresponde un
nmero determinado, el valor de la derivada de esta funcin en el
punto fijo xo.
Siendo. por ejemplo, a = 1, b = 3 y xo = 2. tenemos para
y (x) = X2

para IJ (x) =

+1

X2

+ x)

tendremos J
Sea M = e
tinuas en el segmento [-l.
continua para todos los -1
de y. Entonces.
In (1

EJEMPLO 3.

+ 11 = 4

J {xZ

encontramos

I
l+.x

Y para

I x-Z='3'

Y (.~)

{In (1 + x) =
[-l. l] la clase de funciones y (x) con1)Ysea cp (x, y) una funcin definida y
::;;;;
x::;;;; 1 Y para todos los valores reales
1

J [y (x)) =

j cp (.~,

y) dx

-1

ser una funcional

definida

ejemplo, si cp (x, y)

de la clase de funciones indicada.

1 ~yz'

para

Por

la funcin !I (x) = x tendremos

J!x]=

Ji

x dx

=0 y para y(x)=I+x

l+x2

-1

tendremos
I

1[I+x)=

x dx

1+{I+x)2

In

V5-

arctg 2.

-1
EJEMPLO".

Sea M =

derivadas continua

yl (x)

el

[a, b) la clase de funciones y (x) que tienen

en el segmento

{a.

bl. Entonces

Jfy(x)]=

VI+y'2dx

(2)

ser una funcional definIda en esta clase de funciones. Desde el punto


de vista geomtrico, la funcional (2) representa la longitud del arco
de la curva !J = y (x) cuyos extremos son los puntos A (a. y (a) y
B (b, y (b.

de

kl Se denomina

la funcional

variacin o incremento y (x) del argumento y (x)


J (y (x)lla diferencia entre dos funciones y (x) e Yo (x)

28

C!\P.

11. EXTREl>lO

DE FUNCIONALES

pertenecientes a la clase considerada

ay (x)
Para la clase de funciones

= y (x) k

M de funciones:
Yo (x) 1).

veces dtercncrables tenemos


= 6y(lI) (x).

(<'3y)(II)

Diremos que las curvas {/ = y (x) e !I = 9, (x) definidas en el


segmento la, bl san cercanas en. el sentida de proximidad de orden nulo
si es pequea en la. bl la magnitud I !I (x) - Yl (x) l. Desde el punto
de \' ista geo mt rico. esto significa que son prximas las ordenadas
de dichas curvas en la. bJ,
Diremos qu~' las curvas y = !I (x) e y = {/l (x) definidas en el
segmento la. bl son, cercanas en el sentido de proximidad de primer orden
si son pequeas en [a. bl las magnitudes I !I (x) - Yl (x) I y
I y' (x) - y (x) I Desde el punto de vista geomtrico, esto significa
que en la, b 1 son prx imas tanto las ordenadas de dichas curvas C0l110
las direcciones de sus tangentes en Jos puntos correspondientes.
Las curvas 11=. !I (x) e y = Yl (x) son cercanas en el sentido de
proximidad de k-simo orden si son pequeas en [a. bl1as magnitudes
I y (x) - Yt (x) J, I y; (x) - Yi (x) 1. ... , I yl"-) (x) - ylI:) (x) l.
Si las curvas son cercanas en el sentido de proximidad de k-simo
orden, con mayor razn lo sern en el sentido de proximidad de cualquier inferior.
11
EJEMPLO S La curva 1/ (x) = sen n x con n suctenternen te grande
rt
Y la cUf\'a!JI (x) ~. O son cercanas en [O, n] en el sentido de proximidad
de orden nulo ya que
ly(x)-yJ(x)l=

senn2 x
rt

~n'

o sea, el valor absoluto de esta diferencia es pequeo en todo el segmento [0, ni si n es sucenternente grande.
No hay proximidad de primer orden ya que

I y' (x) - u (x) I = n I cos n'J.x I

!~

ejemplo, ('11 los puntos x =


tendremos I y' (x) - Y (x) I "",
o sea, I y' (x) - y; (x) I puede resultar tan grande como se quiera
si n es suf iclentemento grande.
EJEMPLO 6. La curva y (x) = se~2nx con n sullctentemente
grande
y la curva Yl (x) ..",. O SOI1 cercanas en (0, n] en el sentido de proximidad
de primer orden ya que. tanto
I
I y (x)-Yt (x) 1= sen/12nx <fj2
y, por

=. 11,

1) Para abreviar,
en Jugar de By (x).

en lo que sigue escribiremos

simplemente

, 3.

FUNCIONAL.

VARIACiN

DE

UNA

29

FUNCIONAL

como

I y , (t) - Yl (x) 1=lcosl1xII


-11<: n
son pequeos.

Determinar el orden de proximadad de las curvas en los


problemas que siguen.
21. U(x)=

28. y(x)=

cos nx
n2+1
sen x
11

29. U(x)=sen-

x
11

en [O, 2nl.

Uf (x)

Y. (x) = O en

Ut (x) =0

55!

[O, nI.

en [O, 1J.

Se denomina distancia entre las curvas y::::: y (x) e y = Yl (x)


(a =o;;; x =o;;; b), donde y (x) e 111(x) son funciones continuas en [a, bl.

el nmero no negativo p igual


al mximo del mdulo I Yl (x)- 11 (x) I en
el
segmento
a~

x~

M (l. 1)

b:

P= P [Ul (x), U (x)] =


= mx Iys(x) -y (x) l.
o::>'~b
7. Hallar la distancia
p entre las curvas
y (x) = x e !/l (x) = Xi en el
segmento [O. I J (Hg. 1).
SOLUCIN. Segn la definicin p = mx t
1, osea,
EJEMPLO

O::;x~ I

P=

mx

)(

r-x

Fg. I

(x - x'). La Iun-

O::>':I<~l

cin y = x - x' se anula en los extremos del segmento 10. 1].


Determinemos el mximo de la uncn y = x - X2 en el segmento

te,

l}.

Tenemos

y' = 1-

2x e

v' = O

para x = "'2

de modo que
p= mx Y = <X-X2)1
O~x~t

:1;-

'2

=+.

En los problemas que siguen hallar fa distancia entre


las curvas en los segmentos indicados.

30

CAP.

30. !J (x)

11. EXTRF.MO

DE FUNCIONAI,F.!;

== O en

Yt (x)

3.. Y (x) = sen 2x

!1i (x) = sen x

32. y (x) = x

e !J1

xe:"

(x)

[O, 2].
en

In x en

[o, ~ l
rrl, ej.

Supongamos que las curvas y = y (x) e y = 111 (x) tienen en el


segmento la. bl derivadas continuas de orden n.
Se llama distancia de n-simo orden entre las curvas y = 11 (x)
e y = 111 (x) el mayor de los mxl mas de 1as exprestones
, Yl (x) - y (x) ,. 1 y; (x) - y' (x) 1, ... , 1 y\n) (x) - ycn) (x) 1
en el segmento [a, bl. Representemos esta distancia as

Pn

Pn IYI (x). y (x) =

rnx
O~/:'.f.n

mx

a~x:'.f.b

I yiR) (x) - y(ll) (x) l.

Desde este punto de vista se puede interpretar la distancia definida en


la pg. 29 como distancia de orden nulo.
EJEMPLO 8. Hallar la distancia
de primer orden entre las curvas
y (x) = x' e 111 (X) = x3 en el segmento O ~ x ~ l.
SOI.UCIN. Calculemos
las deriv adas de las funciones dadas:
y' (x) = 2x e Jl. (x) = 3X2 y consideremos las funciones y, (x) =
= Xi - x3 e IJI (x) = 2x - 3Xil. Determinamos sus mximos valores
en el segmento lO. 11. Tenemos y; = 2x - 3x2.
y

fig. 2
Igualando

esta derivada

narios de t a funcin Jl2


yll

a
:t'~-

=..i..

27

(x) :

y el valor

.tl

a cero, encontramos

los puntos

estacio-

O y Xz = ~. i\hora bien, Y2 Ix=-o = O,

de Jli (x) en el extremo

de la derecha

es

a.

FUNCIONAL.

Ya (1)

= O.

VARIACiN

UNA

31

fUNCIONAL

Por eso,

Determinamos ahora la distancia


/j' (x)

DE

= 2x e

Yi

(x)

= 3xs:

Po = mx I Y3 (x)
O~%~t

Po

de orden nulo entre las derivadas

1= mx I 2.x-3x2 1O~x~i

Consideremos el grfico de la funcin y, = I 2x - 3xs I (ligo 2). Puede


verse de l que Po = l. Por consiguiente, la distancia Pl de primer
orden entre las curvas y (x) = Xi e Yl (x) = r ser igul a
PL

mx (Po, po)

l.

33. Hallar la distancia de primer orden entre las curvas


y (x) = In x e Yl (x) = x en el segmento
l, ej.
34. Hallar la distancia de segundo orden entre las curvas
y (x) = x e Yl (x) = -cos x en el segmento [0, ~ ]
35. Hallar la distancia de lOOl-simo orden entre las
curvas y (x) = eX e Yl (x) = x en el segmento [0, I J.

re-

Se llama

(a ~ x:S:;;; b) el
n-strno orden

e-vecindad de n-simo orden

d.e la

curva

!I

= IJ (x)

conjunto de las curvas y = 1J1 (x) cuyas distancias de


a la curva y = !I (x) son menores que e:
Pn = Pn llJ (x), Yl (x)) < e,

La s-vecindad de orden nulo se denomina e-vecindad fuerte de la


y = y (x).
La e-vecindad fuerte de la curva IJ = Y (x) est formada por todas
las curvas comprendidas en la franja de 28 de anchura construida a
partir de la curva y = y (x).
La s-vecindad de primer orden se denomina e-vecindad dbil de
la funcin. 11= Y (x).
2. Continuidad de una funcional. Una funcional J I.IJ (x}) definida
en la clase M de funciones y (x) se llama contlnua en y = Ye (x) en
funcin

el sentido de proximidad de nsimo orden si cualquiera que sea el


nmero 8 > O existe un nmero "l > O t-al que la desigualdad
I J [y (x)] - J l.!Io (x)] I < 6 se cumple para todas las funciones admisibles y = y (x), o sea, para todas las funciones que satisfacen las

condiciones

I !I (x) - !lo (x) I < "l.

I y' (x) - Y (x) I < 11. . ..


. . " I y(n) {x} -

En otras parabras, si se tiene I J ly (x)l {In [y (x),

!lo (x)1

J Iyo (xH I

< 1).

y~n)

(x)

< e siempre

< 1'].
que

CAP. TI. EXTREMO

32

De FUNCIONALES

Toda funcional que no sea continua en el sentido de proximidad


de Ilsimo orden se denominar discontinua en este sentido de proxlmldad. Poniendo
(x) = y~ll) (x)

/j("')

donde

<X

+ <xw(k)

(x)

(k = O, 1, 2, ...

, n),

es un parmetro y w (x) es una funcin cualquiera de la clase


de que

M, podemos persuadirnos
Im

ylh}

(x)

a;... 0.l

y~k}

(x)

(k

.. , n)

O. 1,2,

y, por eso, podemos denir la continuidad de la funcional J fy (x)]


en y = Yo (x) de la forma siguiente
lim J [Yo (x) <xw (x)) = J {Yo (x).

a...O

Demostrar que la funcional

EJEMPLO 9.

t
l'

Jly(x)l= ~ (1I+2y')dx

o
considerada en el espacio CI 10, 1( es continua en la funcian Yo (x) = x
en el sentido de proximidad de primer orden.
SOLUCIN. Tomemos un nmero cualquiera e > O y demostremos
que existe un nmero r> O tal que I J Iy (x)) - J [x) < e siempre
que I IJ (x) - x I < '1 Y 1 y' (x) - I 1 < 'l. Tenemos
1

IJ[I/(X))-J(X]I=I

~ (Y+2yl-X-2)dx

l<
t

~ Jo
e
Tomemos r=3'

J Iy'-lldx.

Jy-xJdx+2

Entonces, para todas las funciones y (x) ECdO, II

tales que

Iy
tendremos

1< 3

(x)-x

I.!I'

I J [y (x) - J [x]

Es decir, para todo

8> O existe

(x)- 1 1< 3

< 8.

un nmero

1')

>

O (por ejemplo,

que Pt Iy (x), x] <


< 'l. Pero, segn la definicin, esto significa precisamente que nuestra
funcional es continua en la funci6n Yo (x) = x en el sentido de prox-

r= ~) tal que I J (y (x)) - J [x] I

< 8 siempre

I 8.

FUNCIONAL.

VARIACIN

DE

UNA

FUNCIONAL

mldad de primer orden. Es fcil ver que esta funcional es continua e


el sentido de proximidad de primer orden en cual quier curva y (x)
E C, lO. 1).
EJEMPLO 10. Consideremos la funcional

= y'

I [g (x)}

(xo).

donde las funciones y (x) E C, [a, b] y Xo E la, b).


Esta funcional es discontinua en el sentido de proximidad de
orden nula en cualquier funcin 11 (x). Efectivamente, escogemos cp (x)
de modo que <p' (xo) = l Y que I <P (x)' < 11 en el segmento la, b}.
Consideremos la funcin y (x) = Yo (x) + <p (x), donde !lo (x) E
E la, bl. Entonces tendremos y' (xo) = li (xo)
l. Es obvio que
P (y (x), !Jo (x) < ", O sea. que las curvas y (x) e Yo (x) son cercanas en
el sentido de proximidad de orden nulo. Al mismo tiempo J Iy (x)) - J [Yo (x) = 1, es decir, los valores de la funcional no son prximos
por cercanos que sean, en el sentido de proximidad de orden nula. los
argumentos y (x) e Yo (x).
Hablando con ms precisin, existe un 8
O (por ejemplo, cualquier 8 < 1) tal que para cualquier"
> O existirn funciones y (x)
para las cuales
Po [y (x), Yo (x)} < 1] y, sin embargo, , J fu (x)! - J (Yo (x)] I ~ 8.
Esto significa precisamente que la funcional J (y (x)) es discontinua
en el sentido de proximidad de orden nulo.
Demostremos que esta funcional es continua en el sentido de
proximidad de primer orde t.
Tomemos un & > O cualquiera. Tendremos

el

>

I J (y (x)] - J
Queda claro

(Yo (x)

que tomando 11 =

= I y' (xo) -!l~ (xo) .


tendremos

I J [O (x)J - J Yo (x) I

<e

siempre que p, [y (x), Yo (x) J < r que es lo que se queria demostrar.


Este ejemplo permite ver que de la continuidad de la funcional en el
sentido de proximidad de n-simo orden no implica, hablando en trminos generales, su continuidad en el sentido de proximidad de orden
Inferior,
EJEMPLO 1). Consideremos Ia funcional
R

J 1.1>' (x)]

Jo

y''J. dx

definida en el espacio el [, n]. Demostremos que es discontinua en la


funcin Yo (x) == O en el sentido de proximidad de orden nulo.
_ sennx
En electo, sea Yo (x) ;: O en lO, nJ y sea {lit (X)
Enton-

= ---.n

ces, Po (Yo (x), Yn (x))


3-01381

::= -

n de modo que Po ~

O cuando

II ~

co,

CAP. 11. EXTREMO

34

DE FUNCIONALES

Por otro lado, la dferencia


11

J [Yn (x)\-J (Yo (x)] =

J\

cos2nx d
n
n
x=2"

no depende de fi. Es decir, J [Yn (x)) no tiende hacia J [Yo (x = O


cuando n -- 00 Y. por consiguiente, nuestra funcional es discontinua
en la {uncin Yo (x) == O en el sentido de proximidad de orden nulo.
Proponemos al lector demostrar que esta funcional es continua
en la funcin Yo (x) == O en el sentido efe proximidad de primer orden ..

Analizar la continuidad de las funcionales siguientes


36. J fy (x)J = y (xo), donde y (x) E C [a, bl y Xo E la, b),
en el sentido de proximidad de orden nulo
37. J fy (x)J = rnx I y (x) 1, donde y (x) son funciones
continuas en el segmento la, bJ, en el sentido de proximidad
de orden nulo.
38.
V si y (x) toma al menos un valor negativo,
J [y (x) =
{

-4- si y (x) == 0,
1 si U(x)~O

e y(x)90,

en el sentido de proximidad de orden nulo


1

I I

39. J (U (x)] = ) y' dx, donde las funciones y (x)

o
tienen primera derivada continua en el segmento [O, 11:
a) en el sentido de proximidad de orden nulo;
b) en el sentido de proximidad de primer orden.
n

J {y (x)J = ) VI + y'Z dx en la funcin Yo (x)

== O,

o
donde y (x) E CI [0, tt):
a) en el sentido de proximidad de orden nulo;
b) en el sentido de proximidad de primer orden

40.

J(

4 t. J fy (x)l =

j (1 +2y'2) dx en la funcin
o

Yo (x)

c;:;,s

0,

s.

FUNCIONAL.

VARIACiN

DE

UNA

donde y (x) E C; I, rr], en el sentido


primer orden.
EJEMPLO

12. Demostrar

FUNCIONAL

35

de proximidad de

que la funcional
1

Jo

J(y{x)]=

Vl+y dx

x3

definida en el conjunto de las funciones y (x) E e [0, 1] es continua


en la funcin yo(x) = x!- en el sentido de proximidad de orden nulo.
SOLUCION. Pongamos y (x) = r
enl (x), donde T) (x) E C (0, 1)
Y a. es tan pequeo como se quiera, tenemos

J [11 (x)]

= J [x2 +

a:f) (x}J =

+(x2+ a:1l)2dx =

xli VI

O
1

x3Vl+x4+2a:x2r+a.2rt2dx

Pasando al lmite cuando

O, obtenemos de esta igualdad

el -+

lm lJy(x)J=

a:..
lo que equivale

Yo (x) = xa

r x3V

1+~dx=J[x2}

JO

a la continuIdad

de la funcional

DEPINICION. Sea M un espacio


!I (x).

en la funcin

lineal normado formado por las

funciones

La funcional L [y (x)) definida


lineal si satisface las condiciones
1)

L {ey (x))

donde e es una constante


2)

en el espacio

el: (y (x)J,
y

cualquiera

L 1111 (x)J

M se denomina

+ v (x)) =

L [Yt (x)J

+ L (Y:a (x)l,

donde 111 (x) E M e 112 (x) E M.


Por ejemplo, la funcional
b

L [y (x))

W+y)dx

definida en el espacio Cl

la,

b] es, obviamente,

lineal.

AP.

TI. EXTREMO

DE FUNCIONALES

Existe otra definicin de funcional lineal:


La funcional L Iy (x) 1 se denomina lineal si 1) es continua y 2) satisface la condicin

+ Y'l; (x) = L (Yl (x)] + L [g,

L [Yl ():)

que sean 9, (x) E M e Y2 (x)

cualquiera

(x)]

E M.

42. Demostrar la equivalencia de las dos definiciones de


funcional lineal.
43. Demostrar que la funcional L [y (x)1 = y (xo) es lineal.
44. Sea L [(y (x)l una funcional lineal. Demostrar que si
la razon
' L11 y(x)
(y(x)J
.
- O.
JI -+- O cuando
11y (x) 11-+ 0, es L ry (x) ] =
3. Variacin de una funcional. Sea J

Ly (x)1 una funcional defini-

da en el conjunto M de funciones y (x). La magnitud


t::.J

(6y

= y'(x)

se denomina

s:

(11 (x)) = J [y (x)


ay] - J [y (x)]
- y (x), donde Ij (x) E M e (x) E M)

incremento de la funcional J {y (x)] correspondiente

incremento 8y del argumento.


EJEMPLO 13. Hallar el incremento

al

de la funcional

[y

(x}) "'"

J y!!' dx
o

definida en el espacio el [a,


SOLUCIN. Tenemos

bJ

si 11 (x)

M=J(x2}-J[x]=

=x

e !h (x) = x'.

x22XdX-)

x.l'dX=)

(2xS-x)dx=O.

45. Hallar el incremento de la funcional del ejemplo 13


siendo y (x) ex e Yl (x) = 1.
DEFINICIN. Si el incremento

llJ = J [y
se puede representar

(x)

J [y (x)]

de la funcional

+ 6yl -

J [y

(x)]

en la forma

Al = L [y (x), ay)

+ ~(y (x),

ay)

11

8y 11,

donde L {y (x), ayJ es una funcional lineal res pedo a ay y ~ (y (x), 6y)- o cuando !l 6y 11 - O, entonces la parte del incremento lineal respecto a ay, o sea, L [y (x), yJ. se llama variacin de la [uncional y se

FUNCIONAL.

VARIACION

DE

UNA

37

FUNCIONAL

representa por (jJ. Se dice en este caso que la funcional J [y (x)) es


d [erenolabte en el punto y (x).

46. Demustrese que la variacin &1 de la funcional

1fy (x)l se determina unvocamente (si es que existe).


EJEMPLO

14.

Demustrese que la {uncional


b

J [y (x)]

!I d:z

definida en el espacio e
de este espacio.
SOLUCION. Tenemos

la,

b] es dierenclable

en todo punto y (x)

6.J=J (y (,r)+6yJ - J Iy (xl) =


b

(y+~y) d:r -

ydx=

J aV

ax

Es decir, l1J

J ay x,

Pero sta es una funcional lineal respecto


a
a (y. Todo el incremento se ha reducido en nuestro caso a una funcional
lineal respecto a 6y. La funcional considerada es diferenciable en todo
b

J 6y

es ! =

punto y (x) y su variacin

dx.

47. Demostrar que toda funcional 1 ty (x)] lineal continua

es siempre dierenciable.
EJEMPI.O

15.

Demostrar

que la funcional
b

J tu (x)l = ) y2dx
a

definida en el espacio e [a, b] es dilerenciable


SOLUCiN. Tenemos
b

6J=

J
o

(y+ay)2dx-)

b
y2dx.=

J
a

en todo punto !I (x).


b

2yOydx+

) {Oy)2dx.

(3)

En el ltlmo miembro de (3) la primera integral representa la Iunclonal lineal respecto a 611 cualquiera que sea-la funci6n fija y (x). Estl

CAP. 11. EXTREMO

38

DE FUNCIONALES

memos la segunda integral de este miembro. Tenemos


b
b

) (y)2 dx=
I y 12 dx
a
a

e
b

~(mx

Jagl)2

Q~x~b

Si

11

ay

r dX=(b-a)lIIIIIZ=((b-a)1I6YIIJlffSYII.

Ja

O, la magnitud
(b - a) 11 ay n -1- O.
hemos logrado representar el incremento llJ de la funcional
suma de L 111(x), y! y de una magnitud infinitsima de
orden con respecto a 110,,11. Segun nuestra definicin, la
considerada es dilerenciable en el punto 11 (x) Ysu variacin es

11 -

Es decir,
como la
segundo
funcional

6J = 2 )

y oydx.

a
t

J y'l.dx

48. En la funcional J (y (x)J =


y fJy =

CtXZ;

comparar /jJ y IlJ para

o
Ct

49. En la funcional J fy (x)l =

tomar y = 2x

= 1; -0,1 Y 0,01.

J x!ldx

tomar y = s: y

o
comparar tlJ y 6J para Ct = 1; 0,1 Y 0,01.
50. Analizar si son o no diferenciables las funcionales
siguientes:
1) J fu (x)] = y (a) en el espacio e la, bl.
2) J IU (x)J = y (a) en el espacio el fa, el,
3) J [y (x)] = VI + y'z (a) en el espacio el fa, bl.
4) J fy (x)l = I y (a) I en el espacio e la, bJ.
51. Demostrar que la funcional J'A [y (x)] es diferenciable
si lo es J fy (x)1. Hallar la variacin de J2 [(y (x)L
52. Sea F (x, y) una funcin continua de sus argumentos
con derivadas parciales continuas hasta de segundo orden
inclusive en el recinto a ::::;;;
x ~ b, -00 <y < + oo. Demostrar que la funcional
ay

CtX;

J !y (x)] = ) F (x, y) dx
o

3.

FUNCIONAL

VARIACION

DE

UNA

FUNCIONAL

39

definida en el espacio e [a, ir] es diferenciable y que su variacin es


b

<'J =
EJEMPLO

~
a

iJF~, y) oydx.
y

16. Consideramos

la funciona!
b

J fu (x)J =

F (x, y, y') dx

definida en el espacio el [a, b) de funciones y (x) que son continuas


en el segmento fa, bJ y (IUe tienen en l derivada continua de primer
orden. La funcin F (x, !I, y') es continua respecto a todos sus argumentos y tiene derivadas parciales continuas hasta de segundo orden
inclusive en el recinto

-00 < y < +00, -00 < y' < +00.

a ~ x ~ b,

Determinamos el incremento IlJ de la funcional correspondiente


Incremento y de) argumento siendo 6y E el la, bl, Tenemos

al

IlJ (y (x =

(F(x, y+ay,

y'+6y')-F(x,

y, y')Jdx.

(4)

(x, U. y', 6g. tly'),

(5)

Segn la frmula de Taylor


F(x. g+y, [1' +6y')-

F (x, y, y') =:

= ~: oy+

~:' ay' -1- R

donde R [x, Y. y'. f5y, 6y') es el trmino complementario


de Taylor. Introduciendo (5) en (4), obtenemos
b

Al (y (x}1=

j (~~
hy+ ::'

oy')

dx+ ) R (x,

y, y',

~!I,

hy') dx,

lineal respecto a
parciales de la
absoluto, de una
e !I'. Tendremos

J R (x,

y, JI', ~U, oy') I dx ~ 2M

(6)

El primer sumando en el segundo miembro de (6) es


6YIY 6y. SuponR'amos que las segundas derivadas
fundn FI.(x, y, y ) respecto a y e y' no pasan, en valor
constante M > O en un recinto acotado respecto a y
entonces

de la lrrnula

J IJ 6yll2
a

dx=2M (b-a) 11 y 112,

40

CAP.

donde

11

8y

11

11, EXTREMO

1_!l!, (1 6y

a..~x.~b

DE FUNCJONALES

l. \ 6y' \). Por conslguente,

el segundo

sumando del segundo miembro de (6) es una infinitslma de segundo


orden respecto a n 6y 11. Es decir, en virlud de la definici6n, la funcional J [y (x)1 es dlerericlable en el espacio el (a, b) y su variacin es
b

~J =
EJEMPLO

J (~~ ~y+~:' Oy')

(7)

dx

17. Hallar la variacin de la funcional


1

J [y (x)]

=J

dx,

(y'ell+xyZ)

-1

SOLUCION. La funcin F (x, y, y') = !leY


xy' es, evidentemente,
conlinua respecto a todas las derivables x, 11 e y' en conjunto y sus
derivadas pardales de cualquier orden respecto a y e y' son acotados
en cualquier recinto acotado de variacin de y. e y'. Por esto, la funcional considerada es derencable en el 1-1, 1) y, segn la f6rmula (7),
su variacin es

8J = ) [(y' ell + 2xy) Oy + ell5y' Idx.


-1

53. En la funcional

J [y (x)J =

j (y' y + xy''l.) dx
t

k(x-l)

tomar y = In x y 6y = e:=T ; comparar I1J y 6J para


k = 1; 0,1 y 0,01.

54. En la funcional
1

J [y (x)] = ) (XZy'2_!I)

dx

o
tomar y = x'J. y 6y = kX3; comparar I1J y tU para k
0,1 y 0,01.

== 1;

55. En la funcional J

ry (x}1 = 5 s"
o

sen x dx tomar

sen x y fJy = k cos x; comparar /.}} y 6J para k = -1;

0,3'

0,03.

p.

FUNCIONAL.

VARIACION

DE

UNA

FUNCIONAL

41

56. Las derivadas parciales de segundo orden de la funcin F (x, Zl. Z2, . , zm+t) respecto a todos los argumentos
son continuas en el recinto a =::;;;; x =::;;;; b Y - 00 <lit < + 00
(k = 1, 2, ... , m + 1). Demostrar que la funcional
b

J {U (x)]

F (x, y, lJ', .. , y<"'J) dx

es diferenciable

en el espacio Cm La, bl y que su variacin es

6J =

J~ [ ::

6y

+ :~ 6y' + ... + a:(~) o!lm)J dx.

4". Segunda definicin de la varIacin de una funcional. Se llama


J [y (x)] en el punto y = y (x) el valor que
toma en a = O la derivada de la funcional J (U (x) + a~!ll (constderada en tanto que funcin de a) respecto al parmetro a:

variacin de la [unctonai

6J = OIZ J [y (x)+a6yll~_o.
Si existe la variacin de la funcional en tanto que parte principal
lineal de su incremento (o sea, si existe la variacin en el sentido de
la primera definicin), tambin existe la variacin en tanto que valor
en a = O de. la derivada respecto al parmetro a y ambas variaciones

coinciden.

EJEMPLO 18. Empleando la segunda definicin. hallar la variacin


de la funcional
b

J [y (.~)] =

J JI"

dx,

a
SOLUCiN. La variacin de esta luncional en el sentido de la prmera definicin es
b

6J = 2

116y dx

(vase el ejemplo 15). DetermInemos la variacin de la funcional


J [y (x)] basndonos en la segunda delnicin. Tenemos
b

Jlv(x)+a6y}=

j (y+aBy)2dx.
o

CAP.

JI.

EXTREMO

DE FUNCIONALES

Por eso,
b

iJ~ JIY(X)-!-MY)=2)

(y+<xy)gdx
(1

de modo que

= iJ~

J fu (x)

+ <x~u] Ia.... o = 2

yy dx,

(l

Las variaciones de la funcional en el sentido de la primera y de


la segunda definiciones coinciden.

Para las funcionales que siguen hallar, en los espacios


correspondientes, sus variaciones en el sentido de la segunda
definicin.
e

57. J

lY (x)} =

J (x+

y) dx.

a
b

58. J{y(x)J=

J (yZ_y'2)dx.
a

59. J (y (x)] = y2 (O) +

J (xy+ y'Z) dx.


O

11:

60. J [y (x)l = ) y' sen ydx.

o
b

6t.

J(YII Uz, ... , Ynl=

F(x, YI' UZ, ... ,

Y'l! y;, y~,

.. , U~)dx,

donde F es una funcin continua de sus argumentos y sus


derivadas parciales respecto a lodos los argumentos son
continuas en un recinto acotado de variacin de los mismos.

OBSERVACION. La segunda deflnlctn de la variaci6n de una funcional es en cierto sentido ms amplia que la primera pues existen
funcionales que tienen variacin en el sentido de la segunda definicin
aun cuando no se pueda despejar la parte principal lineal en el incremento de las mismas. Para explicarlo recurrlremos a las funciones;

3.

FUNCIONAL.

VARIACIN

DE

UNA

43

FUNCIONAL

en este caso nuestra afirmacin equivale a que la exlstencla de las


derivadas en cualquier direccin no basta para la existencia de la diferencial de la [uncin.
Sea
xy

f (x, y)

donde p y q> son las coordenadas polares del punto (x, y). Las derivadas
pardales :~ y

!~

existen

en todo punto y son iguales a cero en el origen

el,

de coordenadas; sin embargo. no existe la diferencial


en el origen
de coordenadas. Efectivamente, supongamos que df existe. En este
caso el gradiente de la funcin f seria igual a cero en el origen de coor-

denadas y, por eso, la derivada df

~iO) en cualquier

direccin tambin

seria igual a cero. Pero es fcil persuadirse de que

dI

(O, O)
di

="2sen

2
q>

lo que, en general, es diferente de cero. Agul cp es el ngulo entre el


vector 1 y el ele Ox.
5. Segunda variacin de una funcional. Una funcional J [.r, y]
dependiente de los elementos x e!/ (que pertenecen ambos a un espacio
lineal) se denomina bilineal si es una funcional lineal en
para x
fijo y una funcional lineal en x para y fijo. O sea. la funciona J [x, 111
es bilineal si

J
J

= al)

(alX]
a2xS' U]
(x. ~1l(1
~2!1!al =

ljJ

[Xl, y]
[x. YtJ

+ aaJ

gl,

[X2'

+ ~2J [x, Y21.

Poniendo en la funcional bilineal y = x obtenemos la expresin


J {x, x] llamada funcional cuadrtica.
Toda funcional bilineal definida en un espacio de dimensin
finita se denomina forma bllineal.
Una Iunconal cuadrtica
J Ix, x] se denomina definida positiva
si J [x. x) > O cualquiera que sea el elemento no nulo x,
Por ejemplo.
1) la expresin
b

Ix, !/]=

A(t) x (/) g(t)dt,

donde A (/) es una funci6n continua

fija. representa

una funcional

bilineal en el espacio

e la. b] mientras

que la expresin

A (l) Xi (t) dt

(l

representa. en este mismo espacio, una funcional cuadrtica que resulta


definida postiva si A (t)
O para todo t E (a, ti);

>

CAP.

u.

EXTREMO DE FUNCIONALES

2) Ji' expresin
b

J (x, xl

(A (/)

1'2

(t)

+B (1) x (1) x' (/) +C

(t)

x'2 (/)]

dt

es un ejemplo de una funcional cuadrtica definida para todas las


funciones pertenecientes al espacio CI [a, bJ;
3) la expresin
h b

J(X,Yl=l

) K(s,t)x(s)y(t)dsdt,
a

donde K (s.t) es una funcin fija de dos variables, representa una runclona] blnea en CIa. b).
DEFINrerON. Sea J fy (x) ) una funcional definida en un espacio
lineal normado. Diremos que la funcional J [y (x)} tiene segunda
variacin si su incremento 6J = J Iy (x) + 8y1 - J [y (x)) puede
ser representado en la forma

s:

=L,

[~g)+2 ~I~y)+~ 11 ~y 1(2,

donde LI (6y) es una funcional lineal, L2 [6y] es una funcional cuadrtica y ~ -+ O cuando 1I 6y \1 -- O.
La funcional cuadrtica L2 [ay) se denomina segunda variacU5n
(o segunda diferencial) de la luncona J [y (x)] y se desIgna por 6"J.
La segunda variacin de una funcional se determina unvocamente
(S es que existe).
EJEMPLO 19. Hallar la segunda variacin de la funcional
1
J [y (x)

=)

(xy2+y'9)

dx

definida en el espacIo Cl [O, 1) de las funciones y (x).


SOLUCIN. Tenemos
6J = J (y (x)

+6$11- J [y (x)] -=-

[x (y+6y)'2+(y' +Oy')3_xy'2_y'3]

dx=

O
1

= ) 2.xyOy+x (6y)2 + 3y'20y' +3$1' (6g')'2


o
t

J
O

(2xy6y+3y'26y')

dx

J
O

(x (6y)2+3y'

+ (6y')3Jdx=

t
(611')'21 dx+ ) (011')3dx,
O

(8)

FUNCIONAL.

VARIACIN

DE

UNA

45

FUNCIONAL

Fijemos y (x). El primer sumando del ltimo miembro de (8) ser


entonces una funcional lineal respecto a 6y; el segundo sumando de
este miembro sera una funcional cuadrtica: finalmente, el ltimo,
tercer, sumando de este miembro puede ser estimado as!
1

I ) (6y')Sdx I~

(rnx 16y' 1)2

J 6y' I dx ~ (

Jo

16y' I

dX)

116y 112

(la norma se toma en el sentido del espacio t (0, 1)), o sea, podr ser
representado en la orma ~ 11 6y II~. donde ~ -+ O cuando 11 6y 11 -+ O.
Por definicin, para nuestra funcional existe la segunda variacin
fJ'J igual a
I

62J =2

(x{liy)2+3y'

(6y')2.) dx,

62.
tica.
63.
siendo
64.

Hallar
Hallar

la segunda variacin

de una funcional cuadr-

la segunda

variacin de la funcional
dos veces dferenciable.
Demostrar que las funcionales de tipo

eF(IIJ

F (y) una funcin

J [y(x)] =

J F (x, y. !/) dx
(1

definidas en el espacio eJ [a, bJ son dos veces diferenciables


si la funcin integrando F tiene derivadas continuas hasta
de tercer orden inclusive. Hallar la frmula para la segunda
variacin.

Consideremos la funcin a> (a) = [y (x)


aOy]. La segunda
variacin 6't.J de la funcional J [y (x)] se define tambin mediante el
valor de la segunda derivada de la funcin ID (a) en el punto a = o:

a21D (al
da'J

I~-o'

Para las funcionales de tipo integral (que predominarn


consideraciones) ambas deriniciones coinciden.

Hallar

la segunda variacin.

r
b

65.

J [y (x)J =

el

F (x, y,

ti, ... , y(ml)dx.

en nuestras

CAP. 11. I!XTRP.MO

66. J (z (x. !J) 1 =

Jf

DE PUNCIONALES

P (x. !J, z,

zx, z/I) dx

dy.

G
11

67. J [!JI' !J2, .. , !Jn] = ) F (x, !Jt, !J~,

... ,

... , y~) dx,


6. Extremo de una funcional. Condicin necesarla de extremo.
DIremos que la funcional J (y (x)] alcanza su mzlmo en la curva
y = Yo (x) si los valores que toma la funcional J [y (x)1 en cualesquiera
curvas prximas a y = Yo(x) no son mayores que J (Yo(x)1, o sea, si
.J

J [y (x)) -

J (Yo(x)} ~ O.

Si .J ~ O y tJ.J = O slo para y (x) = Yo (z), diremos que se


alcanza mximo estricto en la curva y = Yo (x).
Anlogamente se define la curva y = 110 (x) en la que se alcanza
un minimo, En este caso se tiene .J
O para todas las curvas prxlmas a la curva IJ = Yo (x),
EJEMPLO 20. Demostrar que la funcional

>

(y (x)l

=)

+ (2) dx

(x2

alcanza rnlnmo estricto en la curva y (x) s O.


SOLUCION. Cualquiera que sea la funcin IJ (x}. continua en [O, 1],
tenemos
I

= J (y (x)]

- J [O] = ) (x2

l'

+ (2)

dx - )

X2

dx =

J y2 dx #- O;
O

adems, el signo de Igualdad se da 5610 para y (x) == O.


EXTREMOS PUE1ITE y DeBIL. Oiremos que la funcional
alcanza su mdxlmo relativo fuerte en la curva y = Yo (x) si
J Iy (x)] :s;;;; J (!le (x))

J (y (x)J

para todas las curvas admisibles y = y (x) pertenecientes a una s-veclndad de orden nulo de la curva y = Yo (x). Anlogamente se define el
mnimo relativo fuerte de una funcional.
Diremos que la funcional J [y (x)l alcanza su mximo relativo dbil
en la curva Y = Yo (x) si
J [y (x) ~

J [Yo (x)1

para todas las curvas admisibles y = y (x) pertenecientes a una s-ve


cindad de primer orden de Ia curva y = Yo (x). Anlogamente se define
el mnimO relativo dbil de una /lmcional.

FUNCIONAL.

VARIACIN

DE

llNA

FUNC.IONAl

47

Los mx imos y mnimos (fuertes 'i dblles) de la funcional


J (y (x)l se denominan extremos relativos.
Todo extremo fuerte es al mismo tiempo extremo dbil pero no
viceversa.
El extremo de la funcional J (y (x)l referente a la totalidad de las
funciones en las que est definida 1a funcional se denomina extremo
absoluto.

Todo extremo absoluto es al mismo tiempo extremo relativo fuerte


y dbil pero no todo extremo relativo ser extremo absoluto.
EJEMPLO 21. Consideremos la funcional
1'1

J (JI (x) 1= ) y2 (1 - y'2) dx


O

en el espacio de funciones JI (x) E el [O. n) que satisfacen la condicin


JI (O) = JI (n) = O. En el segmento (O. n) del eje Ox hay mnimo dbil
de J. En efecto, tenemos J = O si y = O; por otra parte, para las curvas
pertenecientes a una e-vecindad de primer orden .M este segmento,
donde e es cualquier nmero positivo menor que 1, se tiene I y' I < l
de modo que el integrando es positivo para JI ::;6: O y, por consiguiente.
la funcional se anula slo si y = O. Es decir, la unclonal alcanza
mnimo dbil en la curva y = O.
Mnimo fuerte no hay. Basta tomar
I
y (.1:) = Vii sen nI

En este caso
n

J Iy (x) =

~ J sen2nx
o

(1 -n

cos2 nx) dx=

=1.n J

sen2nxdx-..!....
sen22nxdx,.,,~-~
4 J
2n
8'
O
O
O sea, J < O en estas curvas si n es suficientemente
grande. Por otra
parte, siendo tl suficientemente grande, todas estas curvas se encuentran en una vecindad tan pequea como se quiera de orden nulo de la
curva JI = O. Por consiguiente, no se alcanza mnimo fuerte en y = O.
EJEMPLO 22. (Weierstrass).
Consideremos
la funcional
1

J[y{x)=

x'2y'2dx;

y(-I)=-I.

y!I)=1.

-1

En el segmento [-1, 1) tenemos J Iy (x)] ~ O y, adems, J [11 (x) = O


slo si y' (x) = 0, o sea, si y (x) = e = const. La funcin 11 (x) = e
pertenece a la clase el [-1,1] de las unclcnes que tienen primera derivada continua en ('1segmento (-1, lJ, pero 110 satisface las condiciones
de frontera dadas. Por consiguiente, J [y (x)]
O para todas las
(unciones y (x) E
[-l. 1] que satisfacen las condiciones y (-1) ::i::; -1

el

>

CAP. JI. EXTREMO

48

DI! PUNCIONALES

e 11 (1) = 1. En otras palabras, la funcional tiene cota inferior pero


sta no se alcanza en las curvas y (x) E el 1-1, 11. Efectivamente,
consideremos la familia monoparamtrica
de curvas

x
~

arcfg-

Ya. (x)=

J'
arctg-

~>

O.

n
Todas ellas satisfacen las condiciones de frontera: Ya. (-1) = -1 e
Ya. (1) = 1. Pasando al lmite para a -- O. obtenemos la luncln
-1, si -1 ~;c:<O,
y(x) =
O, si
x=O,
{
-+ 1 ,si
O <x
1.
es decir,
(x) = sg x (Hg. 3). Esta funcin pertenece a la clase de
funciones dferenclables a trozos en el segmento [-l.
1).

<

y ""Yo(x)

FIg. 3
Tenemos
t

J (Ya. (x)J

(=

-t (a2+x2)

arctg2-

2a

arctg2 -a o

2a 1 (I-a
arctg2~

arctg ~ ) .

3.

PUNCIONAL.

VARIACIN

DE

UNA

49

FUNCIONAl.

Queda claro que J {y~ (x)J -+ O cuando ce. -+ O. En la funcin lmite


y(x), que satisface las condiciones de frontera y(-l) = -1 ey (1) ::.:"
= 1, el valor de la funcional J [y (x)l es igual cero: J
(x = O.
Por consiguiente, la funcional J fy (x)) alcanza su valor mnimo
en la curva
(x) = sg x que pertenece. a la clase de funciones dierendables a trozos en el segmento [-1, 11 pero no pertenece a la clase
el [-1, 1].
Teo~EMA (condicin necesaria de ex tremo de la lunclonal}. Si la
funcional diferenciable J [y (x)l alcanza su valor extremo en la curva

= Yo (x), donde Yo (x) es un punto interior del campo de deft/~icin


de la funcional, entonces en y = Yo (x) se tiene
f>J [Yo (x)] = O.
(9)
Las funciones para las cuales 5J = O se denominarn funciones

estacionarias.

Hallar las ecuaciones funcionales para las funciones estacionarias de las funcionales que siguen, empleando la condicin necesaria de extremo (9) y los lemas fundamentales del
Clculo variacional.
b b

68. J [q> (s) =

J J K (8,
(l

t) q> (s) rp (t) ds dt

+ J rp2(s)ds-2

q(s)f(s)ds,

donde K (s, t) es una funcin continua simtrica de s y

b}

a -c s <.
el recinto D { a ~ t _~ b

.!

en

(s) es una funcin continua

en fa, b] y cp (5) es el argumento funcional continuo incgnito.


+""

69. J [q (x)] =

[p (x) rp'2 (x)

+ 2rp (x + 1) X

-00

x <p (x-l) - rp2 (x) - 2cp(x) f (x)] dx,


donde el argumento funcional rp (x); es continuo y tiene derivadas continuas a trozos en todo el intervalo - 00 < x <
< + 00, p (x) tiene derivada continua y f (x) es continua.
70.

4-01387

J [q (x)J =

[p (x)

q>'2 (x) +q(x)

qJ2(x)--2(jl (x) f(x)}dx,

CAP. 11. EXT~fMO

50

DEiPUNCIONALES

donde p (x) tiene derivada continua, q (x) y f (x) son continuas y el argumento funcional <:p (x) tiene dos derivadas
continuas.
4 Problema elemental del Clculo variacional.
Ecuac.in de EuJer
Supongamos que la funcin F (x, y. 11') tiene derivadas parciales
continuas hasta de segundo orden inclusive respecto a todos sus argumentes.
El problema elemental del Clculo variacional
es el siguiente:
entre todas las funciones y (x) que tienen derivada continua y que
satisfacen las condiciones de frontera
y (a)
hallar

la [uncin que ofrece extremo

y (b)

(1)

dbil a la unconal

J 111(x)1 =

j F (x,

Y. y/) dx,

(2)

En otras palabras.
el problema elemental
del Clculo var iaconal
consiste en hallar el extremo dbil de la funcional de tipo (2) en el
conjunto de todas las curvas suaves que unen dos puntos fiJOS PI (a, A)
y p.. (b. B).
TEOREMA l. Condicin necesaria 1) para que la funcional (2), defi

nida en el coniunto de todas las funciones 11= {I (x) que tienen derivada
continua IJ que satisfacen las condiciones de frontera (1), alcance su valor
extremo en la functn y (x) es que esta funcin verifique la ecuacin
de Euler
d

F,,--F
1=0.
Y
dx 11
Las curvas integrales de la ecuacin de Euler se denominan
de Lagrnnge).
En forma desarrollada
la ecuacin de Euler da

males (o curvas

(3)

exire-

V~ (x) FIj'II' -1- y' (x) Fyy'


Fxy'Fu = O
(FY'll' ,*0)
y representa una ecuacin diferencial
de segundo orden de modo que

su solucin "general comprender


dos constantes
arbitrarias
cuyos
valores se determinan.
hablando en trminos generales, de las con diciones de frontera (1)_
La funcional (2) puede alcanzar extremo slo en las cxtrcrnales que
satisfacen las condiciones (1).
1) Esta condicin es necesaria para el extremo dbil. Como quiera
que todo extremo fuerte PoS al mismo tiempo un extremo dbtl, cualquier condicin necesaria para el extremo dbil tambin ser necesaria
para el extremo fuerte.

PROBLEMA

4.

El problema

no siempre

ELEMENTAL

51

de contorno

tiene solucin y si la solucin existe, puede no ser nica.


I./.En qu curvas puede alcanzar Su extremo la funcional

EJEMPLO

J[y(x))=

(y'2-2xy)dx;

y(I)=O,

y{2) -'.-11

SOLUCIONo Aqu tenemos

y=

Utilizando
seguiente

F (x, y, y')

= y''}. -

+ x = O. Su solucin

de Euler da y'

la ecuacin

2xy de modo que


general es

xl

-6 +C1x+C:.

las condiciones de frontera, obt enernos para CI y C2 el


sistema de ecuaciones lineales:

Ct+C2= ~,}

="6'

2C1+Cz

C,=!

De aqu resulta
alcanzarse

y C2=O.

Por consiguente , el extremo

puede

slo en la curva
x
Y='6{I-.x:2).
Hallar

EJEMPLO 2.

las extremales

de la funcional

J [U (x))

=j

(3x - y) .1/ ctx

que satisfagan

las condiciones

SOLUCIN.

La extremal
nuestro

IJ

La ecuacin

= ~x

de frontera

de Euler es 3x -

no satisface

y (1)

1 e U (3)

2y=O, de donde y

la condicin

4- ~.

= ;

X.

y (1) = I y, por eso,

problema

variacional
no tiene solucin.
EJEMPLO 3. Hallar
las extrernales de la funcional
21"(

Jly(x)]=

~ (U/~_y2)dx
O

que satisfagan

las condiciones

de frontera

y (O) = 1 e !J (2n)

l.
4*

52

11. EXTREMO DE FUNCIONALES

CAP.

SOLUCION. La ecuacin de Euler tiene 13 forma 1/'


solucin general es
y = el cos x
e9 sen x,
Utilizando las condiciones de frontera, encon tramos

+ g = O; su

y
donde

e es una

ces

+ e sen x,

constante arbitraria.

Es decir, el problema variacional

considerado tiene un conjunto tnlinlto de soluciones.


Hallar las extrernales

de las funcionales

siguientes.

71. J(y(x)1=)

(12xy-y'2)dx;

y(-1)=I,

y(O)=O.

-1
2

72. Jly(x)1=)

(y'2

+ 2yy' +y2)

x; y(I)=

1, y(2)=0.

73. J [y (x)1 =

J y y (1 + y'Z) dx;

y (O) = y (1) =

;2.

74. J[y(x)l=

~ yy'2dx;

y(O)= 1, y(I)=:;4.

o
n

J (4ycoSX+y"2_y2)dx

75. Jly(x)l=

o
y (O) = 0,

y (n) = O.

76. J{y(x)=

J (y'2_y2_y)ezrdxj

y(O)=O,

y(I)=e-t

O
1

77. J[y(x)l=

J (y'2-2xy)dx;

y(-I)=

-1, y(I)=1.

-1

78. J Iy (x)] =

(y'2_ 2xy) dx; y { -1)

= O,

Y (O) = 2.

-1

79. J [y (x)} = J (xy'2+ yy') dx; y(l) = O, y (e) = l.


I

pnOHLEMA

ELEMENThL

La ecuacin de Euler (3) para la funcional

53
(2) es una ecuacin

dllerenclal de segundo orden y, por eso. la solucin y = y (x) de 13


ecuacin de Euler debe tener segunda derivada y" (x). Sin embargo.
se dan casos en que 1a lunctn que ofrece el extremo a la funcional
b

J {y (x)}

a
EJEMPLO.j.

F (x. y, y') dx no tiene segunda derivada.


La funcional
I

J Iy Ix)) =

y2 (1- y')2 dx

-1

con las condiciones

de frontera
y (-1) = O

e y (1) = 1
alcanza su valor mnimo. igual a cero, en la funcin
si x ~
v x - x si x>O.

( ) _ {O

O,

Aun cuando la funcin v (x) no tiene segunda derivada. sattsace la


ecuacin de Eulcr correspondiente.
Efectivamente,
tenemos F (x, y. y') = y? (1 _ y')2 y, poniendo
y = v (x), obtenemos la ecuacin de Euler
2V(I-v')2+!

(2v2(I-v')]=0.

(4)

Pero, segn la definicin de la funcin v (x), en (-1, 1) tenemos


F , = _2vll (1 - VI) == O y, por consiguiente, tambin dd F, = O;
X

11

"

o sea, a pesar de que la ecuacin de Euler (4) es formalmente de segundo


orden y a pesar de que u (x) no existe, la ecuacin de Euler se convierte
en identidad al sustituir en ella v (x).
TEOREMA 2. Sea y = y (x) soluci6n de la ecuacin ( Euler
d

F y - "'(fX F 11' = O.

Si la funcin F (x, y, y') tiene derivadas parciales continuas hasta de


segundo orden inclusive, entonces la funcin y = y (x) ttene segunda
derivada continua en todos los puntos (x, y) para los cuales
FY'JlI

[x, y (x), y' (x)]

*0.

COROLARIO. La extrema! y = y (x) puede tener puntos angulares


s6/0 en aquellos puntos en los que Fy,y, = O.
Asi, en el ejemplo 4 tenemos que F",y' = 2y2 se anula en los
puntos del eje Ox; la extremal tiene punto. angular en )t = O.
TEOREMA 3. (Bernstein). Supongamos que en la ecuacin

y.

=f

(x, y, y')

54

C,\P. 11. ex'rREMO

DE FUNCIONALES

las funciones f. 11/ y f ' son continuas en lodo punto finito (x, y) para
cualquier valor finito de !J' !J supongamos que existe una constante k > O
y unas funciones
~ = a (x, y) ~ O y ~ = ~ (x. y) ~ O.
acotadas en cualquier porcin [inita del plano, tales que
tv (x, y, g') > k
Y I f (x, y, y') 1 ~ a.y'2
(6)
Entonces, por dos cualesquiera puntos del plano (a, A) y (b, B) de abscisas
distintas (a =1= b) pasa una curva integral y = 'l' (x) de la ecuacin (6),
y slo una.
EJEMPl.O 5. Demostrar que por dos cualesquiera puntos del plano
de abscisas distintas pasa una extremal nica de la funcional

+ ~.

J{y(x\]"""

\ c-211%(y'2_1)<I,1:

SOLUCiN. La ecuacin de Euler para la funcional

yn = 2y (1

considerada

es

-1- y',

de modo que se puede aplicar el teorema 3. En efecto, tenemos en


este caso

(x, y, y')
Adems,

+ y'Z)

y, !J')

EJEMPLO6. Demostrar

fy = 2

I = I 2y (1
~ = 2 I y I > O.

I f (x,
o sea, a. =

2y 11

+ y''}.)

(1

+ y(2)

I~ 2Iy I

y'2

que no hay extrema!

J Iy (x)j =

J (y2+

~ 2

k.

+ 2 I y 1,

de la funcional

VI + y'2) dx

que pase por dos cualesquiera puntos del plano de abscisas distintas.
SOLUCION. La ecuacin de Euler tiene la forma
3

yn=2b'{1+.!I'2)2
(7)
y el teorema 3 no se puede aplicar ya que no se cumple la segunda de
las condiciones (6) (debido a que f (x, y, y') crece, respecto a y', ms
rpido que la segunda potencia de y'). Las condiciones del teorema 3
son de carcter suficiente. O sea, si estas condiciones no se cumplen. de
ello no se puede deducir que no hay extrema) que pase por dos puntos
cualesquiera de abscisas diferentes. Demostremos que por los puntos

A (O, O) Y B (~ ,

2)

no pasa ninguna extrema! de la funcional con-

siderada.
Tomando en la ecuacin (7)

y'=p

=p

dp

d!l '

4.

PROBLEMA

obtenemos
dp

P -=2y

dy

ELEtv\ENTAL

55

.~

(1 +p)Z 2. o sea,

pdp

~2fJdy.

(1 +p2.)2

Integrando, encontramos
I
-~-==!2-C,

lfi"+P"2
(C-y2)

de modo que

VI +y'2=

o sea,
I

donde e es una constante real. Separando las variables en la ltima


ecuacin e integrando del punto A al punto B, obtenemos
I
-=
2

ri

C-y2

VI _(C_yZ)2

(8)

dl/.

Cualquiera que sea el nmero real C. el denominador del integrando de


(8) ser complejo en cierto intervalo (a, ~) c: (0, 2) de variacin de
la variable y. Por consiguiente, la igualdad (8) es imposible. Esto quiere
decir que no se puede trazar extrernal alguna por los puntos A (O. O)

YB(~,

2).

80. Demostrar que por dos cualesquiera puntos del plano


pasa una y slo una extremal de la funcional
J [[1 (x)l =
EJEMPLO

7.

J 1/1+!f + y'2 dx,

Demostrar que toda ecuacin


y{t (x} = f (x, y, y')

(9)

es ecuacin de Euler para cierta funcional


r
J fu (x)}=
F (x, y, y') dx,

1) Cmo se determina la funcin F (x, Y. y') a partir de la funcin


(x, y. y')?

2) Hallar

todas las funcionales

cuyas extrernales

SOLUCION. Busquemos

son las rectas

y = C1x
C2
la funcional cuya ecuacin de Euler

fy-E"'x-f

y,,,y' -f"'I/'Y" =0

CAP. 11. EXTREMO DE FUNCIONALES

56

coincida con la ecuacin (9). Es decir, debe cumplirse la identidad

x, y e y'

Fy-F"'x-FY'IIY'
Derivando respecto a

{J',

-F,v'!

en

(x, y, y');; O.

obtenemos

F1I'II'x+ Fy'Y'YY' FlI'lI'lI,l+FII'lI,fll,=O.


Tomando u = FY'II"
derivadas parciales:

obtenemos para la funcin u una ecuacin en

au
+'
r)x y

(Ju

iJy

+f iJy'
ou

+f 1/'

u=

(10)

Por consi~uiente, la bsqueda de la funcional, o sea, de la funcin


F (x, y. y ), se reduce a la integracin de la ecuacin lineal en derivadas parciales (10) y a la cuadratura sucesiva.
Consideremos la segunda cuestin. En este caso la ecuacin de
Euler debe ser y. = O y para la funcin u se obtiene, de acuerdo con
(10), la ecuacin

j::_+
ax

,2!:..-o .

(11)

y ay -

Integremos esta ecuacin.


La ecuacin de las caraderisticas

tiene la forma

dx
dg
dy'
-1-=7=--0
Integrando

este sistema,
y'

encontramos

Cl

clx

+ C2

de donde Cs = y - xy'. Por eso, la solucin general de la ecuacin


(1t) es
u (x, y. y') = (l) (g'. y - xy').
donde ID es una funcin arbitraria
De aqu

diferenciable

de sus argumentos.

z
F(x, y, z)=a(x,

Y)+z~(x,

y)+

(z-/)I1>(t,

y-tx)dt,

o
donde a (x, y) y ~ (x, y) son funciones arbitrarias

que cumplen la relacin

oa

de sus argumentos

o~

ay = a;: .

Se puede ver de la sol ucin que existe una cantidad infinita de problemas variacionales que tienen la ecuacin (9) como ecuacin de Euler.

4.

PROBLEMA

ELEMENTAL

CASOS ELE.'>\ENT AI.ES DE I NTEGRACION

DE J.A ECUACiN

DE EULER.

= F (x, y).
En este caso la ecuacin de Euler tiene la forma
1, F no depende de y', o sea, F

Fy (x, y) = O.

(12)

La solucin de esta ecuacin finita (no dllerencial) no contiene elementos arbitrarios y, por eso, no satisface, hablando en trminos generales. las condiciones de frontera y (a) = A e y (b) = B,
Slo en casos excepcionales, cuando la curva (12) pase por los
puntos de frontera (a, A) y (h, B), existir una curva en la que podr
alcanzarse
el ex tremo.
EJEMPLO 8. Hallar las extrernales de la funcional

2"'
J (U (x)!

= S y (2x -

y (O) = 0,

U) rlx;

y ( ; )

= ~.

o
y

La ecuacin de Euler tiene la forma 2x-2y = 0, o sea.


Puesto que las condiciones de frontera se satisfacen, la integral

SOLUCION.
X.

,SI

'2

Y (2x -

y) dx puede

alcanzar su extremo en la recta y = x. Para

o
otras condiciones
la extrema! y

(~)=

1,

x no pasar por los puntos frontera (0, O) Y ( ~ ,

1)

de frontera,

por ejemplo. y (O)

Oe y

de modo que el problema variacional con estas condiciones de frontera


no tendr solucin,
20, F depende- de y' en forma lineal. o sea.
F (x, y, y') = M (x, y)
N (x, y)!J'.

En este caso la ecuacin

de Euler tiene la forma

oM

eN

Ty--ax=o.
Igual que en el caso 1, la ecuacin obtenida es fillib:l y no diferencial.
En general, la curva determinada por la ecuacin O..M - laN = O
uy
x
no satisface
las condiciones
de frontera y, por consiguiente, el
problema variacional no tiene, como regla, solucin en la clase de
funciones con tinuas.
Por otra parte, si en un recinto D del
oM - a;
iJN = O . I a cxpreston
"F
(x, y. y ') =
p 1ano x Oy se ulene Ty
"'" M (x. y) dl(

N (x, y) dy es una diferencial

total exacta

y la

CAP. 11. EXTREMO DE FUNCIONALES

58
funcional

(b. B)

J [y (x)] = ) F (x, y, g') dx=


a

(M dx+N dy)

(a, A)

no depende del camino seguido en la integracin: el valor de la Iuncional J ry (x)] es el mismo en todas las curvas admisibles. El problema
varlaconal carece de sen ti do.
EJEMPLO9. Anallzar el extremo de la funcional
b

J(y(x)=

) (y2+2xyy')dx;

y(a)=A,

y (b)=B.

Aqu F depende de U' en forma lineal. Tenemos

SOLUCiN.

aA1

Jy =2y,

aN
iJx. =2y

oM _ aN = O
ay
ax - ,

o sea, el integrando (yz + 2xyy') dx es una diferencial exacta. Por


consiguiente. la integral no depende del camino seguido en la ntegracin:
b

J [U (x) 1 =

(ti, B

(y? dx

+ 2xy dy) = Jr

d (xy2)

= Xy21X~b

= bB2 - aA2

x=-a

(a, Al

para cualquier curva de integracin y (x) que pase por los puntos (a, A)
y (b, B). El problema variacional carece de sentido.
30 F depende slo de y', o sea, F = F (y').
La ecuacin de Euler tiene la forma

Fy,yY =

Q.

En este caso las extrernales son todas las rectas posibles

donde Cl

y = C.x
C~,
son constantes arbilrarias.
10 Hallar las extremales de la funcional

y e,

EJEMPLO

J Iy (x)} =

J VI +y':l

ds,
y (a) = A, y (b) =8.
a
Esta funcional determina la longitud de la curva que une los plintos
(a, A) y (o, 8). Desde el punto de vista geomtrico. el problema consiste en hallar la curva de longitud mnima que une dos puntos fijos.
SOLUCIN. La ecuacin de Euler tiene la forma y" = O.Su solucin
general es

PROBLEMA

La extrernal que satisface


IJ (b) = Bes, obviamente,
y (b, B):
!I=

eLEM~NTAL

59

las condiciones de f rentera y (a) = A e


la recta que pasa por los puntos (a, A)

B-A
b

-a

(x-a)+A.

4. F no depende de y. o SClI, F"= F


En este caso la ecuacin de F.uler os
resulta

(x. y').

Fy' (x. y') :.... O. de donde


(I3)

siendo Cl una constante arbltrar!a.


La ecuacin (13) es una ecuacin dlerencial
de pri mer orden.
Integrndola, encontramos las extrernales del problema.
Entre las curvas que unen los puntos A (1, 3) Y
1.'11 la que puede alcanz ar su extremo la Iunciona l

EJEMPLO 11.

B (2,5) hallar la curva

Iy (x)l='~ )

y' (1

+ x~y')

dx,

que F no depende (.1\: y, la ecuacin de Euler


:'(, Fy. (x, y') .~.O, o sea. ,~x (1.\ 2xZy') = O de donde

SOLUCl<)N. Puesto

tiene Ia forma

1 +2x2y' =C,.
En t onces, y ,C
=1-1 2x2

l-e
l
2

de

do que

010

. Por consiguiente,

q
Y=T"

'C

2,

donce
d

las ex tremales representan

C1*-

una familia

de hiprbolas. Determinemos la extremal que pasa por los puntos


fijos, Para hallar los valores de las constantes Cf y C2 lormamos el
sistema

de donde resu Ita

Cr = -

4 Y

e2. = 7.

La ex trema 1 buscada es y =

=7-~.

5. F no depende explcitamente
de x, o sea, F
En este caso Ia ecuacin de. Euler tiene Ia forma

" -Fyy'Y' _FII'y,yn =0.

F (y, y').

60

CAP. 11. EXTREMO DE FUNCIONAl.ES

Multiplicando ambos miembros de esta ecuacin por y', obtendremos


en el primer miembro la derivada exacta :,. (P-y'FII,),
ecuacin ser

o sea,

la

:1' (F - y' Fy') = 0, de donde


(14)

el una constante arbi traria. Esta ecuacin puede ser integrada


resolvindola respecto a g' y separando las variables o introduciendo un
parmetro.
EJEMPLO 12 (cuerpo de resistencia mnima en un fluido). Determinar la forma del cuerpo slido que, al moverse en un fluido de gas,

siendo

del fluido

x
do gas

Fig.

encuentra resistencia mnima. Para simplificar consideraremos el


cuerpo de revoluci6n (Hg. 4).
SOLUCION. Suponiendo que la densidad del gas es suficientemente
pequea y que las molculas. al chocar con la su perficie del cuerpo, se
reflejan de forma especular, obtenemos para la componente normal de
la presin
p = 2ptill sen2 a,
donde p es la densidad del gas, t! es la velocidad del gas respecto al
cuerpo y f) es el ngulo entre la velocidad y su componente tangencial.
La presin es perpendicular a la superficie de modo que la componente
segn el eje O,. de la fuerza que acta sobre un anillo de anchura
(1 + ytll) IJ2 dx y de radio y (x) se puede representar en la forma
1

dF=2pt/}'sen2 e [2ny (1

+ y'2}2

sen O dx,

PROBLEMA

La fuerza resultante
es igual a

ELEMENTAL

que acta en la direccin


/

61
positiva

del eje Ox

4npo2 seoS 6.11 (1 + .11'2)2" dx.

Supongamos, para simplificar

el problema,

que

Entonces, la fuerza de resistencia ser igual a


I

F= 4n:pU2

y'3y dx.

(15)

El problema consiste en hallar la funcin y (x) en la que F alcanza

su valor menor posible siendo


.ti (O)

.ti (/)

(16)

R,.

La ecuacin de Euler para la funcional (15) tiene la forma

:z

.11'3-3

(17)

(.11.11'2)=0.

La solucin particular .ti = O de esta ecuacin debe ser rechazada en


virtud de las condiciones de frontera (16). La ecuacin (17) puede ser
representada as:
v"
3yy'y = O.
(18)

MuHiplicando p,or y' ambos miembros de (18), vemos que el primer


miembro es (y 3.11)'. 1ntegrando, encontramos
y'SU

= q,

de donde resulta

y'

C1
=-r=yy

y=(C.x+C2).

Utilizando las condiciones de frontera (16), obtenemos


4

R,3
Ct=-de modo que

Ca=O

62

CAP. 11. EXTREMO DE FUNCIONALES

o sea, el contorno Con ex iremos fijos que corresponde a la resistencia


mnima

del cuerpo es una parbola

EJEMPLO 13. Hallar

la extremal

lb' (x)J:.-

1..

de grado
4
de la funcional

b,_
V 1 +y'2.

!I

dx

el

que pasa por dos puntos fijos (o, A) y (b. B) pertenecientes al semiplano
su penar.
SOLUCION. Puesto que la funcin integrando no contiene explcitamente x, la ecuacin de Euler, segn (14), da

ViT?2
11

Despus de simplificar,

encontramos y

VI + y'~ = C\. donde

= Cl. Integrando la ltima ecuacin, encontramos (x

= Gr.

el =

+ Ci)t. + !I' =

o sea, una familia de circunferencias con centros en el eje Ox.


La extremal pedida ser la que pase por los puntos fijos. El problema
tiene so lucin nica ya que por dos puntos cualesquiera del semi pi ano
superior pasa una y slo una semicircunferencia con centro en el eje Ox.
OBSERvACiN. Segn el principio de Fermat, el camino que recorre
un rayo de luz al propagarse con la velocidad v (x. y) en un medio bidimensional no homogneo constituye una extremal de la funcional

X'V~

Iy (x)) = S ()(x,

y)

dx,

Xo

Como hemos visto en el ejemplo anterior, si la velocidad de la luz (1


es proporcional a y, los rayos de luz representan arcos de circunferencias
con centros en el e.je. Ox.
Sea q una curva. Denominaremos longitud ptica de la curva q
el tiempo T (q) que se precisa para recorrerla al moverse segn esta
curva con la velocidad de la luz ti (x, y).
Supongamos que el semi plano superior !I > O es un medio ptico
en el que. la velocidad de la luz en todo punto es igual a la ordenada del
mismo: ti = y. Como hemos visto, los rayos de luz en este medio sern
semicircunferencias con centros en el eje Ox, Se puede demostrar que,
si uno de los extremos del arco AD de la semicircunferencia q se halla
en el eje Ox, su longitud ptica es infinita (rig. 5). Por eso, diremos
que los puntos del eje Ox estn en el infinito. Consideremos que las
semlcrcnferencias con centros en el eje O son recias, que las longitudes pticas de los arcos de estas semicircunferencias son las longitudes
de dichas rectas y que los ngulos entre las tangentes a las semicircunerencias ell el punto de interseccin de las mismas son los ng~los

PROBLEMA

4.

63

ELEMENTAL

entre dichas rectas. Obtenemos una Geometra plana en la que se


conservan muchas proposiciones de la Geometra habitual. Por ejemplo, por dos puntos fijos se puede trazar una recta y slo una (ya que

Fig.5
por dos puntos del semplano se puede trazar slo una semicircunferencia con centro en el eje Ox)_ Dos rectas se consideran paralelas si
tienen un punto infinito comn (o sea, si las dos semicircunferencias
y

Fig. 6
son tangen tes en un punlo B perteneciente al eje Ox). En tortees, por
todo punto A que no se halle en la recia q se pueden trazar dos rectas ql
y q2 paralelas a q. Las rectas que pasan por el punto A v que se encuentran en los ngulos vertlcates 1 y IIl, cortan la recta q nientras que las
rectas que se encuentran en los ngulos II y IV no la cortan.
Hemos obtenido el modelo de Poincar de la Geometra plana de
Lobachevski (Iig. 6).

64

CAP. 11. eXTREMO

DE FUNCIONALES

Hallar las extrema les de las funcionales:


b

SI. J[y(x)J=

l2xy+

(x2

+ eV)y'Jdx;

y (a) = A.

y(b)=8.
t

82. J(y(x)1=

y(O)=O,

(eY+xy')dx;

y(l)=~.

83. J [y (x)j

= l'tJ (y'2_

y2) dx; Y (O)

= 1,

(T). ="""2'
112

l(y'2-y2)dx;
1\

84. J[y(xH=

y{n)=-l.

y(O)=l,

O
1

J (x+y'2)dx
(x)J= J (y2+y'2)dx;

85. J[y (x)l=

y(1)=2.

g(O)=I,

86. J[y

y(O)=O,

g(1)=

1.

O
i

87. J [y (x)l ==

S (y'2 +4y2) dx;

y (O) = e2. y (1)

= 1-

O
1

J
(x)] = J

88. J[y(x)]=

y(O)=l,

(2ell-y2)dx;

y(l)=e.

O
b

89. J [y

(xy'

+ y'2) dx,

(t

90. J [y(x)] =

J" (Y+ y; ) dx,


a

91. Demostrar que no tiene extremos la funcional lineal


b

J[y(x)=

J [p{x)y'+q(x)y+r(x)]dx,
el

donde p (x)

E el la.

bl, q (x)

E e [a,

b)

r (x) E e [a, bJ.

PROBLEMA

ELEMENTAL

65

92. Consideremos la funcional


b

J [y (x)1 =

J F(x,

Y, y')dx

con las condiciones de frontera y (a) = A e y (b) = B. Demostrar que la ecuacin de Euler subsiste al agregar al integrando
F (x, y, y/) dx la diferencial total de cualquier funcin u =

= u (x, y).

J fy (x)l =

93.

J (y2+ s" +2yeX)dx,


a
11/2

94. J (y(x)}

J (y2_

x) dx;

y'z-8ych

1)

y (O) = 2,

Y ( ~ ) = 2 eh ;.

95. Hallar las extremales de la funcional


b

J (y (x)l = ) xny,Z dx
el

y probar que para n ~ 1 no existen extrernales que pasen


por dos puntos situados a distintos lados del eje Oy.
PROBLEMAS

VARIACIONALBS

EN

FORMA

x= <p (1),

donde <p (t) Y

IV (/) son funciones continuas con derivadas continuas

a trozos siendo, adems,

(~~)
Consideremos la funcional
Jc=

donde

dx

Je FU,

+ ( ~~) 2 ;f= O.
ti

x, YI;'

y)dt=

~
dy

x =- e y= di'

~-01367

t ~ IJ

to~

Y=\Il (r),

En muchos
emplear la representa-

PARAMETRICA.

problemas es cmodo, y a veces imprescindible,


cin para mtrica de las lineas

F(t,

x, !J, .;,

y)dt,

(19)

CAP. ii. Ex'tk.eMO

bE I1tJNCIONALES

Para que los valores de la funcional (19) dependan slo de la


lnea, y no de su parametrlzacn que puede efectuarse de distintos
modos, es necesario y suficiente que la funcin integrando no contenga
explcitamente el parmetro t y sea positivamente homognea de grado
uno respecto a Jos argumentos X e

y:

x, y),

F (x, y, k~, ky) = kF (x, y,

k> O.

Por ejemplo, en la funcional


Je =

Je

x dy -!I dx

la funcin Integrando es positivamente


Efeclivamente,
tenemos

homognea de primer grado.

F(x, Y. ;, y)=xfi-Y~
y es obvio que
F(x, Y. k~.
Si la curva

kit) =kF{x,

y,

X. y).

ofrece el extremo a la funcional J e en la clase de lineas e que unen los


puntos fijos (xo. Yo) y (Xli Yl). las funciones tp (t) Y 'i> (t) satisfacen
las ecuaciones de Euler
F,- :

(Fil=O'}

(20)

FV-(j[(F.) =0.
11

Una de las ecuaciones (20) es consecuencia de la otra.


Las ecuaciones de Euler se pueden representar en la forma de
Weiersfrass
F,-F,
XII
1/X
-=
(21)
r
:1 '

FI (;2+y2)'2

donde r es el radio de curvatura de la extremal y FI es el valor comn


de las razones

P,.

F ..

F.,

f 4.

PROBLMA
EJEMPLO14. Hallar

LEMENtAL

las extrernales

de la funcional

(Xl. 11'1)

Je =

1/2y'Z ds,

(O, O)
SOLUCIONo Puesto que puede haber extremales que se cortan con las
rectas paralelas al eje Oyen ms de un punto, consideraremos el problema en forma paramtrica.
Poniendo x == % (t) e !I = Y (t), encontramos
que la funcin

ya

integrando tiene la forma y' ';""x, o sea, es positivamente

. .

homognea

x2

de primer grado respecto a JC e y.


La primera de las ecuaciones

(20) da

_dtd ( l/2-y%)
=0 ,

X2

de donde
yl (

Integrando

:!)

= q.

la ltima ecuacin, encontramos


y' = 2Ct%

Ca

Puesto que la extrema! debe pasar por el origen de coordenadas,


tenemos Ca = O. La segunda condicin

de frontera

da Cl

i;"

o sea, en definitiva,

EJEMPl.O 15.

Hallar las extremales de la uncional


ti

[e=

J [y X2+y2 +aa (Xy-yx)J

di.

to
SOLUCIN.

Poniendo
F(x, !J. ;.

y)= Y;D+V2+a'J.(xY-l/;),

vernos que la funcin F es positivamente homognea de primer grado


Empleemos las ecuaciones de Euler en la forma de

respecto a ~ e

y.

68

CAP. 11. eXTREMO

Weierstrass.

DE PUNCIONALE~

Tenemos

F
%%
F I =---;---= ------,,3l/x
y2
(~Z+g2)2
Por eso. la ecuacin (21) tiene en nuestro caso la forma
1
-=202
r
F

.::0:

-02

Es decir. la curvatura..!.. de la extremal es constante. Por lo tanto. las


r
extremares son arcos de circunferencias; en particular. se tienen circunferencias completas s
x(to)=x(t.), }
y (lo) =y (ti).

Hallar las extremales de las funcionales:

yZ_yzx'l.

te. O)

y2_3e1J1Xx2

(XI. 1/1)

96. Je =

(1, 2)

97. J e

dt.

'.

(o. O)

di.

(1. O)

98. J c=

(K

V~2+y2 -~y)

di,

(-1. O)

donde K> O es una constante.


5 Generalizaciones del problema elemental
del Clculo varlaclonal
1. funcionales que dependen de derlvedes de rdenes superiores.
Supongamos que. se tiene la runclonal
"'1

J [y (x)l=

S F {x,
q

y (x), g' (x) .. " yln) (x)) dx,

(1)

donde F es una funcin diferenciable n


2 veces respecto a todos los
argumentos e y (x) E en [%0. XtJ. y supongamos que las condiciones de
rontera tienen la forma
!I~(Xo) Yo. !I (%0)=!l' ... , y( 1\-1) (xo) = n- 1), }
(2)
y (xl) =!/it !I (Xl) = 111" " yen-t) (xl) = y\,.-o,

Yb

GENERALIZ

"ClONES

OEL

PROBLEMA

ELEMENTAl.

69

Las extremales de la funcional (1) con las condiciones (2) son las
curvas integrales de la ecuacin de Euler - Poissaa
d
d2
dn
Fv--; Fv'+ dx2 Fw- +(_I)n dxn F(n)=0.
la extrema! de la funcional

EJEMPLO 1. Hallar

fy (x-

(360x2y-y"Z)

dx;

!I (O)

O.

y' (O)

SOLUCIN. La ecuacin

Y (1)

1,

O,

y' (1)

2,5.

de Euler - Poisson tiene la forma

tP

360X2+ dxz (-2y")=O,

o sea, ;v = 180x2,

y su solucin general es

/1='2 x6-l' CjX3+CZXZ+C3X+


Empleando las condIciones de lrontera,

Ca="2'
La extrernal

C2=--3.

CJ=1

C,.

encontramos
y

C,.--O.

pedida es
l

Y=2 x6+2 x3-3x2 +x.


Consideremos el caso cuando en la frontera no se dan todas las
condiciones (2) sino un nmero de las mismas de modo que. despus
de emplear las condiciones de frontera, en la solucin general de la
ecuacin de Euler - Poisson contienen todava constantes arbitrarias.
Para resolver este problema es preciso hallar la variacin de la funcional (1). transformarla tomando en consideracin las condiciones de
frontera dadas y obtener condiciones complementarias en la frontera
igualando la variacin a cero.
EJEMPLO 2. Hallar la curva y = y (x) que ofrece valor extremal a
la funcional
b

J (U (x)J

= -}

y''I. dx

(3)

con las condiciones


y (a) = O
SOLUCIN.

y (b) ,= O.

La ecuacin de Euler - Poisson tiene la forma.


plV

= o.

(4)

CAP. 11. EXTREMO DE FUNCIONALES

70

Su solucin general
ti

CI

+ CZX + Cax2 + C4x3

(5)

contiene cuatro constan tes arbitrarias C (i = 1. 2. 3 y 4) Y las condiciones de frontera (4) no bastan para determinarlas. Por eso, como
hemos explicado, calculamos la variacin de la funcional (3). Tenemos
b

6J =

yW{jyW ax.

(6)

Integrando

(6) por partes dos veces, obtenemos


b

BJ

= g"

(x) {jy' (x)

1:- J U"'6U'

dx

~
= y" (x)

(Xli: -

6y'

y" (x) By

(Xli: + ) yIV6y dx,

(7)

La expresin (7) debe anularse en la extrernal ti (x) de la funcional


de la funcin 6y, resulta que glV = O;
sta es la ecuacin de Euler - Poisson para la funcionar (3). Pero si
la integral del timo miembro de (7) se anula, la expresin. de frontera
(3). Debido a la arbitrariedad

lb'" (x) {jy' (xl -

!I'" (x}

6y

(X)ll:

tambin debe ser igual a cero idnticamente. Puesto


y () = O (extremos fijos), resulta que debe ser

que 6y (a) =

y" (b) 6y' (b) - y. (a) 6y' (a) = O.


En virtud de la arbitrariedad
de las magnitudes ay' () y fJy' (a),
obtenemos necesariamente
y" (a)

y" (b)

= O.

(8)

Las condiciones (8) conjuntamente con las condiciones (4) determinan


unvocamente la extremal en la familia (5): g :: O.
2. Funcionales que dependen de m funciones. En el caso de una
funcional que depende de In funciones Y1 (x), 1:Ia(x), . !1m (x)
Xl'"

J (Ui> Y2 ... , Yml =

F (x,

y"

JI'/-, , !I'th

El.

y;, ....

U~)

'1 con las condiciones de frontera de tipo


Yh (xo) = y~,

IJk (xt) =

vi

(k = 1, 2! ... , m)

ax

s.

GENERALIZACIONES

DEL PROBLEMA ELEMENTAL

71

las extremales se determinan del siguiente sistema de ecuaciones diferenciales de segundo orden
F

l/k

d
dx

- -F

(k= 1, 2, ... ,m)

,=0

Vil

1.91

que se denomina sistema de ecuaciones de Euler.


EJEMPLO 3. Hallar las extremales de la Iuncional
2

J[y(x),

j (y'2.+Z2+Z'2)dx

z(x)=

t
con las condiciones de frontera
y (1) = 1, y (2) = 2,

(1)

= O,

z (2) = 1.

SOLUCION'. En este caso el sistema de ecuaciones (9) tiene la forma

y'=O, }
z-z"=O.
Resolviendo

este sistema.

encontramos

y = Clx
C2
y
Z = Cse"
C~e-:c.
En virtud de las condiciones de frontera. tenemos
1
e2.
Ct=l,
C2,=O, e3=e2=I
y
e"=-e2=I
de modo que la extrema) pedida

y=x,

}
sh (x-I)

z=

sh 1

es una curva alabeada que constituye la interseccin de dos superficies


ci 1ndricas.
EJEMPLO ". Hallar las extrernales de la Iunconal

1
n

fy (x), z (x)J =

(2yz - 2y2

+ y'2 _Z'2)

dx

si

1,
z (O) = O
y
z (n) = -1.
de ecuaciones (9) tiene la Iorrna
y' +2y-z=O,
}
z'+g=O,
.
de donde, eliminando la funcin z, obtenemos
ylV
2y
y = O.
La solucin general de esta ecuacin tiene la forma
fI = el tOS1:
el sen x
X (Ca tOS"
C" sen z},
y (O)

O,

!I (n)

SOLUC10N. El sistema

u.

CAP.

72

E XTREMO DE FUNCIONA LES

En virtud de las condiciones de frontera 11 (O) = Oe 11(n)


1
Cl = O Y C3 = - - de modo que

1, tenemos

1t

y=C2 sen x+C,x sen x_!' cosx.


n

z se determina de la condicin 2 = 1/" + 2y. Tenemos


I
z= Cl! ~nx+C~(2cos x+x sen x)+- (2 sen x= cos x).
n

La funcin

Las constantes Coa y C4 se determinan de las condiciones de frontera


(O) = O y z (n) = -1, de donde, resulta que C4 = O Y que Cl! es
arbitrarlo.
Entonces.
I
z=C2 sen x+- (2 sen x-x cos x).

1t

La familia

de extremales
y = C2sen

es

x- _x_cos x,

+
11:

Z = C2 sen x+
(2 sen x-x cos x),
donde Cz es una constante arbitrarla.
SO. Funcionales que dependen de funciones de varias
independientes. Consideremos la funcional

J [z(x, 1/))=

j ~F (X

1/,

2, :~,

~~)

dxay,

variables

(10)

donde F es una funcin dierenciable tres veces respecto a sus argumentos, y supongamos que se pide hallar Ia funcin z = 2 (x, 1/) que sea
continua conjuntamente
con sus derivadas hasta de segundo orden
inclusive en el recinto D, que tome valores fijos en la frontera r del
recinto D y que realice el extremo de la Iuncicnal (10).
Si el extremo de la funcional (10) se alcanza en la superficie
z = z (x, y), la funcin z = z (x, y) satisface la ecuacin de Euler Oslrogradski
o
iJ
F:z.~ aX. {Fp}ay {Fq} =0,
(11)
fj

donde OX {Fp}

ay {Fq}

son

las derivadas

parciales

respecto a x e y, respectivamente:

a
-ax{Fp}

()

ag-{Fq}

az

= Fpx+ FpzTx
=

Fqy+Fq'Z.

aqu se ha lomado, para abreviar,

OZ

ay

+ Fpp

op

oq

ax +Fpq OX '

op

+Fqp Ty+Fqq

az

Pi" =p

iJz

aq

ay ~

aV =q,.

completas

GENE~ALIZACIONES

DEL

P~OBLEMA

73

ELEMENTAL

La ecuacin (11) representa la condicin necesaria de extremo de


la funcional (10). Es una ecuacin en derivadas parciales de segundo
orden; se busca su soluci6n z = z (x. y) que toma valores fijos en la
frontera r.
EJEMPLO 6. Escribir: la ecuacin de EuJer - Ostrogradsk
para

la Iunconal

11%(x, y)] =

J J [( ~: )2_( ~~)2]dXdY.
D

SOLUCION. Tenemos F(x, Y. z, p, q)_p?_qZ

tramos -

a (2p)- oya
ax

Para la funcional
J[2'(X.,

"'a, .... xn)

(-2q) =0, o sea,

02z

fJ2z

OX2 -

iJy?

JI J

y, segn (II), encon-

= O.

F(Xh

Xa

, "'n.

Z,

Pi,

donde

iJz

(k= 1, 2, ... , 11). la condicin necesaria de extremo

Pk=;---

VXIl

viene dada por la siguiente

ecuacinde Euler- Ostrogradski


n

iJ~t {Fpi}=O,

Fz- ~
...
1
0,

en forma desarrollada,
n

FT.- ~

iJ

10

(FXPi+F:ZPiPt

+~
1....
1

,... 1

FpPj

a:: )

=0.

(12)

La funcin z = Z (Xl. X, , xn). solucin de esta ecuacin, debe


satisfacer en la frontera r del recinto n-dimensional D las condiciones
de frontera dadas.
EJEMPLO 6. Hallar las condiciones que debe cumplir la funcin
Z (Xl. X" , xn) para Que la integral de Dirlchlet
D [l (Xt, Xa, .. , xn)!

=)j

...J

~ ( ::, )

i=1

2 dXl dx2 ...

dxn

alcance en ella su mnimo si dicha funcin toma valores c:le~erminados


del recinto Q.

en la Irontera

CAP. 11. EXTREMO

74

DE FUNCIONALES
n

SOLUCiN. En este caso F= ~

::i.)

o sea, F no depende

2,

{...j

explcltamente de

, Xn

Xt. X2.

z. Por lo tanto,

Fz=Fr-Pi =FxiP=O,
F ..

={

2 si 1 =
O si i =1=

Ptp)

1,

y, aplicando la f6rmula (12), obtenemos


n

,... a2z
.4J ox! =0

;=-1

!.1z=O

(ecuacin n-dimensional
de Laplace),
OBSERVACION. Si bajo el signo de la integral figuran las derivadas
de la funcin z (x, y) hasta de orden n, la ecuacin de Euler - Ostrogradski tiene la forma
a
a
)'l..
a'J.
Fz- ox {Fzx}- iJy {FzlI}+ ax'J. {Fz:lCx}+2 oxoy {Fr-XII}+

0
+ ay2
2

{FZIIII}-'"

--

an

011' {FzlIlI ... II}=O.

+(-1)1'\

(13)

EJEMPLO7. Escribir

funcional
J [z (x, y)] =

la ecuacin de Euler -

Ostrogradski

para la

J ) [( g;~) + ( :~ r+
2

+2 (
SOLUCIN.

F=

Tenemos

iJ2z ) 2 + ((j2Z)
( oxll
ay2

02z
ox ay

((jZZ)
Jx ay

+2

)2 -

2zf (x, y)

] dx dy.

-22' (x, y).

Aplicando la frmula (13). encontramos

-2t (x,

y)

a2
+ ox2

a2z) + ay2
iJz

2 ox2

i)2Z)

2 ay2

+
02

+2 ox oy
1)

sea,

( 2 oxfJ2Z)ay

=0,

p.

GENERALIZACIONES

La ltima

ecuacin se representa brevemente


11M. = f (x, y).

DEL PROBLEMA ELEMENTAL

15

as:

Hallar las extremales de las funcionales siguientes:


J

99. J [U (x})

5 (u +2y'2 +JI2) dx;


2

U (O) = 0,

!I (1) =

o,

y' (O) = 1

e y' (1) = - sh 1.

J (240Y-UI'I2)dx;

100. J[y(x)l=

-1

y(-l)= 1,
y(O}=O,
y(O)=O,
y"(-1)=16,
b

10t. J [U (x)] - } (U

u' (-

1) = - 4,5,
y" (O) = O.

+y") dx;

y (a) = Yo, y(b) =Yit

Y (a) = v:

y' (b)=y;.

102. J [U (x)} =

J (y2 +

Uy") dx;

cJ

U(a) =A,.

U' (a) =Az.

y(b)

= B..

y' (b) = Ba.

103. J {g (x)) =

J (y'2+ y"2) dx;

o
g(O)=O, U(l)=shl,
y'(O)=l,
104. Hallar la extrernal de la fundonal

y'(l)=chl.

J (U (x)} = -} } y"2 dx
o
con las condiciones
y (O) = O,

y' (O) = O.

y' (1) = 1.

'11/"

105. J[y(x),

z(x)) =

J (2z-4y2+y'2_Z'2)dx;
o

V (O)= O,

!I ( ~ )

= 1~ z (O)= 0, z ( ~ ) = l.

76

CAP

11. EXTREMO

DE FUNCIONALES

106. J[y{x),

z(x)]=

Z;3 )

(2XY_Y'Z+

dx;

-1

y(I)=O,
107. J[y(x),

y(-I)=2,

z(I)=I,

z(X)J=

z(-I)=

-1.

2(y'2+Z'2_2yz)dx;

y (O) = 0,

= 1, z (O) = 0,

9( ~)

z ( ; ) = 1.

t08. J[y(x),

J (y'2+Z'2+2y)dx

z(x)]=

o
g(O)=l,

y(I)=2'

z(I)=1.

2(0)=0,

109. Probar que la ecuacin de EuJer de la funcional


b

J[y(x),

z(x)] =

F(x, y, z, g', z')dx

tiene las siguientes primeras integrales:


-JF

1) ay' = e si F no comprende !Jj


iJF
2) F - y ,oFoy' - zJ oz
= c' SI F no compren de x.

Escribir la ecuacin de Euler=-Ostrogradskl


funcionales:
110. J [z(x,

y)] =

para las

J ~ [( ~:)4-1-

r+
S J ( ~:~ + ~:~r
D

+( ~;

It 1. J [t (x, g)] =

12zf(x,

y)JdxdY,

dx d!J.

t12, J [z (XI> X2' "',

=)J
.-C(X"

X2 ...

xn)J =
n

". J r~
D

aJ

j....
1

xn)z2+2zf(XI'

(X"

XZ

, Xn)

:;J V-

X2, .. '. xn)jdxtdxa,

.. dxn

ti.

ce

NVARiNCA

LA

ECUACION

DE

11

EULER

113. Deducir la ecuacin diferencial de las superficies


de rea mnima.
114. Hallar la extrema! de la funcional
1 1

J [z (x, y)] =

Io 1o

etll

con las condiciones z (x, O) = O y


6. Invariancia

sen 211 dx dy
2

(X, 1) = 1.

de la ecuacin de Euler

Si la funcional
b

J [y (x)1

F (x, y, g') as

se transforma

efectuando una sustitucin de la variable independiente


o una sustitucin simultnea de la funcin incgnita y de la variable
independiente, las extremales continan determinndose de la ecuacin de Euler que se obtiene a partir del Integrando transformado. En
esto consiste la lnvariancia de la ecuacin de Euler.
Sea X = x (u, v) e y = y (u, e) con la particularidad de que

Entonces

Xu

Xl)

l/u

Yo

=F

o.

J F (x, y, y') dx= J F [ x (u, e), y (u, [/),

!lu+Yllv~
+x"vu'

Xu

X (xu +XDt1~) du =

(JI

(u,

(J,

v~) du

y las extrernales de la funcional inicial se determinan de la ecuacin


de Euler para la funcional
<D (u, (J, v~) du:

d
>1)-<Il ,=0.
du
11
EJEMPLO 1.

Hallar las extremales de la lunctonal


"'1

[r(<p)]=

"'o

y,2.+r'2d<p.

18

cAP. 11. XT~MO bE pUNcioNAlES


SOLUCION. La ecuacin

de Euler para esta funcional

La sustitucin

o.

y,2+r'Z

V,2+r'2 -~

es

e !I=r sen q> da

de variables x=rcosep

y lleva a la funcional
b

fJl (x)]

J -v +
1

11'2 dx

cuya ecuacin de Euler es !lit = O de modo que

+ Cs

11 = C1x

Por consiguiente,
por la ecuacin

las extremales

r sen

<p

de la funcional

CI,'cos

c:P

inicial vlenen dadas

+ C.,

donde CI y C, son constantes arbitrarias.


EJEMPLO 2. Hallar las extremales de la funcional
ID 2

J [y (x)} =

Jo

(e-~1I'2-e"yZ) dx,

SOLUCION. La ecuacin de Euler'para

la forma
yW _ y'
Realicemos

la sustitucin

de variables

inicial se transforma
2

J do (u)J=

tiene

+ et'XlI = O.

x= lnu,
!I =o.
La funcional

la funcional considerada

}
entonces en
2

(e-In Uu2{/2_e1n"v2) '::

(v'2_ v2)du

+ ti = O se ntegra fcilmente:
= el cos u + C, sen u.

Y su ecuacin de Euler v
ti

Volviendo a las coordenadas iniciales


de las extremales en la forma
JI = el cos eO!

JC

e 11. obtenemos

+ e~sen eX.

la ecuacin

s,

NViANCIA

DE

DE. eUtER

LA ECUACIN

115. Ha llar las extremales de la funcional


q:t(

J r sen

J (r(<p)) =

qi

Vr2+r,2,d<p.

11'0

116. Probar

que las extremales de la funcional


epi

J [r (q1

= 1f (r sen ep) V ,.a +,'Zdq>


'Po

se determinan por cuadraturas.


117. Hallar las extremales de la funcional
b

J Vx2+y2Vl+y':ldx.

J(y(x)l=

Igual que en el caso de una variable, la ecuacin de Euler Ostrogradski es invariante respecto a las transformaciones de coordenadas.
EJBMPLO 3. Escribir la ecuacin de Laplace
iJ2z
02Z .
. ox:&
ay2, =0
(1)

en coordenadas
SOLUCIONo

polares.
Consideremos

la funcional

y)]

D(z(x,

1~

(z~+z~)dxdy.

La ecuacin de Euler - Ostrogradski para esta funcional es precisamente la ecuacin (1). Pasemos en la funcional de las coordenadas
cartesianas (x, y) a las coordenadas polares (p, Ip): .t = P cos Ip, !I =
p sen IJ). Tenemos
op
op
alp
sen Ip
o<p cos IJ)
ax=COSCJI.
Olj=senlp,
Tx=--p-'
ay=-p-

Y. por eso,
D (z (p, CJI)}=

J J [(

zp :~

+ zq

~~

+(zp ~~ +zcp

::

)2]PdPd<P=J

J (pz~++Z~)dPd<p.
G

Formando la ecuacin de Euler - Ostrogradsk l para esta ltima


Integral, obtendremos la ecuacin de Laplace en coordenadas polares:
1
Zcpq>
pZpp
zp =O.

p. +

CAP, n. ~XTREM

DE I'IvNClNALRS

7. Campo de extrema les


La familia de curvas y = y (x, e) forma un campo propio en el
recinto D del plano xOy si por cada punto (x, y) de este recinto pasa
una y slo una curva de la familia y = y (x, e).
El coeficiente angular p (x, y) de la tangente a la curva de la
familia y = 11 (x, e) que pasa por el punto (x, y) se denomina inclinacin
del campo en el punto (x, y).
La familia de curvas 11 = 11 (x, e) forma un campo central en el
recinto D del plano xOy si estas curvas cubren sin cruzarse lodo el
y

Fig.

7J

recinto D y arrancan de un mismo punto (xu, Yo) que no pertenece al


recinto D. El punto (xo, Yo) se llama centro del haz de curvas.
EJEMPLO l. Dentro del circulo x~
y2 ~ 1 la familia de curvas
11 = ce=, donde e es una constante arbitraria y, en particular, e = O,
forma un campo propio ya que estas curvas no se cortan en ningn
punto y por todo punto (x, y) del Crculo pasa una y slo una curva
de esta familia (lig. 7). La Inclinacin del campo en un punto cualquiera (ot, y) es igual a

p (x, y)

= Ce= = y.

EJEMPLO 2. La familia de parbolas y = (x


C)' no forma campo
propio dentro del crculo X2
y2 ~
I porque distintas curvas de la
familia se cortan dentro del circulo y no cubren todo el recinto (Hg. 8).
EJEMPLO 3. La familia de curvas y = Cx forma un campo central
en el recinto x > o.

7.

CAMPO DB EXTREMALES

81

Fig. 8

forman campo (propio o central) en los recintos indicados las siguientes familias de curvas?
t18. y=Ctgx;
119. y=C

cos x:

a) Ixl<~;

b)

Ji

2<x~n;
x2

e)

y2

--+9::;;;;1.

120. y=(X-C)3j

121. y=C(x2-2x);
a) O::;;;;x<lj
122. Y =
a)

7t

b) -1::;;;;x~3;

e sen (x - : ) ;

4~x~2;

1t

123. Y = ex+c;

b)
X2

3~x~n;

+ y2~

e)

8~x~

1t.

1.

Si el campo (propio o central) est Iorrnadc por una familia de


ex tremales de cierto problema var lacional , se denomina campo de

extrema les.
6-01387

82

CAP. 11. EXTREMO


EJEMPLO f.

Consideremos

DE FUNCIONALES

la funcional
i

fu (x)) =

J
o

11'2 dx,

Sus extremales son las rectas y = C1x


Cl' La familia de extremales
y = Ca forma un campo propio y la Iamtlla de extremales y = C1x
forma un campo central con centro en el origen de coordenadas.

124. Determinar para la funcional


a

J [y (x) 1 =

Jo

(y'2

+ ya) dx

los campos de extrernales propio y central.


125. Lo mismo para la funcional
n/"

J Jy(x)J =

dx.

(y'2-y2+x2+4)

Supongamos que la curva y

=y

(x) es la extremal

de la funcional

XI

J fy (x)

J (y (x)]

F (x, y. y') ox
xo
que pasa por los puntos A (xo. Yo) Y 8 (x~, !ll)'
Se dice que la extrernal y = y (x) est incluida en un campo propio
( extrema/es si existe una familia de extremales y = y (x. C) que forma
un campo y que comprende la extremal !I = y (x) para cierto valor
C = Ca Ysi, adems, esta extremal y = IJ (x) no pertenece a la frontera
del recinto D en el que la familia y = y (x, C) forma campo.
Si existe un haz de extremales, con centro en el punto (xo. Yo),
que forma un campo en una vecindad de la extremal y = y (x) que
pasa por dicho punto. se dice que se ha encontrado un campo central
que incluye la extremal considerada y = y (x), Como parmetro de la
familia y = y (x, C) se toma el coeficiente angular de la tangente a
las curvas del haz en el punto (xo. Yo).
EJEMPLO 5. Consideremos el problema variacional elemental para
la funcional
2

(y'3+sen2 x) dx.

a) Sea Ij (O) :..:: 1 e y (2) = 1. La familia de ex trernaies de nuestra


funcional viene dada por la ecuacin Y = C,x
C2 La extrernal que
satisface las condiciones de frontera es y = 1. Dicha extremal se
puede incluir en el campo propio de extremales y = Cs. donde es
es una constan te arb j traria.

i.

tAMPO bE EXTREMALES

b) Sea y () = O e y (2) = 4. La extremal q responde a estas


condiciones de hontera es la recta IJ = 2x que puede ser incluida en
el campo central de extremales UI =: Cx (Cl es una constante arbitraria)
con centro en el punto O (O, O).
y

J(

Fig,9
EJEMPLO 6. Consideremos

el problema variacional

elemental

J (1/ (x))

!J'

(2X- -}

!J' ) dx;

-1

y(-l)=O,

I
Y (1)="2'

La solucin de la ecuacin de Euler t ene la Iorma y=x7.+Ctx+

+Cz' La extremal de este problema


en el campo propio de extremalcs

+ ~-! se puede
U=x2+ ~+Cz (rig. 9).
y=x2

incluir

84

CAP. II. EXTREMO

DE FUNCINALES

Probar que las extrernales de los siguientes problemas


variacionales elementales se pueden incluir en un campo de
extrernales (propio o central).
I

J (y'2-2xy)dx

126. J(y(x)]=

y(O)=y(l)=O.

o
1

127. J[y{x)l=

1o
J

y(O)=l,

(2eXy+y'2)dXj

y(l)=e.

128. J[y(x)]

(y2-y'2)dx

(a:;z!:kn);

y(O)=O,

y(a)=O

2.

129. JIy(x)]=

J (y'2+x2)dx

y (O) = 1,

y(2)=3.

O
DEFINICI()N. Sea I> (x, 11, C)=0 una familia de curvas planas. Se
llama Cvdtscrtmtnante de esta familia el lugar geomtrico de Jos puntos
determinado por el sistema de ecuaciones

<D (x, y, C) =0,


o<D (x, y, C) _ O

ac

-.

}
(1)

En el caso general, el C-discrimlnante comprende la envolvente de la


familia, el lugar geomtrico de los puntos ml tiples y el lugar geom
trico de los puntos de retroceso.
La envolvente de la familia cIl (x, y, C) = O es la curva que en
cada uno de sus puntos es tangente a cierta curva de la familia considerada y tal que cada una de sus partes es tangente a un conjunto nnito de curvas de la Iarnil ia.
Si se tiene un haz de curvas con centro en el punto A (xo, Yo), el
centro del haz pertenece al e-discriminante.
EJEMPLO 7 Hallar el C-djscriminante
de la familia de curvas
y = (x - C)2.
SOLUCIN. Las ecuaciones (1) tienen en este caso 1a forma
y- (X-C)2=O, }
2(x-C)=O,
de donde y = O Es fcil ver que la lnea y = O es la envolvente de
esta familia. Efectivamente, en cada uno de sus puntos x = Xo la
lnea y = O tiene tangente comn con la curva correspondiente y

p.

CAMPO DE EXTREMALES

85

= (x - XO)2 de la familia. Adems. si tomamos una parte de la lnea


y
O, por pequea que sea, habr un conjunto infinito de curvas de
la Iamilta tangentes a esta parte. En el caso considerado el e-discriminante consta de la envolvente nada ms.

En los problemas siguientes hallar los C-discriminantes


de las familias dadas.
lS0. y = Cx + O.
131. y (C - x) - C2 = O.
132. (x - C)2 + y2 = 1.
Si el arco AB de la curva y = y (x) tiene un punto comn A",
distinto del punto A, con el ediscriminante
del haz y = y (x, e) que
tiene su centro en el punto A y que. comprende la curva considerada, se
dice que el punto A * es conjugado del punto A.
EJEMPLO 8. Consideremos
la familia monoparamtrtca de curvas
y = e sen x. El e-discriminante
de esta familia se determina por las
ecuaciones
y-Csenx=O,
}
-!en

x=O,

o sea, representa un conjunto discreto de puntos (1m. O), k = O, l.


2, ... (que son los puntos de interseccin de la sinusoide y de] eje
y

2Jr

rlg.

J(

10

Ox). Tomando, por ejemplo, e = 2, obtenemos la curva y = 2 sen x


que pertenece al haz de sinusoides con centro en el punto O (0, O).
Si el otro extremo B (Hg. 10) del arco de la curva y = 2 sen x tiene
la abscisa
E (n, 2n), el arco DB tendr otro punto (a parte del punto
O (O, O perteneciente al e-discriminante, a saber el punto 0* (n, O),
que ser conjugado del punto O (O, O). Si es O
ni en el arco 08
no habr puntos conjugados del punto O (O, O).
I

<x <

86

n,

CAP.

EXTREMO

DE FUNCIONALES

133. Se tiene la familia de curvas y = e (x - 1) x.


Hallar el punto conjugado del punto O (O,O).
134. Se tiene la familia de curvas y = e sh x, Hallar
el punto conjugado del punto O (O, O).
1. Condl<:in suficiente de Jacobl para poder IncluJr la extremal
en un campo central de extremales, Condicin suficiente para que el arco
A B de una extremal pueda ser incluido en un camfo central de extremales
con centro en el punto A (xo, Yo), es que Id punto A conjugado del punto A
no pertenezca al arco A B.
EJEMPLO 9. Consideremos la funcional
a

J (g (x))

=)

(g'2_9y2+eXl_l)

dx;

ti (O) = 0,
ti (a) = O.
Analizar la posibilidad de incluir la extremal y = O en un campo
central de extremales con centro en el punto O (0, O).
SOLUCiN. La ecuacin de Euler para la funcional considerada tiene
la forma y. + 9y = O y su solucin general esg = Cl sen 3x
Ca cos 3x.

Si a =1=

k; , donde k es un nmero

entero, la extremal

que satis-

face las condiciones de frontera es la recta g = O. Consideremos la


amla monoparamtrica de extremales y = Cl sen 3x; es fcil ver que el
C-dlscrlminante de esta familia consta de los puntos
donde

(k; O),

< i-,

en la extremal !I = O no
habr punto conjugado del punto O (O, O) y entonces esta extremal se
es un nmero entero; por eso, si a
podr, obviamente.
centro en

incluir en un campo central

eJ punto O (O, O). En cambio, si

de extremales

con

.n;

11 ~

'3' en la extremal

y = O habr como mnimo un punto conjugado del punto O (O. O) Y


no se cumplir la condicin suficiente de J acobi; en este caso las extremates y = Cl sen 3x no forman campo.
FORMA ANALrTICA DE LA CONDICiN DE JACOBI. Consideremos el
problema

variaclonal

elemental

:le I

J(y(x)]=

F(x, y, y')dx;

y (xo)=Vo,

Y (X)=-=Yt-

:ce

Si la solucin u=u (x) de la ecuacin de Jacob

(FIIII-*

u- :x

FIITI')

'II'u/)=O

(FJI

(2)

que satisface la condicin u (xo) = O se anula tambin en algn otro


punto del intervalo Xo < x < Xl' el punto A conjugado del punto
A (xo. Yo) pertenece al arco AB de la extremal (el punto B tiene las
coordenadas (Xlo 91'

7.

CAMPO DE EXTREMALBS

87

Si existe una solucin. u (x) de la ecuacin de Jacobi que satisface


la condicin u (xo) = O 11que ItO se anula en ningn otro punto del semiintervalo Xo < x ~ x, en el arco AB no habr puntos conjugados del
punto A. En este caso el arco AB de la extrema! se puede incluir en Un
campo central de exiremales con centro en el punto A (xoo y{).
En la euac6n (2) hay gue tomar en las funciones Fy'fl (x. Y. y'),
F 1111' (:c. y, 11 ) Y F '1/'11' (x, y, 11 ) en lugar de 11 (x) el segundo miembro de
la ecuacin de la extremal IJ = 11(x, C~).
EJEMPLO 10. Se cumple la condicin de Jacobi para la extremal
de la funcional

r
a

J (y (x)]

(y'2+xZ)

dx

que pasa por los puntos O (O, O) Y B (a. 3)?


SOLUClON. En este caso la ecuacin de Jacobl tiene la orma
u" = O. Su solucin general es u = C1x
e,. De la condicin u (O) =
= O encontramos e, = O de modo que u = Clx. Estas soluciones
u = CIx (el =1= O) no se anulan para ningn valor de a > O. Por consiguiente, en el arco OB de la extremal no habr punto conjugado del
punto O (0, O). Es decir, este arco se puede incluir en un campo central
de extrernales con centro en el punto O (0, O). Es fcil ver que la extre-

mal buscada es la recta y = ~x

que se puede incluir, obviamente,

en

de extremales y = e,x.
EJEMPLO J1. Se cumple la condicin de Jacobl para la extrema]
de la funcional
el campo central

J[y(x)]=

(a-4= (n+})

(y'2_4y2+e-:x )dx

n)

que pasa por los puntos A (0, O) Y B (a, O)?


SOLUCIONo La ecuacin de J acob l tiene la forma u
4u = O Y
su solucin general es u = Cl sen 2x
C~ cos 2-". De la condicin

u (O)

O encontramos

Ca = O de modo que u

la funcin u no se anula para O


cumple; en cambio, si a

>~

<

Cl sen 2-". Si a

<~

x .;;;;a y la condicin de J aeob i se

la solucin u

CI sen 2;c de la ecua-

cln de J acobi se anula en el punto x = ; perteneciente al segmento


{O. al y en el arco de la extremal y = O (O ~ x ~ a) hay un punto
conjugado del punto A (O, O). Por consiguiente, si a > ~ , no existe
campo central

de extr emales que comprenda

la extremal

dada.

En los problemas siguientes analizar si se cumple o no la


condicin de J acobi,

88

CAP.

IJ.

DE FUNCIONALES

EXTREMO
1

J (12xy+y'2+x )dx;

135. J Iy(x)]=

-1

y(-l)=

-2,

y(I)=O.

J (y'2+9y2-3x)dx;

136. J[y{x)=

y(O)=O,

y(a)=O.

J (1 +y(2)dx;

137. J ly(x)] =

y(O)=y(l)=O.

O
n

=)

ly (x)]

J38. J

y' eY' dx;

y (O) = 1,

y (a) = b.

o
2n
J 39.

Iy (x)]

140. Demostrar
cional

Io

(y'Z - y2) dx;

y (2n) = 1.

y (O) = O,

que si la funcin integrando

de la fun-

Jfy(x}J=

J F(x,

y')dx

no contiene y explcitamente, cualquier extrernal puede ser


siempre incluida en un campo de extremales.

Ir

oBSe~vACI()N. La condicin de Jacobi es necesaria para que la


funcional J
(x)l alcance su valor extremo: o sea. cuando la extrema!
A8 realiza e extremo. el punto conjugado de A no puede estar en el
intervalo XI) < x < Xl' Por ejemplo. la funcional

J[y(x)J=.\

(y'4+I)dx;

y(O)=g(a)=O;

alcanza su valor mnimo en la exremal y == O. En esta extremal no


hay puntos conjugados del punto O (O. O).
EJEMPLO 12. La Iunclonal
!I

T1t
Jlv(x)l=

<V2-V'Z)dxi

'1(0)=0,

!I(:

n)-o;

7.

CAMPO

no alcanza

!Jt)

(O,

DE

en la ex trernal y

extremo

89

EXTREMALES

==

O porque

en el intervalo

est el punto O (a, O) conjugado del punto O (O, O) (ya que

que se anula en x = O es u =
sen x y u se anula tambin en el punto x = tt E (O,
n) ) .

la solucin de- la ecuacin de Jacobl


=C1

Efectivamente. lomemos como curva prxima a y == O la curva


4
sen "5 nx
I/n (x) =
nll
; para esta curva se cumplen obviamente las condiciones y (O) = Y ( 5: )

J= J

1. lt

4
sen
J[

5TlX
n2.

O e

y~ (x) = 5~ cos n ~ x,

Entonces

= O y

J [O)

tenemos

sen2

i;-x)

dx-

n4

T"

_ Jr (_4_)2
cos2 (~x)
tlx= 8n2
5n (_l__ ~)
<O
5/t
5
25
112

para todo entero n ;;;.2. Por consiguiente. la extrema! y == O no realiza


el mnimo de la funcional considerada ya que existen curvas prximas
a U == O en las que son negativos los valores de la funcional. Consideremos abora la familia de curvas Un (x)

== O en el

sentido de proximidad

T7t
IUn

(x)]

= J

sen ~ x cercanas a la curva

de orden cualquiera.

5
J

Es facil ver que

~
sen2~.I:
5
.n2.

T.!t

16

2.

J 252C'OS"5

dx -

xdx=

9tt

40n2

> O.

Por consiguiente, la extremal y


de la funcional considerada.

141. Supongamos

!::!!I

O tampoco realiza el mximo

que en la funcional
b

J fy(x)] =

J F (x, y, y') dx
a

la funcin integrando F tiene derivadas parciales acotadas

de tercer orden respecto a las variables U e y' en cualquier

90

CAP.

11. EXTREMO

DE FUNCIONALBS

recinto acotado de variacin de y e y'. Sean y = y (x) e y =


= y (x) + '1 (x) dos extremales cercanas. Demostrar que la
funcin '11 (x) satisface la ecuacin de Jacobi

:x (FlIlI'T]+FII'v'T]')=O

FIIII'YI+FIIII,T]'-

salvo una infinitsima de orden superior respecto a la distancia de primer orden entre estas extremales.
2. Condiciones suficientes de l.egendre. Condicin sucente para
que la extrema} de la funcional
XI

J [y (x)l ~

J F (x, y, !J') dx;

y (xQ) = Yo,

xo
se pueda incluir en un campo de extremales es que se cumpla la condicin reforzada de Legendre, Esta consiste en que la desigualdad
FII,y'

>O

cumpla en todos los puntos de la extrernal considerada


todos los x E [xoI xll).
EJEMPLO 13. Consideremos la funcional

se

(o

sea, para

J(y(x)]=

y(O)=I, y(2)=5.

(y"+y'2)dx;

Sus extremales son las rectas y = C1x Ca. La extremal buscada que
satisface las condiciones de frontera es la recta y = 2x
1.
En este caso F 11'11' = 12y'i
2 Y en todos los puntos de la extremal y = 2x
1 tenemos F IJ'IJ' = 50 > O. Se cumple la condicin
reforzada de Legendre y, por consgulente, la extremal 11 = 2x
I
se puede incluir en un campo de extremales.
Esto se ve tambin directamente. La extremal 11 = 2x
1 queda
comprendida en la Iarnllia monoparamtrica de extremales 11 =o:
;:: 2x
a. (o: es el parmetro) que lorma un campo propio.
EJEMPl.OH. Consideremos la funcional

J (y (x)] =

(xlly'lI
-1

y(-I)=-I,

+ 12y2)

dx;

y(l)=I.

La ecuacin de Euler para esta uncional tiene la forma


x'yN

+ 2:ty'

Y su solucin general es
11= C1x:'

12y = O

+ Cax ....

58.

CONDICIONES

SUFICrENTES

DE EXTREMO

91

La extremal que satisface las condiciones de frontera consideradas es


g

= :x8.

No puede ser incluida en un campo. La (mica familia monopararntrtca


de extremales que la contiene es y = a:x8. Pero esta familia no cubre
el recinto que contiene el punto de abscisa x = O (porque las extremales
de esta familia no pasan por los puntos del eje Oy con ordenadas distintas de cero).
En este caso tenemos F 11'11' = 2x2 y la condicin de Legendre no
se cumple para x = o.

Analizar la posibilidad de incluir la extrernal en un


campo para las funconales siguientes
i

142. J (y(x)l=

y(O)=O,

~ (y'Z_yy'3)dx;
o

y(l)=O.

143. J[y(x)J=

J y'3dx;

y(O)=O,

y(a)=b>O.

XI

144. J[y(x)1=)

n(y)Vl+y'Zdx;
y (xo) = Yo,

y (Xi) = Yt,

n (y)

> O.

145.

J [y (x)) = ~(6y'2 - y'4.) dx;


'o
y(O)=O.

y (a) =b,

b>O.

8. Condiciones suficientes de extremo de una funcional


Se considera el problema variaclonal elemental, o sea, se considera
la funcional

:1:,

J [y (x))

F (x, 'l.

!I'l dx

con las condiciones de frontera


y (Xo) = Yu,

(1)

(2)
IJ (Xl) = 111'
l0. Condiciones suficientes de Welerstrass. Se denomina funcin
de Weierstrass E (x, y, p, y') la funcin definida mediante la igualdad
E (x, y, p, y') = F (x, y, y') - F (x, y, p) - (!I' - p) Fp (x, y. p)

donde p = p (x, y) es la inclinacin, en el punto (x, y). del campo de

extrema les del problema

variaclonal

(1) y (2).

CAP.

92

[1.

EXTREMO DE FUNCIONALES

CONDlCIONES SUPICIENTC.S DE EXTRE"'O DaBIL.

La curva e realiza el extremo dbil de la funcional (1) si;


1) la curva e es una extremal de la [uncionai (1) que satisface las
condiciones de frontera (2), o sea, es la solucin de la ecuacin de Euler
para la funcional (1) que satisface Las condiciones (2);
2) la extremal
e puede ser incluida en Un campo de extrema les
(esto tendr lugar. en particular. si se cumple la condicin de Jacobi);
3) La funcin de \reerstrass E (x, y, p, y') conserva su signo en
fodos los puntos (x, y) prximos a la extrema! C y para valores de y'
prximos a p (x, y). La funcional J ly (x)) tendr mximo en e si E ~ o
U mnimo si E;;;" O.
CONDICIONES SUFICIENTES DE EXTREMO FUERTE.

La curva e realiza el extremo fuerte de la funcional (1) si:


1) la curva e es una exiremal de la funcional (1) que satisface. las
condiciones de frontera (2);
2) la extrema! e puede ser incluida en un campo de extrema les,
3) la funcin de Weierstrass E (x, y, p, y') conserva su signo en
lodos los puntos (x, y) prximos a la extrema! e y para valores cualesquiera
de y'. Si E ~ O. se tendr mximo y, si E;;;" 0, se tendr mnimo.
OBSERVACiN. La condicin de Weiersirass es necesaria para que
exista el extremo en el sentido siguiente: si para ciertos valores de U'
la funcin E tiene signos opuestos en los puntos de la ex trernal, no se
alcanza el extremo fuerte; si esto ocurre para cualesquiera valores de
U' por prximos que sean a p, tampoco se alcanza el extremo dbil.
EJEMPLO 1. Analizar el extremo de la funcional
1

Jlu{x)j=

{y'3+U')dx;

U(I)=2.

y (O)=0,

O
SOLUCIN.

La ecuacin de Euler para la funcional considerada tiene

g'l' = O de modo que las extrernales

son las rectas y =


La extremal que satisface las condiciones de frontera
es la recta U:::. 2x. La inclinacin del campo en los puntos de esta
t~jIema.l es p = 2. Es evidente que la extremal y = 2x se puede
ioclu ir en el campo central de. extrernal es y = C con centro en el
punto O (O, O). Es fcil probar as mismo que en este caso se cumple la
la forma

::::::~G.l~+ C,.

con dlcin de Jacobi. La ecuacin de J acobi es en este caso - :,. (6y' u')=

= O, donde,

debido a la ecuacin de la extremal, hay que poner y' == 2.


Por consiguiente. la ecuacin de Jacobi toma la forma UN = O de modo
que u = C1x
C2 De la condicin u (O) = O obtenemos
= O.
Puesto que para el :::i= O esta solucin u = el): no se anula en ningn
punto a excepcin del punto x = O, la condicin de J acobi se cumple.
Formamos la funcin M Weierstrass

~ (XI y, p, {)

= y'3

e~

+ y'

_ p3 _ P _ (!J' _

+ 1) =
= (t - p}3 (V' + 2p).
p) (3p2

p.

CONDlIONE.S

SUFICIENTES

93

DE EXTREMO

El primer factor es no negativo cualesqu lera que sean los valores de


l 2. Por consiguiente, se cumplen todas las condiciones de mnimo ddlil. Adems,
es fcil ver que para y' < -4 la funcin E ser negativa y no se cumplir la condicin suficiente de extremo fuerte ya que para el extremo
fuerte se exige que la funcin de Weierstrass f] conserve su signo para
valores cualesquiera de y'. Si se
tiene en cuenta la observacin de
la pg. 92. se puede llegar a la conI
clusin de que en este caso no hay
extremo fuerte.
EJEMPLO 2. Analizar el extre-

y' y el segundo es positivo para valores de y' prximos

mo de la funcional
1

Jlg (x)] =

Jo (X+2Y+!

y(O)=O,

2) dx;

y(I)=O.

SOLUCIN.
La ecuacin de
Euler para esta funcional tiene la ---'o*--"""'7'--_''--~>-:--~'''/(
forma y~ = 2. Las extrernales son
las parbolas y = X2
elx e2
La extrernal
que satisface
las
condiciones de frontera es y =
= X2 -)C. Formamos la ecuacin

de J acob -

:x u'

=- O.

u' = O. Su solucin

sea,

general

es

u = ex + e2
La condicin
u (O) = O da e2 = O Y la con dicln J acobi se cumple ya que
u

= e1x

con

el '=1= o

no se anula

en ningn punto del segmento [O, 1]


a excepcin del punto x = O; es dectr, la ex tremal !I = X2 -)C se

Fig.. 11

puede incluir en un campo central


de extrernales con centro en el punto O (O. O). a saber. en el campo
y = X2
ex (fig. 11). La funcin de Weierstr ass tiene la forma

(x, y, p, y') = ~ (!J' -

= ~ (y'

- p)2

la funcional

> O para

considerada

fuerte igual a J (x';\ -

p)2.

Se puede

cualesquiera

ver

de aqu

valores de y' Por consiguiente,

alcanza en la ex trernal y x] ~

~ .

que E =

X2 -

x mnimo

CAP. 11. EXTltEMO

DE FUNCloNLt:S

Analizar el extremo de las funcionales siguientes.


1

5 ~(y2+

146. J[y(x)l=

; y'2)dx;

u(l)=e.

y(O)=l,

147. J IY(x)] =

Ja

e'IIU'2dx; g(O)

=0, u(I)=ln4.

x3
148. J[y\x) = Jr y'Z
dx; g(I)= 1, g(2)=4.
i
el

149. J[U(x)=

Jo :~ ;

150. J(U(x)=

J (1+x)y'2dx;

U(O)=O,

g(a)=b,

b>O.

y(O)=O,

y(I)=1.

y(O)=I,

o
n/2

151. J[y(x)l=

J (lI'-!j2)dx;

(~)=1.

152. Jly(x)1=

y' (1 x2y') dx; y(-I)=l,

y(2)=4.

-1
t

153. J fu (x)) =

J (y'3+

y,a) dx; y( -1) = -1,

y(l) = 3.

-1

2. Condldones

suficientes

de legendre.

Supongamos

que la

derivada parcial F 11'11' (x, y, y') de la funcin F (x, y, y') es continua


y que la extrernal e est incluida en un campo de extremales.
SI en la extrema! e se tiene F1/'11' > 0, en la curva e se alcanza
mnimo d~bll; si en la extrema! e se tiene FlI'lI' < O, en ella se alcanza
el mximo d~bil de la [uncional (1). Estas condiciones se llaman condiciones reforzadas de Legendre.
En el caso en que F JI'IJ' (x, y. y') ;;;;.O en todos los puntos (x. y)
prximos a la exttemal e y para cualesquiera valores de y' , se tiene mlnmo [uerte y en. el caso en que FfI'y' (x. y. y') ~ O para estos valores de
los argumentos, SI! tiene mximo uerte.

8,

ONDJCU)NES SlJPICJENtES DE EXtREMO


EJEMPO 3: Analizar

el extremo

9$

de la funcional

J(Y(X}l=

~ (y'3-o.y')dx;

y (0)=0, y(I)=-2

o
(a. es un nmero real cualquiera).
SOLUCIONo Puesto que la [uncin integrando depende
las extrernales son las rectas y
CIl:
Cs. La extrernal

slo de !J',
que satis-

face las condiciones de rontera es la recta g = -2x que se puede incluir


en el campo central de extrernales !J = Cx. La inclinacin del campo
en esta extrema! es p = -2. A continuacin, calculamos F 1/'1/' = 6y'.
En la extremal tenemos Fu'u' = -12 < 0, o sea, la funcional alcanza
mximo dbil en la lnea y = -2x. Para valores arbitrarios de y'
el signo de FI/'u' no se conserva y, por consiguiente. no se cumplen las
condiciones suficientes de mximo fuerte.
En este caso la funcin de Welerstrass E (x, y. P. g') tiene la forma
E (x. y, p, !J') =

(J/ -

p)2. (JI'

+ 2p)

y para determinados valores de y' tiene signos opuestos. Teniendo en


cuenta la observacin de la pg. 92, deducimos que no hay mximo
fuerte.
EJEMPLO 4. Analizar el extremo de la funcional
2

J Iv

(x)] =

y (O)=0,

(eY' +3) dx

V (2)

= I.

o
SOLUCIONo Las exlremales

son las rectas y=C1x+C2.'

La extre-

mal que satisface las condclones de frontera es la recta y = ; ; puede


ser incluida en el campo central de extremales y = Cx. Tenemos. adems . FII'II' (x, y, y') = ell' > O para cualesquiera valores de y'. Por
consiguiente,

la funcional tiene mnimo fuerte en la extremal !J=1.

Vr

EJEMPLO 5. Analizar

J(y(x)}=

Jo

SOLUCION. La funcin

el extremo de la funcional

dx

y (0)=0,

y (a)=Yi'

integrando

x y, por eso, tenernos F - y' FU' =

no depende explcitamente
caso,

el o, en nuestro

de

CAP. 11. eXTREMO

DE FUNCIONALES

de donde
J
~-=-~==-=CI

VyYl+y'2
donde CI = (~I )2 . Pongamos
= ~1 (I-cos/),

t
clg"2

y'

= ctg

y {lry'2)....eh

2t . Tendremos

I
y = ej sen'2.2" =

= el sen t dt el sen2..!.. dt
t

2ctg"2

encontramos

_e r
x- I J
Tenemos pues

Adems.
dx =.......!!.J!_

e, integrando,

(1 - cos t) dt
2

(c = ~

el

=T(t+se.nt)

+ C2

)
x =C (t - sen t) + C2,
y= e (l-cos t),

o sea, las ecuaciones paramtricas

de una lamilia

de cicloides.

De la

J(

Fig.

condicin

12

C2 = O. El haz de icJoides
x= C (t -sen r), }

(O) = O encontramos

g=C (l-cos
forma un campo central con centro en
la extremal
x=R (t -sen
y ~.R (1 -cos

1)

el punto

(O, O) que comprende

t), }
t),

donde R se determina de la condicin de que la cicloide pase por el


segundo punto frontera B (a .vI), s a < 21tR (fig. 12).

S.

CONDICIONES

Empleamos

SUFICIENTES

la condicin

DE EXTREMO

de Legendre.

FU'lI'=

97

Tenemos

>0

Yv (1 +y'2)2
para cualesquiera valores de y'. Es decir, para a
considerada tiene mnimo fuerte en la cicloide

< 2nR

la funcional

x=R(t-sent),
}
g=R (l-cos t).
Empleando la condicin de Legendre, analizar el extremo

de las funcionales siguientes:


1

J54. J[y(x)]=

S (y'2+x2)dx;

y(I)=l.

y(O}=-I,

o
3

155. J (y (x)] =

x~s dx;

y (2) = 4,

y (3) = 9.

2 Y
2

156. J{y(x))=

5 (xy''''-2yy'8)dx;

y(I)=O,

y(2)=

1.

1
a

157. JIY(x))=

5
J yy'

(l-e-II'S)dx;

y(O)=o,

y(a)=b.

158. J(y(x)]=

2dx;

y(O)=p>O,

y(l)=q>O.

159. Analizar el extremo de 1a funcional


1

J lY(x)} = ) (ey,z+y2+x2)dx;
y(O)=O,
o
para distintos valores del parmetro ~"

y(t)=];

EJEMPLO 6 (problema de Euler), Una barra vertical de longitud 1


Se somete a una carga axial P. Para un valor determinado de P (fuena
crtica de Euler) la barra se comba. Se pide determinar el valor mlnimo
de la fuerza P querovoca
la flexin longitudinal.
SOLUCIN. Sea
el mdulo de elasticidad, sea I el momento de
inercia mnimo de las secciones transversales de la barra, se p el radi
de curvatura y sea q .el ngulo entre la tangente y el eje.
1-01387

CAP. Il.

9B

La energa
frmula

EXT~EMO

potencial

DE FUNCIONALES

de la flexin

se determina

mediante

la

Ut="2 El

f dS
J V'
o

Si el ex tremo de la barra desciende en


I

O" =

cp) as,

(I-cos

la energa potencial de la barra disminuye

en

Uz= Pa=Pl-P

~ coscpdS.
O

Si antes de la deformacin

la energa potencial es igual a cero,


despus de la deformacin estar dada por la Irrnula
l

U=U,-U:l=

J (~

El p~ +P~oscp)dS-Pl.

dS
Puesto que p = dcp Y (para pequeos valores

de q

cos q>:=:::: 1-

cpu

T'

se tiene
I

fI

j [E

f ( ~~ ) 2_ P!p2] d,
O
En el caso de equilibrio, la energa. potencial toma su valor mini
mo. Por eso. el problema se reduce a la determinacin del mnimo de
la integral

U=

[El

( ~; ) 2- P<pll] dS:=::::~

En este caso
F=EI
Y la ecuacin

(!~) _Pcp~
2

de Euler tiene la Iorrna


q>"

+a.'aq>=O,

donde

a.2 =

La solucin general de esta ecuacin es


cp = Cl sen a.x
Ca cos

:1 .

a.x.

58.

CONDICTONES SUP1CIENTES DE EX1REMO

99

Puesto que tg q ~ q> para valores pequeos de q y como, adems,


tg q> = y', se tiene
y' = Cl sen ax + el cos ax,
de donde
Ctcosez,x+C2senax
C.
g=

ex

Si el extremo inferior de la barra est en el origen de coordenadas,


ser y = O para x = 0, o sea, Cl == C = O:

y= -1. sen ex.


a.
Veamos si se cumplen las condiciones de Legendre y de J acobl,
Es obvio que la condicin de Legendre se cumple:

(JzF

= 2EI > O.

oq>'1!
La ecuacin de J acob

tiene la forma
E l z" + Pz = O z" + a.'z = O
con la particularidad
de que z (O) = O. Por eso, la solucin
ecuacin de J acob i ser
z = A sen ax.

La funcin z se anula para


condicin

Xli

=.!!!:..
(k =
a

de J acobi se cumplir

si

de la

1, 2, ... ) de modo que la

l,;;:a.!!...
a.

De aqu

n3
P>--El.
El valor mntrno de la fuerza critica de Euler ser

ni
Prnfn=--EI
y la ecuacin

de la curva de Ilexin ser


Cz
nx
y = -;:;:sen -1- ,

3. Figuratrl2.

Sea dada la funcional


b

J IY (x)1= } F (x, y. y') dx .


a

Tomando x e y como parmetros, consideremos la funcin y =


y, y') en tanto que funcin del argumento !J'. El grtco de
esta [uncin en el plano de las variables (y'. Y) se denomina figuratriz.

= F (x,

100

CAP.

rt. EXTREMO bE PUNCIONA LES

Es fcil probar que la funcin de Weierstrass E (x, y. p, !J') representa


la diferenca entre las ordenadas de la figuratriz y las ordenadas de la
tangente a la liguratriz trazada por el punto de abscisa y' = p. Si la
{uncin de Weerstrass conserva su signo para ciertos valores de g'.
ello significa que la figuratriz est por encima o por debajo de la
tangente para esos valores de !J'. En este caso hay mnimo dbil. Si
la Ilguratrtz est a un lado de la tangente para todos los valores de
y' y para los valores de los parmetros x e y prximos a los puntos de
la extremal, hay extremo fuerte.
La condicin suficiente de Legendre se expresa en estos trminos
as: si para todos los puntos (x. y) prximos a la extremai la ftguratriz
es cncaua hacia las y positivas o negativas, hay extremo fuerte.
EJEMPLO 1. Analizar el extremo de la funcional
(1

Jly(x)=

~ y''Iodx.;

y (0)=0,

y(a)=b,

b>O.

SOLtxrON. Las exlremales son las rectas g = C1x


C2 La extremal buscada viene dada por la ecuacin y = ~ x, Puede ser inclulda
ti

Fig. 13
en un campo central de extremales, La figuratriz es la parbola
y = y'~ (f1g. 13). Es fcil ver que toda 1a figura triz est por encl ma

CONDICIONES SUFICIENTES

8.

de la tangente

101

DE EXTREMO

trazada a la misma en el punto p =

s: cualesquiera
a

que sean a y b (a =F O). Por consiguiente, la funcional considerada tiene


mnimo fuerte en la extremal y
EJEMPLO 8. Analizar

= ~ x_

el extremo de la funcional

Jlu(x)l=

g'Sdx;

y(O)=O,

y(a}=b,

b>O.

O
b

La extremal buscada es la recta y = - x que se puede


a
incluir en el campo central de extrernales y = ex con centro en el
SOLUCiN.

Fig. 14
punto O (0, O). La fuguratriz.

es la parbola cbica Y = y'S (fig. 14).

Para valores de y' suficientemente

prximos al valor p =

!!..
la
a

ratriz est sobre la tangente a la misma en el punto de abscisa y'

rigu-

= !...
a

De la lig. 14 se puede ver que la figuratrlz corta la tangente en el punto


de abscisa y'

=-

2b ya la izquierda de este punto aparece por debajo


a

102

(.AP.

de la tangente.
b

=-x.
a

JI. ~XTRf.MO

Por' lo tanto.

DE PUNCIONALl,S

hay

mnimo

dbil

en la extremal

Ntese que para p = O (esto corresponde al caso fJ == O en el que


la extremal es un segmento del eje Ox) la tangente a la figuralriz es
el eje Oy' y el punto O (O, O) es un punto de inflexin de la figuratriz.
Teniendo en cuenta la observacin de la pg. 92. vemos que en cualquier vecindad del punto O (O. O), por pequea que sea, la Ilguratrlz
tiene ordenadas tanto positivas como negativas. Por lo tanto. la
funcin de Weierslrass E tiene signos opuestos para valores de JI'
tan prximos a p = O como se quiera y. por consiguiente, en esto caso
no se alcanza ni siquiera el extremo dbil.
EJEMPLO 9. Probar que la ex trernal JI = O del problema variacional

=)

JIY(Xil

(y't-yy'3)dx;

y(O)=y(I)=O;

realiza el mnimo dbil de la funcional.


SOLUCION. En este caso la condicin de Legcndre da
FlI'II,I/la.e()= (2-6yy')

1/1'-"'0
-.: 2 > O.

o sea, se alcanza mnimo db il en la extremal y = O. Demostremos


(fue en esta extrema! no se alcanza el mnimo fuerte. Consideremos la
y

y'

Fig. 15

>

figuratriz y = y'" - !JY" para los valores y


O (fig. 15). De la fig. 15
se ve que la tangente a la Ilguratrtz en el punto de abscisa p = O corta
la figuratriz
con y

en el punto y' =

> O. prximos

_!_.
Es decir, para los puntos (x, y),
y

a los puntos de la extremal y

O. la funcin de

Weierstrass E es positiva para valores de g' menores que ~ y es negatl-

8.

CONDICIONES

!'UP(CJENT~S

103

DE FXTl~F.MO

2...

va para y' >


Segn la observacin de la pg. 92, no hay mlnlmo
y
fuerte. Una situcin semejante Se tiene tambin para y < O.
Lo que destaca este ejemplo es que en l la condicin FI/'Y' > O
se cumple en la extremal para cualesquiera y' y, sin embargo, ello
no implica la existencia de extremo fuerte.

Empleando la figuratriz analizar el extremo de las funcionales siguientes:


j

160. J [y (x)] =

Jo

161. Jly(x)l=

J y (l+x y')dx;

(1+x)y'Zdx;

y(O)=O,

y(l)=-2.

y(-1)=y(2)=1.

-1

162. J[y(x))=

J (1-e-II'4)dx
O

y(0)=0,

y(a)=b,

(b>O).

y(a)=b

(10).

163. J(y(x)J=

J (6y'2_y'''+yy')dx;
O

y(O)=o,
OBSERVACIN. La 110 negatividad
dicin necesaria, pero no suficiente,

de la segunda variacin
para que la funcional

alcance mnimo en la curva.


EJEMPLO 10. Consideremos la funcional

es con[y (x)

1[y (x)1=

J y' (x-y)

dx

o
en el espacio e [0, l], La ecuacin de Euler tiene la forma FII = O
y = O. Para O ~ x ~ 1 la segunda variacin de la funcional en la
extremal y = O
1

{Ji! (O,

6Y1 =

X (<'Iy)3 dx

o
es positiva para todo ay ::6 O. SIn embargo, en cual quier vecindad del
cero la funcional J [9 (x)l toma tambin valores negativos; basta fijar

(.;AP. 11. EXTREMO DE FUNCIONALES

lO4
8

>- O Y considerar

la funcin

!J~ (x) = {

-x+s,
O,
8'

Entonces tendremos J (Ya (x)] = -6

x:;;;"e.

< O para

cualquier e> O.

DEFINICION. Se dice que \a funcional cuadrtica L, (h] definida en


un espacio narmado es fuertemente posi/lva si existe una constante
k> O tal que
para todo h.
CONDICION SUFICIENTE DE MtNIMO. Condicin suficiente para que
la [unclonal J 111 (x) definida en un espacio normado tenga mnimo
en el punto estacionario y = Yo es que su segunda variacin sea fuertemente positiva en y = Yo. o sea, que se cumplo. la condicin
(J'J I Yo. y) ;;;, k 11 6y IIz

= consto k
4, Supongamos

donde k

> O.

que se busca el extremo de la funcional

%1

J [lIit

Uz, . oo, Ynl=

j F (x.

!lit yz. ''''

{In.

!ll,

Y. oo', y;'> dx,

(3)

:1:0

que depende de n funciones Yl (x), Ya (x) ..


ciones de frontera
YA (Xo)

= YkG.

YIt (Xl) = YIt]

(k

, !/n (x), con las condi-

1, 2 .

o. n).

La condicin reforzadcl de Legendre consiste en que las desigualdades

F~.",
1 n
FI/'II'

In

FII''''
.
n1 F~.",
ni

o o

>0

(4)

F",,,,
nn

se cumplan en todos los puntos de la extremal considerada de la Iuncional (3).


La oondicion reforzada de Jacobi consiste en que el segmento
(.to. "J1 no contenga punto conjugado del punto XO

CONDICIONES

f 8.

SUFICIENTES

105

DE EXTREMO

La condicin reforzada de Legendre (4) conjuntamente con la


condicin reforzada de J acobl garantizan por lo menos la existencia
del mlnlmo dbil de la funcional (3).
EJEMPLO 11. Analizar el extremo de la funcional
1

J fu (x), z (x)]

y (O) = 0,
y(l)=t,

z(I)=2.

SOLUCION. Las ecuaciones

(5)

(y'B+2'2) dx;

o
z (O) = o,

(6)

de Euler para la funcional

y- = o,

(5) son

z = O.

de modo que

y=Cj +C2x, }
z=Cd C/ox,
Empleando

las condiciones

CI
La ex tremal

= O.

(7)

(6), obtenemos

C. = l.

Cs

=O

C, = 2.

buscada

y=x, }
z=2x

(8)

representa una recta que pasa por el origen de coordenadas.


Tenemos
Fy'Y' = 2.

La condicin

Fy'z'

reforzada

FY'y,=2>0.

= 0,
FZ'II' =
y
de Legendre se cumple:

IFr.'Y' FY'~'1=12
FY'I/'

Fz'z'

FZ'%'

2.

1=4>0.

10 2

Veamos si se cumple la condicin reforzada de Jacobl.


Una de las definiciones de punto conjugado es la siguiente (vase
[3)).
Supongamos que se tiene una familia de extremales de la funcional (3) que arrancan del punto inicial (xo. YIO. ' , ., YnO) en direcciones prximas rero linealmente
independientes.
Se dice que el
punto x" E [xo, Xl es conjugado del punto Xo si existe una sucesin
de extrema les, que arrancan todas del punto inicial y que son tan
prximas como se quiera a la ex tremal considerada, tal que cada una
de estas extremales corta la extremal considerada con la particularidad de que las abscisas de los puntos de interseccin convergen hacia
el punto X,
En nuestro caso las extremales son las rectas (7). Todas las extremales que arrancan del punto (O, O, O) cortan la extremal (8) en este

c,'\I'. 11. EXTREMO DE FUNCIO .'ALES

106

punto solamente Por lo tanto. el segmento (O IJ de variacin de x


no contiene punto conjugado del punto Xo = c. Es decir, se cumple
tanto la condicin relorz ada de Legendre como la condicin reforzada
de Jacob de modo que la extremal (8) realiza el mnimo dbil de la
funcional (5).

Analizar el extremo de las funcionales siguientes:


t

164. J[y(x).

z(x)=

J 'Vl+y'2+z'2dx;
o

y(O) ,,,0, y(I)=2.

z(O)=O,

z(1)=4.

165. J [y (x), z (x)] = ) (y'2 +Z'2

+ 4z) dx;

o
y(O)=O,

y(I)= 1, z(O)=O,

z(I}=O.

9. Extremo condicionado
Sean F (x, Y. [1') y G (x, y, y') dos

l0. Problema Isoperlmtrce.


funciones.

El problema isoperimtrico consiste en lo siguiente: entre todas


las curvas y = y (x) E el [':<0' Xl] a lo largo de Ias cuales la funcional
;tI

[y (x)]

JG

(x, y, !J') dx

xo
t iene

un valor fijo!

hallar la curva en la que la funcional


!lel

J [y (xli

=)

F (x., y, y'l dx

xo

alcanza su valor extremo.


Suponemos que las funciones F y G tienen derivadas parciales
continuas de primer y de segundo rdenes para Xo ~ x ~ Xl Y para
valores cualesquiera de las variables y e y'.
TEOREMA DE EULER. S la curva y = y (x) realiza el extremo de la
funcional
Xl

J !y (x)l-=

Xo

F (X, y, y') dx

p.

EXTREMO CONDICIONADO

107

con las condiciones


Xi

G(x, y, y')dx=l,

K[y(x)l=)

Y (Xo) =Yo.

Y (Xt}=Yt.

xo
y si U = Y (x) no es extremal de ta funcional K existe una constante A
tal que la curva y

!I (x) es extremal de la funcional


XI

L (11 (x)) =

[F (x, y, y')+W (x, y, .11') dx,


:ro
EJEMPLO I (problema de Dido). Entre todas las curvas cerradas
de longitud 2l hallar la curva que comprende el rea mxima.
SOLUCION. Observemos, ante todo, que dicha curva debe ser convexa. Efectivamente,
de lo contrario encontraramos una recta L

Fig. 16
(Iig. 16) tal que, al reflejar en elJa la parte BCD de la lrontera, obtendramos un recinto de rea mayor que el inicial siendo la longitud
de la frontera la misma que antes.
Observemos tambin que toda recta que div ide por la mitad la
cerrada que comprende el rea mxima ha de dividir por la mitad la
propia rea. Efectivamente, supongamos lo contrario y sea L1 una recta
que no cumple esta propiedad. ReUejando en L1 la parte de la figura
de rea mayor, obtendremos una curva de idntica longitud pero que
comprende un rea mayor.
Tomemos como eje Ox cualquiera de las rectas que dividen por
la mitad la curva; llegamos entonces al problema siguiente.
Hallar la lnea y = y (x), Ij (-a) = y (a) = 0, de longitud fija
1 > 2a que conjuntamente con el segmento -a ~ x ~ a del eje Ox
encierre el rea mxima. Por lo tanto. el problema se reduce a hallar
el extremo de la funcional
el

J[y(x))=

-a

ydx;

y(-a)=y(a)=O;

108

CAP. 11. EXTREMO

con la condicin complementarla

DE Pl:1NCIONALES

de que

K(y(x))=

VI+y'3dX=1

(/>2a).

(1)

-4

Formemos la (uncin auxiliar

H=F+AG=y
y consideremos

la Iuncicnal

(X)+A 'Vl+y'2

(x)

auxlllar
a

L ty (x)) =

JH

(x, y, y')

dx,

(2)

-a

La ecuacin de Euler para la funcional (2) tiene la forma


-d (

dx

AY'

VI +y'a

) -1

de donde

Ay'

VI +y'"
Resolvrendo la ltima ecuacin respecto a y', encontramos

dy

x+e

-= 'V),II_(x+e)i

(3)

Integrando la ecuacin (3). obtenemos


(x+cl)a+ (y+e2)2=A:a,
o sea, la cireunlerencia de radio ~ con centro en el punto (-el. -es).
Las constantes el y C~ as como el parmetro A se determinan de las
condiciones de frontera y (-a) = y (a) = O y de la condicin isoperimtrica (1). Tenemos
C=A2-(C1-a)2,

q= ),3_ (e. +a)Z,


de donde
de modo que

Y=V~-V~

~xt~eM

109

CNDICINAD

La condicin (1) da entonces

1 ')..

_dx

l=

=~arcsenT

V),2.-XZ

!x-a
x--01

=2J..arcsenT

el

-a
o sea,

--=sen

2~ .

Resolviendo respecto a ), esta ecuacin trascendente, encontramos un


valor determinado A = 1..0 Y despus encontramos el valor de C2 =
= V),g - a~.
Es fcil persuadirse
pre solucin.

Efectivamente,

reducida a sen t

tomando

sen ;').. tiene siem-

~J.. = t, esta ecuacin quedar

~a t, donde ~ =<1 por hiptesis del problema.

de la tangente a la funcin y = sen

La ncllnacln

es :

de que la ecuacin ~

mientras que la inclinacin

t en el punto t

=O

de la funcin y =1 es menor. Por

consgulente, los grficos de estas funciones tienen un punto de nterseccin como minimo, a parte del punto O (O, O).
PRiNCIPIO DE RECIPROCIDAD EN EL PROBLEMA ISOPERIMETRICO.
de la funcional

Las extrema les

%1

J
Con la condicin

19 (x)I=

F (x, y, y') dx

complementaria
%t

K I.Y (x)]= ) a (x, Y. y') dx= consl


"'O

coincIden con las extremales de la funcional K [1/ (x) ron la condicin


(y (x)] = const.
Basndonos en el principio de reciprocidad. deducimos del problema de Dido el resultado siguiente: entre todas las curvas cerradas que
comprenden un drea fiia. la circunferencia es la curva de longitud
mlnima.

Es Icll obtener este resultado directamente


forma paramtrica
del pro blema var actona 1.
Sean
x=X{t),
x (to)=x(tt),
}
!I = y (/), y (to) =u (t1),

to~

si se recurre

t '"

tit

8,

la

110

CAP, 11. EXTREMO De FUNCIONALES

las ecuaciones de una curva cerrada.


el extremo de la funcional

El problema consiste en hallar


1

f (~2+y2/
con la condicin

dt

J (xy-yx)dx=C.

Introduciendo la funcin
1

F={X2+y2)2+

~(x'-y~),

encontramos (vase la pg. 6S) que la curvatura

de la curva que

realiza el extremo es constante:

.!.=A.
r
Por consiguiente, la extremal buscada es una circunferencia.
Utilizando el principio de reciprocidad. se pueden resol ver, sin
realizar clculos, algunos problemas cvarlacionales de la Geometra
elemental.
EJEMPLO 2. Demostrar que: 1) entre todos los tringulos de base
y permetro fijos. el de rea mxima es el tringulo issceles; 2) siendo
fijas el rea y la base, el tringulo issceles es el 'tringulo de permetro mlnimo.
SOLUCION. 1) Tomemos una elipse cuyos focos son los extremos
de la base de los tringulos considerados (fig. 17). De la propiedad
y
CO(o,b)

Fig. 17
de 1a elipse deducimos que todos los tringulos A CB tienen el mismo
permetro. Es evidente que el rea mxima corresponder al tringulo

n.

EXTREMO

CONDrcrON\DO

111

de altura maxirna lo que signHica que el ver tice del tringulo debe
coincidir con el vrtice
o de la elipse. El tringulo ACoB es Issceles.
2) Segn el pr incip io de reciprocidad, siendo fijas el rea y la
base. el permetro mnimo corresponde al tringulo issceles.
EJEMPLO 3. Hallar el mnimo de la integral

J [y (x))

"

y'2dx

l)

It

y2dx= 1, y(O)=y

con las condiciones)

ln),=O.

SOLUCiN.

Formemos la funcional auxiliar


l't

L [y (x)

(y'2+Ay2) dx

f consideremos

su ecuacin
2A.y-

:x

de EuJer

o sea. y"-ly=O.

(2y'}=O.

(4)

Vl.

Su ecuacin caracterstica
es ,2 - 1.. := O, de donde '1,2 =
Est claro que A debe ser menor que cero: si aceptamos que A > O,
la solucin general de la ecuacin (4) tendr la forma y = ele V~x +
Cae- V;:x, las condiciones de frontera y (O) = y (n) = O se cumplirn slo para Cl = 0, C2 = O, o sea, resultar y == O Y no se cumplir

en este caso la condicin

J" f

dx

1; de la misma forma, si J.. = O.

la solucin de la ecuacin de Euler (4) que satisface las condiciones


de frontera tambin ser la funcin !J e; O. Por eso, consideramos que
A. < O de modo que 'l.a == i V=Xl y la solucin general de la
ecuacin (4) es !I = el sen
-1...:<
C2 cos
h.x:. La condicin
y (O) = O da Ct = O Y la condicin
!J (n) = O da -A = kl
(k = 1, 2, ... ). Es decir, y = Cl sen k.x, donde el no se ha determi-

1f

nado

an. Utilizando

la condicin

y2 dx

O
1'1

J Qscn kxdx=
2

1,

1, obtenemos

CAP. 11. EXTREMO DE flUNC10NALES

11.2
de donde

el =

males y =

V !.o

V!

sea, IJ ==

sen kx pasan

y=

slo para dos, a saber,

=J

sen kx. Todas las extre-

por los puntos

(O, O) Y (n, O), pero

Ji !

sen x, se cumple

de Jacobi. En estas dos extremales


J [y (x)l

V!

la condicin

se tiene

y'2 dx =

2 cos2
J -;t

X dx= J.

EJEMPLO 4 (problema de Kelv in . Supongamos


que en el plano
xOy est distribuida
una masa de densidad continua !1 (x, y) y supongamos que se tiene en el plano una curva e suave a trozos y dos puntos
P1 y P2 sobre la misma. Entre todas 1as curvas de longitud rija t
que unen los puntos Pi y P'1. hallar la curva que conjuntamente
con
el arco PIP'/. de la curva e forme un recinto D de masa mxima. Los
puntos P1 y P.! pueden coincidr.
SOLUCION. Consideremos
la funcin
v (x, y) =
Segn

la Irrnul a de Green,

J J l.ttx,

!1(x, 11) dx,

tenernos

j J ~~ dXdy=fVdYt

y)dxdy=

donde el contorno r se compone de la curva L y de la parte PIP,


de la curva C. A lo largo de esta ltima parte la integral toma un
valor determinado que designaremos por K. Aceptando que la curva
L est dada M forma paramtrica

x=x (/), }
11= Y (t),
tendremos

lo ~ I ~ th

entonces

J J !1(x,

tI

y)dxdy=

V (x, y)Ydi+K.

t~

Por consguiente, el problema ha quedado


nacin del mxtmo de la funcional
It

Ir,=

JV
t2

(x, 1/)

y dt

reducido a la determi-

EXTREMO CONDICIONADO

9.

con la condicin

113

de que
tl

~ Y;Z+y2dt=1.

Consideremos la funcin auxiliar

F=Vy+" V;:>. +yZ


y empleemos la forma de Weierstrass de la ecuacin de Euler. Tenemos

av

F = iJx'

F.

xv

yx

F, =

= O.

-==- = --__" 2
A

yZ

(x2+y2)

de modo que la ecuacin de Euler en la forma de Weierstrass


1

(:l>

av

7='1'

iJx

o, recordando la expresin de la funcin V (x. U'J.


I

7=

~t (x, y)

A.

donde r es el radio de curvatura de la curva pedida.


En el caso en que fA. (x, y) = const resulta que la curvatura de
la curva pedida es constante y, por consiguiente, las extrernales son
circunferencias, Queda claro que realizan el mximo de la funcional
JL'

Tambin se denominan problemas {SorerimtricoS los problemas


variacionales en los que se pide hallar e extremo de la funcional
Xl

(Yi> I/z, .. '. Yn]=

J F (x, Yt. !lz .

" Un,

Yi, Y, ., '. Un) dx

(5)

:reo

con las asl llamadas condiciones isoperimtricas


:>:1

..~ G (x,

Ul' !lz....

Yn.

y~. y;, . '"

y~)dx=ll

l6)

:>:0

(i=I.2,

... , m),

donde 1, son unas constantes.


Para obtener la condicin necesaria fundamental en el problema
soperimtrico sobre la determinacin del extremo de la luncional (5)
8-01R67

CAP 11. EXTREMO De FUNCIONALES

114

con las condiciones (6) hay que formar la lunclonal auxiliar


Xl
m
4l>

IY,

Y2, ... ,

Unl

J (F + ,

,G ) dx ,

.=1

xo

donde Al son unas constantes y escribir sus ecuaciones de Euler. Las


constantes arbitrarias Ch C2, , C2n de la solucin general del
sistema de ecuaciones de Euler as como las constantes Al' Az,
. . . , Am se determinan de las condiciones de frontera

!JII (.to)

!111o.

YII (Xl)

= YII1

= 1, 2, ... ,

(k

n)

isoper imtricas (6)

y de las condiciones
XI

IOidx='t

... , m).

(1=1,2,

X(I

EJEMPLO 5. H aliar

sobre el extremo

la ex tremal
de la lunctonal

en el problema

isoper imtrico

IY (x). z (X)

(y'2

+ Z'2_

4xz' -42) dx;

II

y(O)=O,

z(O)=O.

y(I)=I,

2(1)=1;

con la condicin
1

(y'2-xy' _Z'2) dx= 2.

la funcional 3uxi liar

SOLUCION. Formamos
1

a>IY(x),

z(x)j=

J [y'2+Z'Z-4xz'-42+)"

(y'2-xy'_z'2)

(1

y escribimos

para ella el sistema de ecuaciones de Eu ler


-

d~ (2y'

+ 2)..y' -

).x)

-4 -.!!_ (2z' -4x-21.z')


dx

.esolviendolo,

encontrarnos

= O.
=0'

'

dx

S 9.

EXTREMO

115

CONDICIONADO

Las condiciones de frontera dan

Ct

_ 3J..+4.
2

C3=2 {J -A}

C2=O,

de modo que

y=

J..x2+(31..+4) x }
4 (1 +J..)
,

z=x.
Para determinar 1.. recurrimos a la condicin Isopertmtrica (7). Puesto
2)..x+3A+4

que y = 4 (1 +i..)
y l = 1, obtenemos
I

r [(2).X.+3A+4)2

,] dx=2.

(2A.x+3A.+4)x
4(1+/..)

16(1+).)2

de donde, despus de unos clculos sencillos pero voluminosos,


nemas para i.. la ecuacin siguiente:

(23).2 +46"-+24)

De aqu resulta 1.,1 =

- :~ y

obte-

= 48 (1.2 +21..+ 1).

1..2 = -

!~.

Introduciendo en (7), vemos

"2= - ~~ no satisface

que A.t= - :~ satisface y


perim tri ca.
La extrema! buscada se determina

y= 7X-;_5X2

la condicin

;so-

por las ecuaciones


, }

z=x.
Hallar las extrernales en los siguientes problemas isoperirntr icos.
166. Entre todas las curvas planas de longitud 1 con
extremos en los puntos fijos Mo (xo Yo) y MI (xh Yl) hallar
la curva con ordenada mnima del centro de gravedad (problema sobre fa forma de equilibrio que toma un cable pesado
homogneo por accin de la gravedad).

J67. J{Y(X)l=.l

y'2dx;

o
1

condicin

J' y dx = 3.
o

y(O)= 1,

y(I)=6;

con

la

rt.

CAP.

116

EXTREMO

DE FUNCIONALES

168.

J [y (X) I=

J (X2+y'2)dx;

y(O)=O,

con

y(l)=Oj

o
1

la condicin

io

y2 dx = 2_
I

169. J [y (x)) =

Iy''l.

Y (O) = 0,

dx;

Y (l) = '4; con la

o
t

condicin ~ (y - y'2) dx = l~ ,
71
20. Tambin es un problema vartactonal de extremo condicionado
el problema de Lagrange en el Que se pide hallar el extremo de la
funcional J [YI' Y'i" ... , Yn] con la particularidad
de que se imponen
ciertas condiciones de enlace a las funciones de las cuales depende
la luncional J,
El problema se plantea as. Hallar el extremo de la luncional
XI

J F(x.

JI% Y'l., . , Ynl=

111. Y2, ... , Un, Y, y~, .. -, y~)dx;

(8)

xo
YJ(XO)=Yio,

(j=1,2

yj(x1)=YJI

.....

rt);

con las condiciones


, Un)

=O

1,2, ... , m; m

< n)

<p, (x, Yl.


(i
que se consideran

Y2.

(9)

independientes,

Las funciones !JI. Y2' .. Yn que realizan el extremo


de la funcional (8) con las condiciones (9) satisfacen, siempre que los
factores A.l (x) (i = 1, 2, . . . m) se escojan debidamente, las ecuaciones
de E uler de la funcional
TEOREMA.

In

1[ + ~

Xi

J. =

>'"0

"j<Pi

J x

' ... 1
In

Para abreviar

pondremos F+

2: X<p=ct>(x,

Yt.

y'}., ,. -,

Yn' Y,

i-1

y. . .. , y~). Entonces

las funciones

Xl (x) e y (x) se determinan

de

EXTREMO

9.

CONDIC10NAUO

117

las ecuaciones de Euler


ID' -~<ll',=o
IIJ
dx 'Ji

(j=I,2,

... ,n)

'f de las ecuaciones


q>i (x, YI' 112, , y,.) = O
(1 = 1,2 . , " m).
Las ecuaciones !PI = O tambin se pueden considerar como ecuaciones
de Euler para la funcional J* si se acepta que los argumentos de esta
funcional son tanto las funciones !l1' !/2. ' ., Ij" como las funciones
A." (z), hl! (x], .. Am (x).
EJEMPLO 6. Hallar
la distancia
mnima entre los puntos
A (1, -1, O) Y B (2, 1, -1)
que pertenecen a la superficie 15x- 7g
2 - 22 = O.
SOLUCiN. Comosesaben la distancia entre dos puntos A (xo, !lo; 20)
y B (Xl' lit> 21) en la superficie q> (x, y, z) = O se determina por la
frmula

Xi

1=

J 'Vi

+y'2.+z'2.dx,

"O

donde y = y (x). %
Z (x),
Se pide. pues, hallar el mnimo de 1 con la condicin q> (x, Y. z) =
= O. En nuestro caso,
.lO
Formamos

1,

X:!

2.

la funcional

q> (x, y, z)

= 15x - 7y

+z -

22.

auxiliar

J. =

j lVI +y'2+z'2+),,(x)

(15x-7y+z-22)1

dx

y escribimos para sta las ecuaciones de Euler


). (x) (-7)--dd ( 't
y'
) =0.
X
V 1 +y'Z+%'2

d (

A.(X}.l--d-x

Z'

VI+y'2+z'Z

=0.

(lO)
( 11)

Resolvamos el sistema de ecuaciones (lO) y (11) empleando la condicin de enlace


15x - 7y
z - 22 = O,
(12)
Las funciones incgnitas y = y (x) y z = z (x) satisfacen las siguientes
condiciones de frontera:
ti (1) = -1,
y (2) = 1,
z (1) = O, z (2) = -1.
(13)
Multiplicando por 7 la ecuacin (11) y agregndola a (10), obtenemos

_:!_ (
dx

y' -1- 7z'


) _ O
tI'~+ z'~ - ,

VI+-

CAP. 11. EXTREMO

118

DE FUNCIONALES

de donde

y'+7z'

( 14)

De (12) tenemos

z' = 7y' - 15.

(15)

Introduciendo este valor de 2' en (14) y resolviendo la ecuacin diferencial obtenida, encontramos 11 = C1X
Cz Las condiciones de
frontera (13) dan
1 = 2 Y C~ = -3
de modo que
(16)
y = 2x - 3.
Teniendo en cuenta (16), de (15) resulta
z= 1 - x
(17)
(es obv io que la funcin (17) satisface las condiciones de frontera).
De (lO) o de (11) obtenemos /,. == O. La distancia buscada es

1=

J lfl+y'2+2:'Zdx=V6.
1

Este resultado se puede obtener


ciones geomtricas evidentes.

inmediatamente

de considera-

30, Lneas geodsicas. Sea

(18)

r (u, v)

la ecuacin vectorial de una superficie.


Se llama lnea geodsica la lnea de menor longitud que pertenece
a la superficie considerada y que une dos puntos rijos de la misma.
Las ecuaciones de las lneas geodsicas se pueden obtener como
las ecuaciones de Euler correspondientes
al problema v ar iacional
sobre 1 a distancia mnima en la superficie entre dos puntos rijos:
Toda lnea perteneciente a la superficie r = r (u. v) se puede
representar
por 1as ecuaciones paramtrcas

u = u (/), ti --=: ti (t).


La longitud de su parte comprendida entre los puntos correspondientes
a los valores lo Y t1 del parmetro t es igual a
t

J[u(l),

v(t)[=

VEU'2+2Fu'v'-I-Gv'2dt,

(19)

!(l

donde E, F y G son Jos coef lcien tes de la primera forma cuadrtica


de la superficie (lB), o sea,
E
Aqu

(orou' Tu'
a, )

((Ir

au'

Qr )

Tu '

G=

(.El:...
ou' ~).av

(a, b) es el producto escalar de los vectores a y b .

9.

t::XTREMO

CONDICIONADO

1I9

Para la funcional (19) el sistema de ecuaciones


la forma
EuU'3 +2Fuu'v'
Guv'~
d
2 (Eu' +Fv')
VEu'+2Fu't.l-;
Gv'2-dtVEu'Lt-2Fu't)'~Gv'l!.

de Euler tiene

Evu'LJ2Fvu'r./+Gvu'll
VEu''J.+2Fu'u'.+Gv'2

_~

=0
'

2(Fu'JGv')
dt VEu'?.+2Fu'v'+Gl.'2

=0.

EJEMT'LO 7. Entre todas las curvas que estn sobre una superficie
esfrica de rado R y que unen dos puntos fijos de la misma. hallar la
curva de longitud mnima (la curva ~codsica).
SOLUCION. Sean (jJ y O las coordenadas
del punto en la esfera
y sea q> -"c q> (e) la ecuacin de la CUTva pedida. Tenemos entonces

r (<p. e) = x (ep. e) 1 + y (ep. e) j

donde

x ,...,.R cos

<p sen S,

>

z = R cos
Por eso,
E

(If. fl) /l,

R sen {f' sen e.

= Rf. sen2 O; (J = (re, rij)


segn la frmula (19). tenemos
91

J[<p(6)1=R. ~

e.
= R.2,

(rlp' r~)

De aqu.

F = (re. rq) = O.

el

Vd6z+sen ijacp'=R
ll

eo

VI

+ sen 6q>'z(O)

d6.

60

El integrando no conttene
ecuacin de Euler ser

la funcin

incgnita

q (O) y, por eso, la

de modo que

VI + seoS 6q>'2 (9)


dedonde
<p' (e)

C1
sen6Vsen26-q

sen!a

'v

1---

CI
sen2 9
Integrando,

V (1-

Cn- q ctg2

e - V

q
senzO
Cid ~etgO)
(1 q ctg*

cn-

obtenemos

el etg e
+ C?,
I-q

q (6) = arccos .. r
y

o'

CAP.

120

1I

EXTREMO

DE FUNCIONALES

cp (e) = arccos (e ctg 9} + c2,

donde

De aqu

e ctg a= cos (q> (9)- e:!)

ctg

e = A cos cp (O) +8 sen ep (O),

(20)

donde
8
Multiplicando

por R sen

sene2

e .

ambos miembros

de (20). obtenemos

R cos B = ARcos ep sen e + B R sen <p sen e


o. pasando a las coordenadas cartesianas.
z = Ax + By.
Esta es la ecuacin de un plano que pasa por el centro de la esfera
y que corta su superficie segn un circulo mximo. Por consiguiente,
1 a lnea ms corta (lnea geodsica) es el arco del circulo mximo.
EJEMPLO 8. Demostrar
que para una superficie de revolucin
es constante en cada uno de los puntos de las geodsicas el producto
del radio del paralelo por el seno del ngulo entre la geodsica yel
meridiano (teorema de Clairaut).
SOLUCION. En coordenadas cilndricas la ecuacin de una superficie de revolucion tiene la forma
x = p cos ep,
y = p sen P,
z = f (p).
Determinemos los coeficientes E, F y O:
E = 1
(2,
F = O,
G = p2.
. Por eso, la dilerenclal ds de la longitud de arco en +a superficie de
revolucin tiene la forma

ds=

V p2+(1 + ,~.)pf2

dcp.

En la superficie de revolucin las lneas geodsicas sern extremales


de la funcional
11>1

) V p'+(1 +f~9)p'Zdcp.
qJo

Como la funcin integrando no contiene explcitamente


lnrnedia tarnen te
p~
const,
p2+(1 +fp'} pfi

p. obtenemos

10.

PROBLEMAS

o sea, p9 ~~ =const.

CON

FRONTERAS

Observando que p :;

121

M6vll_ES

=senw

(fig.

18), obte-

nemos p sen o ee const que es lo que se queda demostrar.

170. Hallar

la distancia ms corta entre los puntos


1) en la superficie x
y -1- z = O.
171. Hallar las lneas geodsicas del cilindro circular

A (l. O. -1) Y B (O,-1,


r

R.

10. Problemas vartacionales


con fronteras mviles
1, Problema elemental con fronteras mvlles. Sea F = F (x, y, U') una
funcin diferenciable tres veces respecto a sus argumentos y sean
11 = q> (x)
e
U = IV (x)
(1)
donde
p (x) E CI fa, b] Y - '\ji (x) E
E CI la, b], dos curvas en el plano xOU.
Consideremos la funcional

J [U (x)} =

F (x, y, g') dx

(2)
fig. 18

'V

definida para las curvas suaves U = 11 (x) cuyos ex tremos A (xo, Yo)
y B (Xl. 111) se encuentran en las curvas (1) de modo que 110 = <p (xo)
e UI = W (XI)' Se pide hallar el extremo de la funcional (2).
TEOREMA. Supongamos que en la curva Yo : Y = (x) se alcanza
el extremo de la funcional

J (y (x)! =

F (x, y, y') dx

entre lodas la curvas de la clase el que unen dos puntos arbitrarios de


dos curvas fijas y = <p (x) e y = W (x). Entonces la curva 'Yo es una
extremal y en los extremos A (xo. Yo) Y B (Xl' Yl) de la curva 'Yo se cumplen
las condiciones de transversalidad
[F+(Ip'-t}FII,

1;0:_:0:0=0, }

(3)
[F+('IjJ'
Fy,J Ix~;q =0.
Es decir, para resolver el problema elemental con lronteras
mviles es preciso:
1) Escribir y resolver la ecuacin de Euler correspondiente.
Como resultado, se obtiene una Iamtlla de extremales !I = f (x, el. C3),
que depende de dos parmetros el y C3

-1)

122

C,\P.

2) Determinar
de transversalidad

11. E XTREMO

DE FUNCIONALES

las constantes CI, Cz Xo y


(3) 'j de las ecuaciones
{(xo,

e 10

{(XI,

C, C2,)=\II(Xt).

C2) = q (xo),

de las condiciones

Xl

(4)

3) Calcular el extr emo de la funcional (2).


EJEMPLOl. Hallar la condicin de transversaldad
cional

para la fun-

'q

Jf

J (y (x)1 =

(x. y) earctg 1/'

V I -1- g'~ dx,

(x, y)

-*

O.

XI>

SOLUCIN. Supongamos que el extremo de la izquierda de la extremal se ha fijado en el punto A (xo. Yo) mientras que el extremo de la
derecha B (Xl> YI) puede desplazarse por una curva y = lj1 (x). Tendremos entonces
[f "j- ('IjJ' - g') F 11,1Ix_xI = O.

En nuestro caso es
F

= f (x.

y) e~\rctg ,'

La condicin

(x,

y)

V 1 + y'Z

de transversalidad

earctg 1/'

F 1/' = l (x. y} (;'arc.tg 1/'

se representa

1 -+-y'

-~'-"-_

Vl+y'~

as

V 1-1- y'2 +
+W-y')/(x,

De aqu obtenemos,

y) eflrctg

debido a la condicin

'I>'-y'
I+1j'y'

1/'

I-i-Y', ]

VI +y ~

(x, y)

=0.

x=X

=1= O,

-l.

(5)

Desde el punto de vista geomtrico, Ia condicin (5) , lgnica que las


extremares y = y (x) deben cortar la curva y = ~ (x) por la cual
se desplaza el punto extremo B (XI> yd de modo que el ngulo entre
n

estas curvas sea de T'


Efectivamente, la relacin (5) se puede transformar del modo
siguiente: supongamos que la tangente a la extremal en el punto
8 (Xl. Yl). que pertenece a la curva y = 'IJl (x), Iorrna ngulo a. con
el Ox y que la tangente a la curva rija y = q (x) forma ngulo ~
(fig. 19); entonces, se tiene tg a. = y', tg ~ = '11>' y el primer miembro
de la Irrnula (5) da tg (~ - 0.); pero como -1
~ - a.
trar.

= - : ' de

donde a. = ~ +

~ que

tg ( -

: ), resulta

es lo Que se quera demos-

PROBLEMAS

lO.

EJEMPLO 2.

x-y = 6.

CON FRONTERAS

Hallar la distancia

SOLUCION. El problema consiste

MOVILES

de la parbola y

= x2

123
a la recta

en hallar el valor extremo de la

integral
:11:1

ry (x)) = J V 1 +y'Z dx
Xi)

con la condicin de que el extremo de la izquierda de la extrema! se


puede desplazar por la curva y = Xl mientras que el extremo de la

Fig. 19
derecha, por la recta y = x - 5. Por consiguiente, tenemos en este
caso <p (x) = X2 y ljl (x) = x - 5. La solucin general de la ecuacin
de Euler ser y = e1x
e2, donde el y el son constantes arbitrarias
que deben ser determinadas.
Las condiciones de transversalidad
(3) tienen la forma

donde y' = Cl' Las ecuaciones

(4) lienen en nuestro caso la forma

C1XO+e2=X~,
CtXt+Cz=xl-5.

CAP,

124

n.

I?XTREMO

DE FUNCIONALES

Tenemos, pues, un sistema de cuatro ecuaciones con cuatro incgnitas Cl C2 Xo Y Xl:

VI+C:+(2xo-Ct)

-.

el

=0,

C.

=0,

v I+C:

VI +C:+(i-Ct),

v l+q

1
j

CtXo+C2=X~,
CtXt+Cz=Xt-5;
resol vindolo, obtenemos
3

Ct=-l.

C2=4'

xO="2

Es decir, la ecuacin de la extrema! es y

= -x

23

x(=g'

!y

la distancia

de la parbola a la recta es igual a


2.3

T
l=

J V +<-I)ldX=V !:
1

23

=19y'2.

2X

2'

{72. Hallar la distancia ms corta del punto A (1, O)


a la elipse 4X2 + 9y'i = 36.
173. Hallar la distancia ms corta del punto A (-1. 5)
a la parbola y2 = x.
174. Hallar la distancia ms corta de la circunferencia
X2 + y2 = 1 a la recta x
y = 4.
175. Hallar la distancia ms corta del punto A (~1, 3)
a la recta y = 1 - 3x.
176. Demostrar que en el caso de la funcional

'1

Jfy(x)J=)

h(x,

y)V1+y'2dx,

'*

donde h (x, y)
O en los puntos frontera, las condiciones
de transversalidad tienen la forma

y' (x) = -

epll(X)

y' (x) = -

1j),I(X) ,

o sea, las condiciones de transversalldad se reducen a las


condiciones de ortogonalidad.

S JO.

PROBLEMAS

CON FRONTERAS

MOVILES

125

2, Problemas con fronteras mviles para funcionales de la forma

:el

[y (x).

z (x)] =

F (x, v. z, !J', z') dx.

(6)

:ro
Al analizar el extremo de la funcional (6) aceptamos que por lo
menos uno de los puntos frontera A (xo. Yo. %0) o B (Xl. Yl. 21) se
desplaza por una curva fija.
El extremo de J [y (x), z (x)1 se puede alcanzar slo en las curvas
integrales del sistema de ecuaciones de Euler
F....--F
dx

11

,=0,

Ft-~F.dx

:t

}
=0.

Supongamos que el punto A (xo. Yo, %0) est lijo mientras que el
otro punto frontera B (XI, !/l. 21) se. desplaza por una curva definida
mediante las ecuaciones
y=q>{x), }

z ='Ijl (x).
En este caso,

la condicin

de transuersalidad

tiene la forma

,+ (1jJ'-z') Fz.11:t_:t1=0.

[F+(q>' -!J') FII

Anlogamente se escribe la condicin de transversa1!dad para el


extremo de la izquierda (si ste tambin se desplaza por una

EJEMPLO 3. Hallar

la distancia ms corta del punto M (xo. Yo,

20)

a la recta
y=mx+p,

z =nx+q.
SOLUCIONo El problema se reduce a la determinacin

(mnimo)

del extremo

de la integral
~1

Ig (x), Z {x)J=

J VI

%0

+ g':l+z'2dx

(7)

CAP. 11. EXTaEMO

\26

con la condicin de que el extremo


desplazarse por l a recia

DE FUNCIONALES

de la derecha de la extremal

y=mx+p,

puede

(8)

z=nx+q.
o sea, en nuestro
la forma

rp

rp y IP tienen,

caso las funciones


(x) = mx

La solucin general
Euler ser

+p

'i1 (x)

del correspondiente

nx

respectivamente.

sistema

q.

de ecuaciones

de

y=G1X+G2, }
=G3X.+C~,

(9)

donde e (i = l. 2. 3 Y 4) deben ser determinadas.


La condicin de transversalidad
(en el extremo
tiene la forma

[ VI +y'3+z'2+(m-y')

y'

VI+y't+z'~
+(n-z')

de donde,

Z'

VI +y'2+ z'Z

=0,

X-XI

puesto que y' = el y z' = Gil' obtenernos


I

La relacin

de la derecha)

(10) expresa

-1-

mC}

/lC:!

la condicin

(lO)

O.

de perpendicularidad

la
recta buscada (9) y la recta dada (8).
Empleemos el hecho de que la recta buscada (9) pasa por el punto
M (xo. Yo, zo):
(11)
y tambin el hecho de que e 1 extremo de la derecha se desplaza por
la recta (8):

elXl+e2=mXl+P.
C3Xl

+C,=nxd-q

entre

(12)

De las cinco ecuaciones (I, (11) Y (12) debemos determinar CJ. C.,
Ca. e, y Xl (.to, Yo. 2o. m. n, p y q son nmeros dados). Para calcular
la integral (7) basta conocer Xl. el y C,. Tenemos

x _ ;_;xo::..,+..:......_m_{:;y,::,o
---=p,-!-,},....:.+_n"...(.:....~~o
-~q)
l+nZ+mZ

1-

C _ mxo+m/l
t-

e _
3 -

(2'0-9)-(1+n2) (yo-p)
m (yo-p) +n (zo-q) -(m2+n2) Xo t
rlXo+mn(Yo-p)-(l+mZ)(zo-q)
m (Yo-p)+n(2o-q)-(m2+nZ}Xo

P~OBLEMAS CON FRONTERAS MOVIlES

10.

127

Introduciendo estos valores en (7), obtenemos


h = mn J (y (x), z (x)l =

-V/'
x +(y -p)z+(z
.o
o
2

_ q)2_ rxo+m(!lo-p)+n{zo-Q)2
1 + n2 + m2

Si el punto frontera A (xo, Yo,

est fijo y el otro punto lronter a

20)

B ('~1' YI' Zl) pue-de desplazarse por


condiciones de lransoersalidod sern
(F -11' Fu,+(q~-Z')

una

superficie

Z ~..::.<p (x, y),

F~,.lLI:;;xl =_0, }

+ F~'CfllJ 1:x_xl -

IF!J'

las

(13)

O.

Las condiciones (13) conjuntamente


con la ecuacin 2 = <p (x, y)
permiten determinar, hablando en trminos generales, dos constantes
arbitrarias de la solucin general del sistema de ecuaciones de Euler
(las otras dos constantes se determinan de la cond icin de que la
extremal ha de pasar por el punto fijo A (xo, Yo, 20'
Si el punto mvil es el punto frontera A (xo, Yo, 20), obtenemos
para x ;-:. Xo unas condiciones anlogas completamente a las condiciones (13).
EJEMPLO 4. Hallar la distancia ms corta del punto A (1, 1, 1)
a la superficie esfrica
X2

+ + =
y2

Z2

SOLUCION.El problema consiste


lunclonal

J.

en analizar

(14)

el extremo de la

J[y(.~),

z(x)J=

IV

1+!I'2+z'2dx.

(15)

XI

donde el punto B (XI> fll. 21) debe estar en la superficie esrtca (14).
Las ex tremales de la funcional (15) son las recias
y=Ctx+C2,
}
(16)
z =C3x+C(o.
De la condicin de que la extr emal (16) pase por el punto A (1,.1, 1),
obtenemos
(17}

tienen la lorma

128

CAP.

n.

EXT~EMO

DE FUNI,;(ONALES

teniendo en cuenta (16), despus de unos clculos sencillos encontramos de aqu


ZI-CaXt=O,
CtZt-C3I1t=O.

}
(18)

donde Xl> 111 'i Zl son las coordenadas del punto buscado B.
De la condicin de que la extremal (16) pasa por el punto
B {Xl' 1/1' %1) tenemos
Yl = CX+Cz,
ZI =C3X+C4

}
(19)

De (17), (18) Y (19) encontramos


el = 1, C,,= O. C3=
de modo que la ecuacin de la extremal
y=x.

es

(20)

z=x.

Puesto que el punto 8 (XI. 111. Zl) debe estar en la superficie esfrica
(14), obtenemos, tomando en consideracin (20). que x~ + x~
xl =

va .
1

1, o sea. que XI =
Por consiguiente, obtenemos dos puntos

(_1

B
I

_J

_1)

V3' V3' ~3

B (

1 )

-V'j' -'V'3' -V3 .

Es fcil ver, por razones geomtricas, que en la extremal (20) que


une los puntos A y Bl la funcional (15) alcanza su mnimo igual a
1

Jm1n=

J VI+I+ldx=V3-1
1

Va
mientras que en la extremal
cional alcanza su mximo

(20)

que une los puntos A

y B~

esta fun-

Jmx=

\
1

lf3dx=

V3+ l.

- 113
OBSERVACiN l. Al deducir las condiciones de transversalidad (18)
hemos considerado que <p (x, g) == VI - x2 - 1/2. Es fcil ver que
las condiciones (18) subsisten si q> (x, y) == - VI _ x'l _ y2.
OBSERVACIN 2. Queda claro, por razones geomtricas,
que la
extremal (20) es ortogonal a la superficie esfrica x2 + y'J + i" = 1.

PROBLEMAS

10.

CON

FRONTERAS

MVI LES

129

EJEMPLO 5. Consideremos

de la funcional
O (O, O, O).

el mismo problema sobre el extremo


(15) pero tomando como A el centro de la esfera

SOLUCION. Las ex remales de la funcional son las rectas (16) y la


condicin de que la extrernal pase por el punto O (O, O, O) da inmediatamente C~ = C, = O.
Las condiciones de transversalidad
sern las mismas
Zl-C3Xl=O,

(21)

CiZt-C3!/[ =0,
y las condiciones

en el extremo

mvil sern

Jlt =CIXt,
z1

(22)

=CaXl'

Por ltimo,
(23)
Para determinar las cinco magnitudes Cl C" Xl. Yl Y z. tenemos
cinco relaciones (21), (22) Y (23) de las cuales solo tres son indepenclientes:
(24)

Empleando

las relaciones

(24), encontramos

l
x1=1(1+c:+q'

Y=Y'I+q+q'
1

zl

= -::i'~~:::::::::~

y I-I-Cl+q'

donde Cl y Ca son unas constantes arbitrarias.


Consideraciones geomtricas aclaran esta arbitrariedad: la distancia del punto O (O, O. O) a la superficie esfrica (14) es la misma en
cualquier direccin, o sea, para cualesquiera valores de Cl y Cs.
El valor de la funcional J [y (x), z (x)] en las extremales

!/=Ctx.
z=Csx

es igual a

Vl+chci
J !y (x),Z (x)l=
9-01381

~
'O

VI

+ct+qdx= l.

130

CAP.
EJ.EMPLO 6.

cional

11. EXTRflMO

OS

f1UNCJONALE.S

Hallar la condicin de transversalidad para la un-

Jf
%1

J[V(x).

z(x)J=

(x. 11. z)"VI

+y'2.+z'2dx,

(25)

si el punto A (xo. !lo, to) est fijo y el punto B (Xl. !JI, Zt) se encuentra
en la superficie z = q> (x, y).
SOLUCIN. En este caso las condiciones de transversalidad sern
(1+q>~z/)I'-'~1==O,
}
q>f2')I;a:..Z1=0,

(y'

o sea,
I

y'

q>; ,.... 1 = q>v

I :r.=o""1=-=1
~ 1-XI'

la condicin de paralelismo del vector T {l. y', z'}


tangente en el punto B (x .. (1' %1) a la extremal buscada y del vector
11 {q>;, <p~, -1 } de la norma a la superficie z = q> (x, 11) en este mismo
punto. Por consiguiente. para las funcionales de la forma (25) las
condccnes de transversalidad se reducen a las condiciones de ortogoh'alidad.
Representan

177. Demostrar que, si la condicin de transversalidad


coinciden para todos los
datos iniciales, la funcin integrando F tiene la estructura
siguiente:
F = (x, Y, z) VI y'2+Z'2,
y la condicin de ortogonalidad

donde

x,

y y

(x.

y, z) es una funcin diferenclable cualquiera de

z.

t78. Hallar
a la superficie
t79. Hallar
a la superficie
180. Hallar

la distancia ms corta del punto M (0,0, 3)


z = Xl + y'.
la distancia ms corta del punto M (2, 0, 5)
z = XZ
yz.
la distancia ms corta entre las superficies

~+ r; +~= 1

~.
J

x2+UZ+z2=4.

181. Analizar el extremo de la funcional


J fy (x)J Z (x)l =

(y'2+z':&+2yz) dx

S 10.

PROBLEMAS CON PRoNtERAs

si y (O) = z (O) = O Y el punto B


el plano

(Xl.

MO\lI LES

Yl.

21)

131

se desplaza por

Xl'

3, Distancia geodsica.

El valor de la integral
B

[y (x))

= .~F (x,

(26)

y, y') dx,

ca1culada segn una linea


desde el punto A hasta el punto B, se
denomina !-longitud de la nea y. Si y es una extremal se dice que
J [[1 (x)] es la distancia geodsica entre los puntos A y B, o simplemente
Jdistancla, y la propia extrernal se demomina J-recta.
EJEMPLO 7. Hallar la distancia geodsica del punto A (O, O) al
punto B (1, 1) si esta distancia se define mediante la funcional
B

J [y (x)) =

g2,/,'J. dx,

SOLUCION. La distancia geodsica del punto A al punto B es Igual


al valor de esta funcional en la extremar que une dichos puntos. La
ecuacin de Euler es

2yy'Z -

:x

(2YZy') =0

yyo" +y'2= O.

Es Hici1 ver que


l/JI"

d
+ {J'! = '7lX
(yy'),

de modo que 2yy' = CI e ya = C1x


C:a. Empleando las condiciones
de frontera y I:r=o = O e !l1:':=1 = 1, obtenemos el = 1 Y C. = O.
Por consiguiente, la extremal que une los puntos A y B es la parbola

ya = x.
Tenemos ahora 2yy'

1, yy'

~lI. por consiguiente,

(yy')2

!.

Por definicin, la distancia geodsica entre los puntos A y B es igual a


1-

1 (A. B)=

J ! !.
dx=

Supongamos que se tiene una lnea Z: q> (x, y) = O.


La distancia geodsica entre un punto B que no pertenece a Z
y esta linea se define como la distancia geodsica del punto B a un
punto A E Z que se obtiene calculando la funcional (26) segn la
extremal 'i que une los puntos B y A con la particularidad de que y
corta transversalmente
la lnea
en el punto A.

:c

9*

DE FUNlONAL2S

CAP. 11. EXTREM

Se denomina -circunferencia (o circunferencia geodtsica) la lnea


formada por los puntos que estn a una misma distancia geodsica
de un punto rijo. Anlogamente se definen los conceptos de -elipse
y de J-biprbola,
EJEMPLO 8. Hallar
la circunferencia
de radio R y con centro
en el punto O (O, O) si la distancia geodsica se define mediante la
funcional
B

J [U (x)]

y2/1'2 dx,

A.

SOLUCION. Las extremales de la funcional cortan transversalmente la clrcunlerenclegecdsca.


Para las extremales tenemos (vase
el ejemplo anterior)

y.

= el".

2yy'

= el

y. por consiguiente.

fJ'_
y
" _ 2:c

De la condicin

de transversalidad

y'y' (2ep' - yr) = O


encontramos

Que el coeficiente angular de la tangente

ferencia es ep' =

U;

erenca es y' =

. de donde

a la circun

y, por eso, la ecuacin diferencial de la circunresulta la ecuacin de la -circunfe-

rencia: u' = Cx. Para determinar el valor de e observemos que el


punto (CS, C) est en la circunferencia geodsica y que la ecuacin de]
radIo geodsico (o sea, de la extremal) que pasa por dicho punto es

tf = ~ . De

aqu tenemos !lU'

=~

es
R=

j (lIy')2dx=
o

y, por lo tanto,

es

Jo

4~2 dx= ~

Es decir, C = 4R y la ecuacin de la circunferencia geodsica de radio R y con centro en el origen de coordenadas es ya = 4Rx.
EJEMPLO 9. Hallar la circunferencia
de radio R y con centro en
el runto O (O, O) si la distancia geodsica se define mediante la funciona
B

Jry(x)J=

J lfl+!l dx.
2

10.

PROBLEMAS

CON FRONTERAS

MVILES

133

Las extremales de la funcional son las rectas y =


De la condicin de que la extremal deba pasar por el
punto O (O, O) encontramos Ce = O de modo que y = C1x y, por consiguiente, y' = .!L
x
La condtcn de lransversalldad coincide en este caso con la
condicin de ortogonalidad y, por eso, el coelcente angular de la
tangente a J-circunferencia es -q>' = - :' . Por consiguiente, la
SOLUCION.

= CIx + C,.

= _..:...
De aqu resul ta
y
la ecuacin de la -circunferencia:
XII + y'l = Cll El punto
(C, O)
est en dicha circunferencia. La ecuacin del radio geodsico que pasa
por este punto es y = O de modo que y' = O y
ecuacin diferencial de la J-crcunenca es y'

e
R= ) dx=C.

o
Es decir, C = R y la ecuacin de la circunferencia geodsica buscada
de radio R es la ecuacin de la circunerencia corriente ;(2 + y2 = Rll.
OBSERVACION. Los conceptos introducidos permiten hablar de la
Geometra no eucldea con la diferencial de arco
ds = F (x, y, g') d:c.

VI +

Si F =
y'2, las J -rectas se convierten, como hemos vlsto, en
las rectas corrientes y nuestra Geometra se convierte en la eucldea

corriente.
Si F es una {uncin arbitraria, que s610 satlsace las condiciones
habituales de ser continua y derivable respecto a los tres argumentos,
la Geometra construida muy poco recuerda Ia corriente: no siempre
se puede trazar una J -recta por dos puntos y puede suceder que por
dos puntos pasen varias J -rectas y, por consiguiente, que la J -dlstancta
entre dos puntos no sea una funcin

unvoca de las coordenadas.

182. Hallar la distancia geodsica del punto A (O,O)


al punto B (1, 2) si esta distancia se define mediante la
funcional
J(y(x)J=)

(!f+ y'2.) dx.

183. Hallar la distancia geodsica del punto A (O, 1) al


punto B (1, 1)si esta distancia se define mediante la funcional
J [y (x)) = ) (l2xy+ y'2) dx,
184. Hallar la J-circunferencia de radio R = 8 y con
centro en el punto O (O, O) si la distancia geodsica se define

134

CAP. 11. EXTREMO DE. FUNCIONALES

mediante

la funcional

J [y (x)] =

J y'3

dx.

t 1. Problemas discontinuos. Variaciones unilaterales


La extrema! y=y

1. Problemas discontinuos.

(x) de la funcional

:<1

lb' (x)l =

(x, y. !I)

(1)

dx

Xo

es una funcin que tiene dos derivadas continuas siempre que la derivada F V'II' (x, y (x), y' (x sea diferente de cero. Sin embargo, existen
problemas variacionales en los cuales el extremo se alcanza en una
curva suave a trozos solamente.
o) PROBLEMAS DISCONTINUOS
DE PRIMERA ESPECIE. Consideremos
el problema sobre la determinacin del extremo de la funcional (1)
aceptando que las curvas admisibles satisfacen las condiciones de
frontera
(2)
y (xo) = Yo, y (Xl) = Yl

<

el punto de abscisa e (xo < e Xl)'


Este punto angular puede darse 5610 all donde F 11'11' = O (vase el
teorema 2 de la pg. 53). En el punto angular la extrema] debe satisfacer las condiciones de Weierstrass - Erdmann
y pueden tener un punto angular en

FlI,I:l:=c-o-FII,I:l:=c+o=O,
(F - ti' FII,)

/x ... c-o-(F-

y' FV'} 1:1:""'4)+0=0.

(3)

Conjuntamente con las condiciones de continuidad de la extremal


buscada, estas condiciones permiten determinar las coordenadas del
punto angular.
En cada uno de los segmentos [xo, el y [e, Xl! la extremal debe
satisfacer la ecuacin de Euler, o sea, una ecuacin diferencial de
segundo orden. Al resolver estas dos ecuaciones se obtienen cuatro
constantes arbtrarias que, hablando en trminos generales, se determinan de las condiciones de Irontera (2) y de las condiciones (3) en el
punto angular.
EJEMPLO J. Hallar las extremales quebradas (si es que existen)
de la funcional
a

J Iv(x)J=

J (y'2_
11

y2)

dx,

n.

P~OBLEMAS

DISCONTINUOS

1.35

SOLUCION. Escribimos la primera de las condiciones (3) que deben


cumplirse en el punto angular:
F 11'

I:.:"",c-o = Fu' b:=e+o

(O <:: e < a).

En nuestro caso tiene la forma

y' (e - O) = y' {e O)
Y significa Que la derivada y' (x) es continua en x

= c. Por conslguenle. no hay puntos angulares. Esto se puede ver tambin de que en
nuestro caso F 1/'V' = 2 > O en todo punto. Por lo tanto. en el problema considerado ~~ extremo puede alcanzarse slo en curvas suaves.
EJEMPLO 2. Hallar
las extremaJes quebradas de la funcional
2

Jly(x}j=

S (y'4_6y'2}dx;

y(O)=O.

y(2)=0;

aceptando que y' puede Ser discontinua en el punto correspondiente


a la abscisa x = c.
SOLUCIONoEn este caso F11''''' = 12y'2- 12 se puede anular, y, por
eso, puede ocurrir Que la extremal tenga puntos angulares. Puesto
que la Iuncn integrando depende s610 de y', las extrernales son las
rectas
Pongamos

-< <

-<

y_ = mx + 11 (O X
e) e U. = p
q (e x ~ 2).
De las condiciones de frontera encontramos n = O y q = -2p
modo que
y_ = mx e 11+ = P (x - 2).
La condicin de continuidad de ta extremal da
me
Escribamos

las condiciones

=p

(4)

(e - 2).

de Welerstrass r",,=4y'S-12y'

de

Erdmann.

Tenemos

F-y'FII,= -3y"+6y'2.
Puesto que y:" =

m e y:

= p,

obtenemos

4m3-12m=4p3-12p,

-3m'+6m2= -3p4+6p2,
o sea.
(m-p) (m2+mp+p2-3)=O,

(m2-p2) (m2+p2-2~=O.

(5)

CAP. 11. EXTREMO

136

DE FUNCIONALES

m = p. m = -p o

La segunda de las ecuaciones (5) da inmediatamente

m"

+ pZ

-2

= O.

La solucin m = p debe ser excluida: en este caso la extrema! tiene


derivada continua y de la condicin (4) obtenemos m = O, o sea, la
extremal es un segmento de) eje Ox.
Por consiguiente. para resolver el sistema (5) hay que resolver
dos sistemas de ecuaciones:

m=-p. }

6)

~+~+~=3

y
m2+p2=2, }
m2+mp+p2=3.

(1)

113

La solucin del sistema (6) es: m=V3, p=y In = - V3,


p = V3. La solucin de! sistema (7) es m = p y debe ser excluida.
Es decir, m::: -p y la condicin de continuidad (4) da e ,= 1.
Por consiguiente, las extrernales buscadas son:

V3x,

y= {

-]3 (x-2),

e
g=

-vax,

V3 (x -

O~x<
2).

1 '" x

1,

< 2.

185. Hallar las extremales con punto angular para la


funcional
2

J (y (x)] =

1o y'2

(y' - 1)2 dx;

y (O) = O, y (2) = 1.

186. Hallar la solucin c.on un punto angular en el problema sobre el mnimo de la funcional
4

J(y(x)]

(y'

_1)2. (y'

+ 1)2 dx;

(O) = O,

(4) = 2.

187. Existen soluciones con puntos angulares en el


problema sobre el extremo de la funcional
Xl

J [y (x) 1 =

J (51'2+ 2xyxg

y2) dx;

y (xo) = Yo,

Y (Xl) = Yi?

PROBLEMAS

~ 11.

DISCONTINUOS

137

188. Hallar la solucin con punto angular en el problema


sobre el extremo de la funcional
I

J{Y(X)I=)

y(-I)=.O,

y2(l_y'2)dx;

y(I)=l.

-1

189. Hallar la solucin con punto angular en el problema


sobre el mnimo de la funcional

;le.

(y" - 2y'2) dx.

%1

190. En el problema sobre el extremo de la funcional

(""r
J

1/1)

sen y' dx

(o. O)

hallar la solucin continua y la solucin con punto angular.


OBSERVACION. Las condiciones
(3) de Weierslrass-Erdmann
admiten la siguiente interpretacn geomtrica.
Consideremos la fguratriz o sea. la curva Y = F (x, y, y') en
tanto que funcin de y'.
Las condiciones (3) signHican entonces que para los valores de los
parmetros x = e y y = el, que corresponden al punto angular, la
figuralriz debe tener una misma tangente en los puntos de abscisas
y:" = y' (e - O) e y.j. = !J' (e
O).
Al mismo tiempo se obtiene una Interpretacin clara de la condcin FU'I/' =60 que excluye la posibilidad de puntos angulares en las
extrernales. Efectivamente. si, por ejemplo, es F I/'Y' > O, la fguratriz
es cncava hada las Y positivas y no podrn coincidir sus tangentes
trazadas en dos puntos distintos. Es decir, en este caso la extremal
no podr tener punto angular.
Consideremos de nuevo el problema sobre la determinacin de las
ex trema les quebradas de la funcional del ejemplo 2 de este pargrafo.
Tenemos

J!U (x)]

(y'"_6y'2)

dx;

U(O)=O,

y(2)=O.

O
-

Las extremales son rectas. En este caso la Iiguratr iz y = y'. del punto (x, y). Tiene una tangente comn en

GIf'2 no depende

138

CAP. JI. EXTREMO DE FUNCIONALES

los puntos de abscisas y'


de Weierstrass-Erdmann

= V3 (fig.
quedarn

20). Por eso, las condiciones


cumplidas si como extrernales

Fig. 20
quebradas

~ con

tomamos las quebradas

cuyos lados formen ngulos de

el eje Ox.

En la quebrada !11 con un punto angular (Hg. 21) la funcional


toma el valor J {ud = -18. Este mismo valor tendr Jiu (x}) en la
y

x
Fig. 21
quebrada Y'J con dos puntos angulares,
con tres puntos (lig. 23), etc.

(Hg. 22), en la quebrada Ya

b) PROBLEMASDISCONTINUOS DE SBGUNDA eSPECIB. Se denominan


problemas discontinuos de segunda especie los problemas sobre el extre-

PROBLEMAS

S 11.

DlSCONTI NUOS

139

mo de la funcional
XI

1 [(x)

F(x, y, y')dx;

y (X2)=Y2;

Y (X)=Yf.

(8)

;ll:f

en la que la funci6n Integrando es discontinua.


Supongamos, por ejemplo. que F [x, Y. y') es discontinua a lo
largo de la curva y = <!> (x) y sea F (x, y, g') igual a F1 (x, y, g')
a un lado de la linea y = dl (x) e Igual a F, (x, y, y') al otro lado.
y

Fig. 22

SI existe la extremal quebrada, deb.er componerse de los trozos


de las extremares y = Yl (X) e !I = y, (x) que tienen un punto comn
(c. <!> (e, e E (Xl. XI)' en la linea de discontinuidad.
Para hallar la
y

Flg. 23
extrema! quebrada obtenemos dos ecuaciones diferenciales de Euler
cuyas soluciones generales contienen cuatro constantes arbitrarias Cl
Cg C, y C,. Para determinar estas constantes as como la abscisa e
del punto en el que la extremal encuentra la curva y = CD(x) tenemos:
1) dos condiciones de fronteras (8), 2) dos condiciones segn las cuales
las ordenadas de los extremos de las ex trernales en' el punto de [uncin
han de ser iguales a la ordenada de la curva.y = CD(x) y, por ltimo,
3) la condicin de la un.ci6n
..

Fi +(q.'

-!/) Fi'

IlC"C-O""".F2.+(<D' - y') !'21!' 1~=e+O..

{9~

CAP< 11. EXTREMO

140

DE FUNCIONALES

Hablando
determinar

en trminos generales, estas condiciones alcanzan para


la extremal quebrada.
EJEMPLO 3 (problema de la refraccin de un rayo de luz). La luz
se propaga con una velocidad constante Ell en el medio 1 y con una
velocidad constante VII en el medio 11. La curva y = (l) (x) separa los
medios 1 y II.
Deducir la ley. de refraccl6n del rayo de luz que va desde el punto
A del medio' 1 hasta el punto B del medIo 11 si se sabe que es mnmo
el tiempo durante el cual el rayo de luz recorre este camino.
SOLUCIONo El .problema consiste en hallar el mnimo de la inetgral
J [y (x)=

r Vf+Y'2

dx+ Jb

01

v-:::Y;Z

(l

dx,

(10)

V2

ya que la primera y la segunda integrales de (10) representan, respectivamente, el tiempo que necesita el rayo de luz para llegar del punto A
a la linea de separacin y de la linea de separacin al punto B.
Tenemos un problenia discontinuo de segunda especie siendo

Para determinar los trozos de las extremales debemos hallar las extremales de la funcional

JV

1 +1I'zdx

que, como se sabe, son rectas. Por consiguiente

Y1=mx+n
Escribamos la condicin
F

aF 1
1-91 a9~

iJF2
iJy,

F'J.-yz-,
Introduciendo

U2=P;<+q.

(9). Tenemos

11-::::fiii
UI.

gi'
t1tVI+Y'

(11

Vl+u'

(12

Y 1+ !li' .

estas expresiones en (9), encontramos

I
VI

+ <Il'Y

V 1+lI'

1+ ID'u
02

Vl+!l"

(11)

Sea y el ngulo que forma con el eje Ol' la tangente a la lInea de separacin en el punto de abscisa e, sea a el ngulo que forma con el eje
O el rayo de la izquierda y sea fl el ngulo que forma con el eje Ox
el rayo de la derecha. Entonces, se tiene ~' = tg y, Yi = tg a, =

y,

n,

P~08LEMAS

141

DISoNTINUOS

= tg ~ y la condicin (11) toma la forma


1
Vi

+ tg

tg i'

VI + tgZ

[1%

+ tg ~ tg i'

V 1+ tg2~

cos (y-ex)

tJz
donde 'V - ( y 'V - ~ son los ngulos entre los rayos y la tangente
a la llnea de separacin. Tomando en lugar de stos los ngulos lp y e
entre la normal a la lnea de separacin y los rayos, incidente y refractado, obtenemos
sen <p
tll
--=-=consl,
(ti

sen

ClZ

o sea, la tey de refraccin del rayo de luz.


20. Varlac:lones unilaterales.
funcional

Se pide hallar el extremo de la

Xli

J [y (x)) =

F (x, y, g') dx;

con la condicin
(12)
O ( y - lp (x) ~ O)
(las condiciones de limitacin pueden ser de forma ms compleja).
En este caso la extrernal buscada puede estar formada por trozos
de extrernales que pertenecen al recinto (12) y por trozos de la frentey -

q> (x) ~

ra y = q> (x) de este recinto. En tos puntos de j uncin de estos trozos


la extremal buscada puede ser suave y tambin puede tener puntos
angulares.
La condicin en el punto de [uncin tiene la lorrna
[F(x,!J,

y'l-F(x,

y, tp')_(q/_!J')

Fil' (.x, y, y')J1

-=0.

~-x

_Si_FIJ'JJ' ::;60, la extremal es tangente en el punto de [uncln


y) a la frontera y = q> (x) del recinto.
EJEMPLO 4. Hallar en el recinto y ~ X2 el camino ms corto del
punto A (-2, 3) al punto B (2, 3).
SOLUCION. El problema consiste en hallar el extremo de la Iunconal

M (x,

J [y (x) I=

jV

+y' '1. d

-2

con las condiciones

!I ~ xi.

y (-2) = 3,

y (2) = 3.

(13)

i42

CAP. 11. extREMO DE FuNCIONALES

Las extremales

de la funcional (13) son las rectas

En nuestro

= el + Csx.

caso

F".".=

1I

'4= O

(1 + y'2)2

Y la extrernal buscada se compone de los trozos AM y NB de las rectas


tangentes a la parbola y = xt y del trozo MON de esta parbola
(Hg. 24). Representemos las abscisas de los puntos de tangencia por
y
(utilizamos la simetra del problema). En el punto de tangencia

-x

A (-2,J)

Fig. 24
colnclden las ordenadas y los coeficientes angulares de la recta y de la
tangente a la parbola de modo se tiene

el+e~="i: }

(14)

ez=2x

Por otra parte, la tangente


consiguiente,

debe pasar por el punto B (2, 3) y, por

(15)
CI se, = 3.
Eliminando CI y C, de (14) y (15), encontramos
4x
3 = 0,
de donde Xl = 1 y i. = 3. El segundo valor de X no sirve. Es decir,
i = 1 Y el = -1 Y C. = 2. La extremal buscada (nica) es

x' -

-2X-
II=-

x?

2x-l

Queda claro que la funcional

si
si
si

-2<:.%<-1.
-1

,x<

l.

l,x<2.

(13) alcanza en ella su mnimo.

TEORtA

HAMI L tON ..J ACOBl

DE

143

191. Hallar las curvas en las cuales puede alcanzarse el


extremo de la funcional
10

J[y(X)]=)

y'3dx;

y(O)=O,

y(IO)=O;

o
si las curvas admisibles no pueden pasar por el interior del
crculo que limita la circunferencia (x - 5)2 + y2 = 9.
192. Entre las curvas que unen los puntos A (a, Yo)
y B (b, y) hallar la curva que ofrece el valor extremo a la
funcional
b

J [y (x) = y VI-y2y'2dx
(1

con las condiciones y~O, l-!ly'2~O.


12. Teora de Hamiltoo-Jacobi. Principios variado
nales de la Mecnica
1. Forma cannica (hamlltoniana)
de las ecuaciones de fulero
Las ecuaciones de Euler para la funcional
J [YIt Y2. ""

Unl =
=

F (x, Yt. !h....

, !/n.

!/i,

y;, .... y~) dx

(1)

Xi

tienen la forma
d

Fu --d
k

F ,=0
I/It

(k= 1,2 ...

, n).

(2)

En el caso en el que el determinante


FJI'I/'
F 1/''''
, F 1/''''
11
la
in
FII,,,,F
al

FII'IJ'

11'11'

in

tI!

pondremos
F , = PIt
IIk

(k

(3)

1, 2, . . n).

De las ecuaciones (4) se puede expresar


. , Un' PI, P'A
, Pn:

gt = <Pk (x, !l1. Y

,*0

Jlk

(4)

en trminos de x, JI1, JI"

" !In' Ph Ps

Pn)'

i44

CAP. JI. EXTREMO DE FUNCIONALES

La (uncin H de las variables


definida mediante la igualdad
H=[-F(x.

1It, 11"}.."0'

Un.

x, Yl. y"

!/. y;, ....

' .

o, Yn' Pi, P2. , . Pro

U,I)+

11

+ ~ YP
~1

(x, !lb Y2,

o oo,

y,.. y;'. y~.. , Y~)]l ,

~~

se denomIna hamiltonlana de la funcional (1).


El hamiltoniano satisface las relaciones sigu lentes
aH
dyl!.
01{
dp",
{)Ph=--X'
iJy",=-dX
(k=1,2, ... ,n).

(5)

Se dice que las ecuaciones (5) son el sistema cannico O hamiltoniano


de las ecuaciones de Euler (2); las varIables 111. !h, ... Yn' P, PI . , .
. . . Pn llevan el nombre de variables cannicas,
OBSERVACiN 1. La condicin (3) en el caso de la funcional
OCa

J (U (x)l=

J F (x,

y, y'

FII'II,4: O en [XI. x2)'

x da

:I:

OBSERVAcrN 2. Hablando en trminos generales. las ecuaciones


(4) no se pueden resolver unvocamente respecto a Yi en todo el segmento IX1> !Cal. Si se cumplen las condiciones del teorema de existencia
de la (uncin Imptcita , las ecuaciones (4) admiten solucin unvoca
localmente.
EJEMPLO l. Formar el sistema cannico de las ecuaciones de Euler
para Ia funcional

J
.1t

J [Ut. Y21=

(2Y1Y2-2U~

+y? _Y~Z)

dx.

SOLUCiN.

En nuestro caso

F (x,

!/lt

Y2'

!/. Y2) = 2ylY'J. - 2yj

Ponemos
F , = PI

lit

F,

1/2

Entonces
P1 = 2y';

+ !/? - u'.

= PI'

Aqu el determinante
F V'l/'
F l/'V'
11
1 2
FI/''J' FI/'V'
2 1
a 2

Resolviendo

respecto a y~ e

Y2 Ias relaciones obtenidas. encontramos

' - Pi
!J1-2

y'

P2
'=-2'

TEORfA

12.

DE HAMILTON-JACOBI

Formamos el hamlltoniano
H = (-F

+Yi.Fr.+yF

145

de la funcional considerada

y.)

a,

Pi

"1-2'
II,=_PI

2.

=(-2YIY2+Zgl+y;,a-Yit)

Empleando las relaciones


ecuaciones de Euler

dYt
dx

1'1

111"'2'
II'__ P.!
2
2

(5), obtenemos el sistema cannico de las

=.!!1. .

(.[i= -411t

Incgnitas de x,

d!l2=_~.

2'

dPt

Aqu Y1 = III (x), y,

p.
pi
=2Yf-2I1Y2+-;f---f,

dx

2 '

.
+2Y2; dP2_211
7iX- t

= Y:I(x],

Pl = PI (x) y PI

}
.

= Pa (x) son funciones

EJEMPLO 2. Formar el sistema cannico de las ecuaciones de Euler


para la funcional

J
SOLUCiN.

rUi. 92)=

YIY (x2+ lIi +lI) dx,

Aqu

+ u + gil

F = yfy (x2
Determinamos

las derivadas parciales


F 11'

= U~yl,

= !I~YI

F y'

Pz=y~y:

Ponemos
Pl=y~yl

Estas relaciones no comprenden las derivadas lIi e Y: de las funciones


Incgnitas lI1 e Ya; por eso, no se puede expresar y' e Y en trminos
de PI y PI' Por consiguIente, no se puede formar e\ hamiltoniano de
esta funcional. En este ejemplo no se cumple la condicin (3):

I 1

EJEMPLO S. Formar

Euler

_1 OO O01 =- O

F "iY F "li \
FI/'v' Fy'Y' -

el sistema

cannico

para la funcional
J [y (x) I =

10-01381

xgy'3dx.

de las ecuaciones

de

146

CAP. 11. EXTREMO

FUNCIONALES

De

SOLUCiN. Tenemos
F

Pongamos

xyy'3

Fu'

3xyy"J.

de donde

3xlIy'2,

J/ ~:

y' = -

11'=

3~g

La uncional considerada tene dos hamll tonianos

,,-- Vp-=
~

Hi=(-F+yIF""j.

=2Xyy/31
2

H,_=(-F+y'F1/'

lp=3V3

1/- .

"Ji

En concordancia con esto obtenemos


ecuaciones de Euler:

dI!
dx

..

I"pa

dp

p3 ,
xy

dos sistemas cannicos de las

/p

1 ..

=-~3y3

xy'

= JI 3xy'

/7

dX"="3 V

P
V3x"V

1/'__ '

3xy3'

1
l

Formar los sistema cannicos de las ecuaciones de Euler


para las funcionales siguientes:

J xyV y' dx,


194. J [y (x)J = J xyy'2 dx.
193. J [y (x)} =

195. J [y

n = J

t 96. J [111. Y2}:;::

+ y Vl + x':>' dx,
J (y? + y: +y~2)dx,
~ X2

S 12,

'l'EOIHA

147

HAMILTONJACOBI

J (xz+Yty;l+y,.y~~)dx.
J lYlt Y~J = J (2xYt - y~2+-} "!I~a) dx,

197. J[Uft

198.

bE

Yzl=

2, Ecuacin de HamUton-Jacobl.
Teorema de Jacobl. El
sistema cannico (5) de las ecuaciones de Euler es el sistema de ecuaciones de Euler para la funcional
J (Ui, !/a, ., " !/n] =

r[~

(x, Uit 112' . , !In' Pi' Pa, .. , Pn)] dx

Pk!/~ -H

~, 1-1

si 1/1- Y... , Yn' p" PI' . " Pn se consideran en tanto que funciones incgnitas de x.
Esta funelonal J es la solucin de la ecuacin en derivadas parcales de primer orden

aw

( x, Ut,

~+H

ox

aw iJW
iJW )
Uz, . , Un' -iJ -iJ , . -, -iJ
=0,
Yn

112

!Jt

que se denomina ecuacin t Hamilton-Jacobi.


TeO~EMA DE JACOBI

Supongamos qu.e W es la integral completa


y satisface la condicin

de la ecuacin de Hamllton-Jacobi
2W

ec,
a2w

OYt

Q2W
iJYlOCa

--,

Ol-W

iJ~W
OY.

oc;

a2w

oY2OC. iJYaiJC2 - .. iJY2iJCn

cJ2w
aYn

se t

()2W

aYn

:::/= O.

cJ2w

ec; ... aYn iJCn

Entonces las igualdades

aw

iJW
OCA =8k,

oy/, =p/,

(k= 1, 2, .. _, n),

dJJnde Ch. y Bk son unas constantes arbttrarlas, determinan una solucin


tUl sistema cannico (5) lpendiente tU 2" constan/es arbitrarias.
EJEMPLO 4.

Hallar

las extremales de la funcional

J [y (x)] =

lit) V X2 + !l V 1 + y'2

dx

:tI

utilizando la solucin de la ecuacin de Hamilton-J

acob.

JO

CAP. 11. EXTREMO

148

DE FUNCIONALES

SOLUCION. Para obtener la ecuacin de Hamilton-J


acobi ormarnos el hamiltoniano
de la funcional considerada. Tenemos

H=

-lf x2+yZ_pZ.

La ecuacin de Hamilton-Jacobi

aw
--

I./:
r'

ex

tiene la forma

(OW)2
-

XZ+y2_

ay

=0

OX
+ (dW)2
Tu = x2+y2.
( OW)2.

(6)

Representemos la ecuacin (6) en la forma

(~~
r

-x2

+( ~~)2 -

y2 =0

Y apliquemos el mtodo de separacin de variables.


la ecuacin (6) se ver illca si se exige que

) 2. _
( aw
ox
donde

e es

x2

=_C

una constante arbitrarla.

aw
ox =Vx2-C

(aw)
ay

2_

Queda claro que

y2=C
I

De aqu encontramos

aW =lfyZ+c.

oy

La integral completa de la ecuacin (6) ser

W=

J Vx2-C

dx+ V y2+Cdy=

=+xlf.t2-C

- ~

In)x+ 1fx2-C 1+; y V y2+C+

+;
donde C y Co son constantes
De la relacin
determinamos

4Vx2-C

~~

=+

In

ls+ yy2+cl+co1

arbitrarias.
I

donde

A es una

constante arbitrara,

la solucin general de la ecuacin de Euler, Tenemos

___!_ x+,VXCCI+ E.
2

+TV;+c+i-

(x+lfxz-C)

In!y+Yy2+cl+

lfx2-c

TEORIA

~ 12.

Despus de unas simplificaciones

In

IY+Vyq=c

x+Vx2-C

149

DE HAMILTON-JACOBI

I=A.

sencillas, obtenemos

o sea,

u+Vyq:c

=A

X+Vx2-C

(A= eA.),
de donde resulta

es decir,

definitivamente

una familia

de hiprbolas.

Hallar las extrernales de las funcionales siguientes:


~2

199. J [y (x)) =

j xy -vy' dx,
e

200. JIY(X)]=JxyyI2dx;

y(l)=O,

y(e)=l.

201. J [y (x) =

a (y)

V 1 +y

'2dx.

202. Hallar el mnimo de la funcional


1

J ({-

y'2+ yy' +s' + y) dx


o
si se desconocen los valores en los extremos del segmento.
203. Hallar la funcin del campo p (x, y) y el propio
campo de extremales que pasan por el origen de coordenadas
para la funcional
J [y (x)] =

(~.II)V--

:Y'Z-dx

J[y(x)l=)

(y>O).

(O, O)

204. Entre las lneas que unen el punto x = O con el


punto Nf (Xl, Yl), donde Xl> O e Yl> 0, hallar la lnea

en la que alcanza su mnimo la funcional

CAP. 11. EXT"REMO DE FUNCIONALES

150

XI

J [y (X)]

= ) vr::y'2 dx

Supongamos

(y> O).

11

que se tiene la funcional


:rll

J [y (X)[

F (x, y, g/) dx

Xi

y = q> (x, C). Entonces en todo


punto del campo se conoce la direccin de la transversal del campo que
pasa por este punto. Todas las transversales del campo se obtienen
como las soluciones de la ecuacin diferencial de primer orden
y se conoce su campo de extremales

F!J'

rx, 'P (x,

C), q>~ (x, C)] :~ = H (x,

(jl

(x, C), q>~ (x, C)),

donde en lugar del parmetro e, que determina las ex tremales del


campo, hay que introducir su expresin en trminos de las coordenadas
de los puntos de) campo. Aqul H (x, y. p) es el hamiltonano.
EJEMPLO 6. Hallar las transversales para el campo de extremales
y = ex de la funcional

J [y (x)]

y'2 dx,

:I:{

SOLUCION. Formamos el hamiltonano

de la funcional considerada.

Tenemos
F

= y'2

Poniendo p=FlI"

F 11'

2g'

(F 11'11,2

=1= O).

encontramos y'=r~ y

H=(-y'2+2y'y')

= P: .

II'-~

Las transversales

se obtienen resolviendo la ecuacin diferencial

F 11'
donde en lugar de
coordenadas

2C

que tomar su expresin en trminos de las

de los puntos del campo: C = Jf_.


JI:

2y'
Puesto que

II/-cx :~ = H 1,,-211' _

e hay

Iv-cx :~ = ~ /P_2C

Tenemos

2C ~~ = C2.

e ~ o, se tiene 2 :; =C, o sea, 2!!

= ~. De aqu

encontramos <jue la familia de transversales son les parbolas 1J2~

e",

12.

TEORIA

DE

HAM!LTON-JACOBI

151

205. Hallar las transversales del campo de extremales


ex de la funcional

y =

~2

J F (y/)

J [y (x)j =

dx,

206. Hallar las transversales del campo de extrema les


x
e de la funcional

y =

J (xy't. -

X2

J (y (x) I=

2yy'3) dx.

XI

207. Hallar las transversal es del campo de extrernales


x2
y=x-c
de 'la funcional
"'2

J W(x)] =

J Vy(I-y'2)dx

(C>O, x>O,

y~O).

Conocen do 1a ecu acin de H amil ton -J aco b i

aw
(
ax + H x,

y.

W'
) =o
ay

de la funcional
:x:z

fu (x)J =

F (x, y, ;y') dx,

=1

se puede reconstruir

la funcin integrando

cin de la ecuacin diferencial

F (.r. !J. y'). Esta es solude primer orden

F - zF; = -H (,(, y, F~,


(7)
donde H (.r, IJ, p) es el hamiltoniano
de la funcional considerada
y F (x, !I, z) es 1a f uncin inc6gni ta (se considera que x e y son parmetros). Despus de determinar F (x, y, z) hay que tomar en ella la
derivada y' en lugar de z,
OBSERVAC.ION. La ecuacin (7) es la ecuacin de ClairauL Como
regla, la solucin general de la ecuacin de Clairau t se omite pues
en este caso la funcin integrando F (x, y, y') es lineal en y' y el
problema varacional no siempre tiene solucin (vase el 4). Por
eso, se toma slo la solucin ingular de la ecuacin de Claraut que
ser precisamente la funcin buscada F (x, y, z).
EJEMPLO 6. La ecuacin de Hamilton-J
acobi en el problema
Xa

sobre el extremo

de la Iunclonal

1 [y (x)l = ) F (x, y, !J')


:lI4

t4 tene

CAP.

152

11, EXTREMO

DI!

FUNCIONALES

la forma

aW)2 + (aW)2
Tu
( ox

= ,~2+y2.

Hallar la funcin F (x, y, y').


SOLUCION, R.esolviendo la ecuacin dada respecto a la derivada

aw
ox '

tenemos

aw '.V/
-ax=

x2+V2-

(aW)2
iJy

o sea,

Por constguien le, el hamiltoniano

es

H= -Vx2+y2_p2.
La ecuacon (1) para la determinacin
forma
F-z-=
dF
dz

Ji"

de la luncln F tiene la

;c2+y2__(dF)2
dz

(8)

Derivando respecto a z ambos miembros de la ecuacin (8), resulta

dZF

Dejando a un lado el caso dz2 =0 (que da la solucin general, tenemos


dF

dz

z =-,~
/====::;:dF;::::::;02

Resolviendo

,\,2+V2-

(dZ)

esta relacIn respecto a la derivada


dF

_zV~

Tz- Vl+t2 .

::

' encontramos
(9)

12.

TEORrA

DE

HIIMILTON

..JACOBI

153

Introduciendo (9) en (8), obtenemos


F=z

vii+Y2
+..V xa+ y
VI+:z2

V:c2+y:qll

2_ 22 (x2+y2)

Por consiguiente,

I+z~

la funcin integrando

+z:1.

buscado tiene la forma

F=Vx2+y2 Vl+y'2.
En los problemas que siguen hallar las funcionales a partir

de sus ecuaciones de Hamiltoo-J acobi:

209. 4

aw
aw _ . .2
(Ix oy - ,{,.-y

2JO. 4xy
21 J.

..2.

aw (aW)2
Tx+
a

aW)2
( X-ax

= O.

+( y aW)2
ay

=x2 +y2.

30. Principios variaclonales de la Mecnica.


a) PRINCIP(O DE HAMILTON - OST'ROORADSI<I.Supongamos que se
tiene un sistema de n puntos materiales Mil. (XII' 1111.. 2'1I.) (k = 1, 2, ,." n)
con masas respectivas mk (k = 1, 2, .. " n). Supongamos que el
movimiento del sistema est sometido a enlaces
(jlJ (x, y, z, t) = O
(J = 1, 2, ... , m;
m ~ n)
(10)
y se realza bajo la accin dejas fuerzas PII. (X,." Yk' ZJ) (k = 1, 2, oo., n)
que tienen el potencial (funci6n de Iuerz a) U = U (x.\. Yk, zk. t):

su

Xh=;;-,
IJxk

eo

Y,,=:;--,

..yft

eo ..

ZJI.=~

"'k

La energa cintica de este sistema ser igual a

Supongamos que este sistema pasa de cierto estado A correspondiente


al momento del tiempo t = to a otro estado B correspondiente al
momento de tiempo t = tI' Entre todos los desplazamientos posibles
del sistema de A a B se escoje la clase de movimientos admisibles que
concuerdan con los enlaces dados y que hacen pasar el sistema del
estado A al estado B en el Intervalo de tiempo dado (io, '1)'
El principio de Hamilton-Ostrogradski consiste en lo siguiente:

entre todo los moulmienlO&admisIbles que hacensasar el sistema

54

CAP.

11. EXT~EMO

DE FUNCIONALES

del estado A al estado B, el movimiento real se caracteriza por el


cumplimiento de la condicln necesaria fjJ = O de ex/remo de la
funcional
ti

J (T-I-U)dt.

J=

(11)

to
A cada movimiento admisible del sistema le corresponden 3n
funciones Jek (1), !lk (/), %/1 (1} (k = 1, 2, ... , n) que estn definidas
en el intervalo [to, ',], que satislacen las ecuaciones (lO) 'i que toman
determinados valores en los extremos del Intervalo [to, t11. Por consiguiente, tenemos un problema variacional con los enlaces (lO) y con
fronteras fijas.
Para resolver este problema formamos la funcin auxiliar de
Lagrange
m

F=T+U

+ J AJ (t) 'Pi
;=1

y escribimos para ella el sistema de ecuaciones de Euler-Ostrogradski:

mkXk-XB

u'PJ

m
~

J..j

(t) axl>. =0,

j ... 1

u'PJ

mkYIt-Yk-

~ AJ(t) U!lk=0,

(12)

j-t

mllzli.-ZIt-

O'PJ

~ AJ(t)

OZh

=0.

;=1
El sistema (12) conlclde con las ecuaciones diferenciales del movlmlento real del sistema.
b) PRINCIPIO DE 1.A ACCIN ~MINIMA EN LA FORMA DE LAGRANOB.
Supongamos que Jos enlaces ''Pi y el potencial U no dependen del
tiempo t. En este caso tiene' lugar la integral de energa T - U
= h = consto La integral

/1

J Tdt
lo

se denqmina I!c~i(l{l.De la integral


IJ

(11) se deduce que

,~

ti

(T+U)4t=~

(Tdt-)

li

h dt,

12.

TEOR fA

DE

HAMI L TONJ ACOBl

155

El principio de la accin mnima en la forma de Lagrange consiste


en lo siguiente: para el mooimiento real. la integral de la accin debe
tomar su valor mnimo. O sea.
It

J=

J Tdt=min.
to

El principio de la accin mnima puede ser representado en la


forma de Jacobl
~ V2(U+II)

ds=mn

'\1

es la diferencial del arco y) en la que no interviene el tiempo.


OBSERVACION l. Aqui se consideran admisibles los movimientos
que satisfacen las ecuaciones de enlace ~J (x. Y. z) = O (} = 1, 2. m}
y la ecuacin T - U = h con el mismo valor de h que para el movimiento real y que tienen los estados inicial y final rijos. siendo tambin fijo el momento Inlclal lo del tiempo. El momento final del tiempo
no se fija para estos movimientos.
OBSERVACIN 2. La energa potencial figura no en la integral sino
en la condicin complementaria T - U = h. Formamos la funcin
auxiliar de Lagrange
(ds

F =2' T+2' (U+h)+ LJ "'JfPJ'


;-1
Despus escribimos las ecuaciones de Euler-c-Ostrogradski
problema

mkxk

iJU

mhYlI.=-a

..
mhzll.

= ;--+2
vXh.

au

para nuestro

a~

'" Xl -rt
.t:.J
vX)t
;-1
m

iJCFl

-+2 .a
~ "J;-,
v!lk

YIt

iJU
OZIl

;-1
m

+2 ~

OCPJ
"J iJzll

j ...
t

que representan

las ecuaciones del movimiento real.


Basndose en el principio de la accin mnima, hallar
la trayectoria del punto material (de masa unitaria) que se mueve
por accln de la gravedad.
SOLUCIN. Tomando ~l eje Oy hacia arriba. el potencial de la fuerza de la gravedad es
EJEMPLO 7.

(l3)

156

CAP,

11. EXTREMO

Segn el principio
cada 'Yo la integral

DE FUNCIONALES

de la accin mnima,

para la trayectoria

J=SV2(U+4)dS

bus(14)

debe alcanzar su valor mnimo. Por consiguiente, la trayectoria ser


una extremal de la funcional (14). Introduciendo (13) en (14), obtene-

mos

Xi

J=

S V2(h-gY)Vl+y'2dx.

La ecuacin de Hamilton-Jacobi

-ax- JI
c)W

-. /'

2h-2gy-

o sea,

(~~) 2+ (~~

(15)

tiene la forma
(

aw ) 2. =0,
ay

=2 (h-gy).

Su integra l completa es

W=Ax+

Jr

l/2h-2gy-A2

1-

dy=Ax- 3g (2h-2gy-A2)

donde A y B son constantes arbitrarias.


Determinamos las extremales de la funcional

+B,

(15):

X+~(2h-2g!l_A2)2
g

=C,

o sea,

y= ; - :; - 2~2 (x-C)2;

y e

constantes.

En particular, las exremales que pasan por el origen de coordenadas se determinan de la condicin y (O)=0. Obtenemos una Iarnilia rnonoparamtr Ica de parbolas

s=>- 2A2 x2+

:t:2ii=Ai
A
x.

212. Hallar en el plano la trayectoria de un punto que


se mueve por efecto de una fuerza repulsiva que acta desde
el eje Ox en direccin del eje Oy y que es proporcional a la
distancia del punto al eje Ox aceptando que la integral de
la fuerza viva tiene la forma ~ - ~s = O Y basndose en

12.

i1~ORtA

DE

HAMIL 'tONJACOS)

i 57

la integral de la accin

J y VI +y'2dx
X2

J [y (x)] =

(y> O).

213. Un punto material describe la circunferencia p =


q> son las coordenadas polares) de radio R
bajo la accin de una fuerza central
inversamente proporcional a la quinta potencia de la distancia al centro que se
encuentra en el origen de coordenadas. Demostrar que la
integral de la accin alcanza mnimo fuerte en cualquier
arco de esta circunferencia (- ~ < cri ~ q.l ~ (f1\J < ~).
214. Analizar el movimiento de un punto material por
efecto de una fuerza central de atraccin proporcional a la
distancia al centro O basndose en el principio de la accin
mnima y aplicando el mtodo de Hamlton-J acob.
=

2R cos cr (p,

:6

Capitulo JI
M:TODOS DIRECTOS EN EL CLCULO VARIACIONAL

13. Mtodo de diferencias finitas de Euler


Consideremos el problema variacional
mo de la funcional

elemental:

hallar el extre-

J [y (x)] = ) F (x, y. y') dx

!J (a)=A,

y(b)=B,

(1)

Segn
no en las
sino en las
rectilneos

el mtodo de Euler, los valores de la funcional (1) se toman


curvas arbitrarias que admite este problema variacional,
quebradas compuestas por un nmero dado n de segmentos
cuyos vrtices tienen abscisas fijas

a+6x,

a+26x, , .. ,

a+ (rt-I) Sx,

b-a
ax = --.
rt

donde

En estas quebradas la funcional J [y (x)l se convierte en una Iuncln


(Yi, Y2, ... , YV-l) de las ordenadas g., y" ... , Yn-I de los vrtices
de la quebrada. Las ordenadas Yl, Y2 " Yn-l se escogen de modo
que la funcin I> (Yl, !/i, ... , Yn-l) tenga extremo, o sea, se deterrninan del sistema de ecuaciones

(IJ

oI>

iJy? =0, ... ,

~-o
oYn-l

La quebrada as obtenida es la solucin aproximada del problema


varacional (1).
EJEMPLO. Hallar la solucin aproximada del problema sobre el
mnimo de la funcional
1

J{y(x)l=

J (y'2+2y)dx

y(O)=y(I)=O.

o
SOLUCION.

Yo

1-0

Tornemos x = -5Y (O) = O,

Ya = Y (0,6),

!JI = y (0,2),
Yl = Y (0,8),

0,2 Y pongamos
Y2 = y (0,4),
Y5 = Y (1) = O.

MeTODO

Sustituimos

b~

DIFERENCIAS

159

FINITAS

los valores de la dervada segn la frmula aproximada


(x ) '"'" YIt+t -YIt
Yk' -y'
k""
6%

Entonces
lit-O
11' (0)---oy-,

y' (O2),

Y2-1I1

0,2

la integral

por una suma empleando

) f (x)

dx ~ [f (a)

'13;;;112 ,

O-Y411,'(0 8) =~.

Y ' (0,0.1)= 1I~~Y3,


,

Sustituimos
rectngulos:

y' (0,4)=

'

la rmula

de los

+t (.~1)+ f (x2) + ...+ f (xn.-t)J 6x.

(l

Tendremos
(l)

(Yt, 112,Ya, 11",)


=[

t.~) + (
2

Y~~

Y1 ) 2

+ 2Y1+

+ ( !/3-;:./1.)2 +2Y2+ ( Y";;1I3)2 +2Y3+ ( _

,J

t.~)2 +2y

.0,2.

Formamos el sistema de ecuaciones para determinar las ordenadas


1111 g .. Ya e y!" de los vrtices de la quebrada buscada;
1

ofI)

111

0,2 oYt = 0,02 --

112-Yt

Y3-Y2 +2=0

a<D =

Y3-Y2

Y4-YS +2=0,

_1_ am
0,2

_1_
0,2

oY'l.
aYa

0,02

0,02

0,02

I aa>
y",-Ys
0,2 oy4. = 0,02

o sea,

117.- lit
0,02 +2=0,

0,02

114
0,02 +2=0,

2Yt-Y2=_O,04,}

-yd-2yz-ys=
-Y2+2Ya-YI,,=

-Y3+2Y4

-0,04,
-0,04,

= -0.04.

La solucin de este sistema es lit = -0,08, 112 = -0,12, y, = -O, J 2


e Y4 = -0,08. Estos valores de la solucin aproximada coinciden
con los valores que tiene en los puntos respectivos la solucin exacta

x2-x

y= -2-'

CAP. J 11. MTODOS OIRttCTOS

160

Hallar las soluciones aproximadas de los problemas sobre


el mnimo de las funcionales:
1

215. JIy(X)1=)

(y'2+y2+2xy)dx;
o
SUGERENCIA. Tomar 8..1:=0,2.

y(O)=y(I)=O.

216

.1 [y (x)] =

J (y'2+

1) dx;

o
!J (O) = O,
b) Y (O) = O,

a)

y (1) = O;
y(1)=

l.

14. Mtodo de Rtz. Mtodo de Kantorvch


10. Mtodo de Rltz. La idea del mtodo consiste en que al hallar
el extremo de la funcional J [y (x)) se consideran, en lugar del espacio
de las funciones admisibles, s610 fas funciones que se pueden representar como cornb lnaciones lineales de las funciones admisibles:
n

Yn (x) =

IX,IPI (x).

(1)

j ... 1

!x,

donde
son unas constantes y el sistema {<Pi (x)}. llamado sistema
de [unciones coordenadas, est formado por funciones lp, (x) que son
linealmente independientes y que constituyen un sistema completo
de funciones en el espacio considerado.
Hablando en trminos generales. cuando pedimos que las funciones Yn (x) sean admisibles, Imponemos a las funciones coordenadas
<PI (x) ciertas condiciones complementarias como, por ejemplo, llrnltacienes en cuanto a la derivabilidad o en cuanto a la verificacin de
las condiciones de Irontera.
En estas combinaciones lineales la funcional J fu (x) se conv erte
en una funci6n de los argumentos !Xl' IX2, .. , Ctn:
J [gn (x)l = <D (Ct., IX2..
, IXn)
Determinamos los valores !Xl. IX" , IXn que ofrecen extremo 8 la
funcin <D (!Xl' !X2, . , !Xn); para ello resolvemos el sistema de ecuaciones

o<D

~=O
UIX

(i= 1,2, ... , n),

no lineales, como regla. respecto a !Xh a2, . , ct,u e introducimos


en (1) los valores encontrados para !X,. La sucesin {Yn (x)} que asl
resulta es una sucesin minimlzante, o sea, la sucesin de los valores
de la luncional {J [Yn (xH) obtenida El partir de elJa converge hacia

J 14.

MSTODO DE RITZ. MSTODO DE KANTOROVICII

el minimo o hacia la cota inferior de la funcional J


de
lim J {UIl (x)]

n ... oo

no se deduce an que

t!)t

fu (x)l. Sin embargo,

= min J 1.9 (x)J

11m Yn (x) = U (x). La sucesin minlmtzente

n ... oo

puede no converger hacia la funci6n que realiza el extremo en la


clase de las funciones admisibles.
Se pueden indicar las condiciones que garanticen que el mnimo
absoluto de la Iunconal exista y se alcance en las funciones (l/n (x)}.
En el caso en el que se trata del extremo de la funcional
%1

J (y (x)i =

JF

(x, 11. y') dx;

%1

11 (XI) = Yit

V (x2) = 112:
estas condiciones son:
1) la funcin F (x, y, t) es continua respecto al conjunto de sus
argumentos para cualquier z y para (x, y) E D. donde D es un recinto
cerrado del plano xOy al que pertenecen las lineas Un (x);
2) existen unas constantes ex > O. p > I Y ~ tales que

F (x. y, z) ~ I z IP

+p

cualquiera que sea z y para cualquier punto (x, y) E D;


. 3) la funci6n
F (x, !I, z} tiene la derivada parcial continua
Fz (x. y, z) y esta derivada es una funcin no decreciente
de
Z (00 < z < +(0)
cualquiera que sea el punto (x, y) E D.
En particular.
las condiciones enunciadas se cumplen para las
funcionales

J (y (x))

(p (x) y'2+Q (x) y2+2r (x)

yl

dx;

~I

if.(xl)=a,

y (x2)=b;

donde p (x), q (x) y r (x) son funciones dadas, continuas en [XI> xsI.
con la partlcularidad
de que existe la derivada continua p' (x) de
p (x) y de que p (x) > O Y q (x) ~ O.
Si por este mtodo se determina el extremo absoluto de la funcional, el valor aproximado de su mfnimo se obtiene por exceso y el
valor aproximado de su mximo, por defecto. Al aplicar este mtodo,
el xito depende en gran medida de la eleccin adecuada del sistema
{Q?t (x)} de funciones coordenadas.
En muchos casos basta tomar la combinacin lineal de dos o tres
funciones 'l>J (x) para obtener una aproximacin bastante satisfactoria
de la solucln exacta.
.
Si hay que determinar el extremo aproximado de la funcional
J (z (Xl' xs. _ ., xn)) que dependen de las funciones de varias variaH-01381

CAP, 111. MeTODOS DIRECTOS

162
bies independientes.

se escoge un sistema de funciones coordenadas


11'1(Xl. Xa

Cj), (Xl'

.ta, .. ". Xn) ..

y la solucin aproximada
forma

, xn),

, Cj)m (XI.

Xli'

, Xn),

del problema variacional

...

se busca en la

m
Zm(Xh

X2, , Xn)= ~ akll'k(x" Xz..


11",,1

, Xn).

donde los coeficientes all son unos nmeros constantes. Para determinarlos se orrna, por analoga con lo que hemos explicado, el sistema
de ecuaciones

!(1) = O (k = 1, 2,

VaA

... , a), donde Cll (ah a,l, ... , a n)

es el resultado de introducir 2m en la funcional J (z (Xl. Xli' . "' x~)I.


EJEMPLO
1. Hallar la solucin aproximada
del problema sobre
el mnlmo de la funcional
.
t

J (y (xH=

(y'2_ y2+ 2xy) dx;

(2)

Y (O) = g (1)

Y compararla
SOLUCiN.

==

O;

con la solucin exacta.


Como sistema de funciones coordenadas Cj),\ (x) tomamos
epA (x) = (J - x) x1l. (k = 1, 2, . " .).

Es. evidente que las funciones epk (x) satisfacen las condiciones de
frontera Cj)k (O) = Cj)1I (1) = 0, son linealmente independientes y forman un sistema completo en el espacio ello, 11.
Para k = 1 tenemos 111(x) = IX (x - x'). Introduciendo
esta
expresin de Yl (x) en la funcional (2), obtenemos
i

JllIt(x)J=

(a;f{I-2x)2+al{x-x2)2+2alX(X-x2)ldx=

o
1

J [al (1-4x+4x

2-

-,z+ 2z3-x4)

+ 2!li (x2-z3)]

dx=

11

3.1
"')
=lO
ai+"6l%l= ....(at.

Ei coelcente af se determina de la ecuacon


0Cll
OGt

=-sat

1
+"6=0.

ie ,

METODO DE RITZ. MarODO De KANTOROVICH

de donde resulta a1=

5
-18'

163

Por consiguiente,

Yt (x)
= --x+-x
2
18
18'
SOLUCION EXACTA.

La ecuacin de Euler de la funcional conside-

rada es

y'
Resolviendo

esta ecuacin
y =

x.

1'1=

lineal

no homognea,

encontramos

el cos x + Ca sen x + x,

Empleando las condiciones


definitivamente

de frontera y (O)

=y

(1)

= O,

obtenemos

sen x
y=x-se!'
Comparemos

las soluciones

y aproximada:

exacta

II

SolucIn exacta

0,00
0,25
0,50
0,75
1,00

Hallar

EJEMPLO2.

Sotucln

aprexlrnade

-0,044
-0,070
-0,060

-0,052
-0,069
-0,052

la solucin aproximada

de la ecuacin

no

lineal
3

1'1-=2'
que satisfaga

Ias condiciones

SOLUc,rON. A este problema

1'12

y (O) = 4, Y (1) = 1.
de contorno le corresponde el proble-

ma variacional
1

J IY(x)=

y (0)=4.

(y'2+y3) dx;

y (1) = 1-

Buscaremos

la solucin

en la forma

Y1 (x) = 4 -

3x

+ al

(x -

)(2);

es evidente que 1'11 (x) satisface las condiciones de frontera dadas cualquiera que sea el valor de ~.
Tenemos
j

J [JI. (x)l=

S (la. (t -2x)-3}2+
O

[4-3x+a. (x-x2)]3) dx,

CAP. 111. MerODOS

164

DIRECTOS

de donde
I

= ~{(1-2x)2[a.t(I--2x)-31+

oJ[y(x)}

aa.t

()

+3 (x-x2) [4-3x+CXl
La condicin

{x-x2)l2} dx.

(jJ [!JI (x)] =O toma la forma

oa.

9cx~
490CXl+ 1407 = O
obtenemos la solucin del problema
!JI (x) = 3,0413x16,0413x
4
posi Uva en lodos los puntos.
y para

O!.l

-3,0413

Hallar las soluciones aproximadas y compararlas con las


exactas en los problemas que siguen sobre el mnimo de las
funcionales:
1

217. J[y(x)l=

J (y'2+2y) y(O)=y(l)=O.
o
J + y2 + y'2) y
dx;

218. J [y (x)] =

(2xy

dx;

(O) = Y (2) = O.

o
2t 9. Hallar la solucin aproximada del problema sobre
el mnimo de la funcional
I

J(y(x)1=

J (y'2-k if')dx;

y(-l)=y(I)=O;

-1

con la condicin complementaria

J yZdx = l.
-1

EJEMPLO 3. Hallar la solucin aproximada


el extremo de la funcional

J [z (x, g)=

Jr Jr

Jz )
l ax
+ (ayaz )
e- (

del problema

2z dx

sobre

dy,

donde D es el cuadrado -a ~ x ~ a, -4 ~ Y ~
en la frontera del cuadrado.
SOLUCIN_
Buscamos la solucin aproximada
Zo (x, y)

= 0.0 (x2

aa)

VI' -

aa).

a. siendo z = O
en la forma

, 14.

METOOO

DE

RITZ.

M~TOOO

DE

165

KANTOROVICH

Es evidente que esta funcin lO (x, 1/) satisface las condiciones de


frontera planteadas. Introduciendo Zo (x. y). z" (x, y) y Zy (x, y) en
la funcional e integrando. obtenemos
J Izo (x, y))

11$

32

512

32

= ~5 o;~as-9ctoas=!ll (etol.

Tenemos despus
0(1)

---ct
oaooa
- 8_-a
45 6-O

5
de donde cto= 16az de modo que

ZO('~I y)=

-,
5
16a2 I.x2-02)

(1/2_02).

220. Hallar la solucin aproximada del problema sobre


el extremo de la funcional
J (z(x, y)J = )

J r(~: _y)2 +( ~~+xf J dxdy,

donde D es el recinto limitado por la elipse ~: -+ ~: = 1.


221. Hallar la solucin aproximada Z3 (x, y) del problema sobre el mnimo de la funcional

j J [( ~:) + (~;) J dx dy.


2

J [z (x, y)] =

donde D es el recinto: x> O, !J > O Y x


cin z (x, y) satisface en la frontera F: x
+ y = 1 la condicin z Ir = x'l + y'l.

+ !J < 1, si la fun=

O, y

Oy x

20. Mtodo de Kanterovleh. Este mtodo ocupa una posicin


intermedia entre la resolucin exacta y el mtodo de R itx y se aplica
para analizar el extremo de las luncionales
J (z (Xl. xa, .. , xn))
(3)

que dependen de funciones de varias variables independientes (11 ~ 2).


Igual que en el mtodo de R itz, escogemos un sistema {<Pk (XI. X2 ... XII)
de (unciones coordenadas y buscamos la solucin aproximada en la
forma
(4)
pero considerando los coeficientes a) (xJ) como Iunciones incgnitas
de una de las variables independientes.

CAP. 111. MerODOS

166

DIRECTOS

En las funciones (4) la funcional (3) se convierte en una funcional


a2 (xJ)'
, Clm (Xj)]
que depende
de m funciones
al (X/), a. (xJ)'
, G'tm (xJ)' Estas funciones se escogen de modo
que la funcional 1alcance el extremo y se determinan de las condiciones necesarias de extremo para la funcional 1.
Empleando el mtodo de Kantorovch, se obtiene una solucin
aproximada, como regla. ms exacta que la solucin que da el mtodo
de ~ Hz con las mismas funciones coordenadas 'P" (Xl, x:, ... x n)
y con el mismo nmero m de trminos en la aproximaci6n.
EJEMPLO 4. Hallar la solucin aproximada
de la ecuacin de
Poisson

1[t (xJ)'

~r=

si

-1

{-a~x-<a
-b -< Y -< b'

en el rectngulo D:

z = O en la frontera.

SOLUCiN. La ecuacin ~z = -1

es la ecuacin de Euler-Ostro

gradski para la funcional


J

[2 (x,

y}J =

j J [( ~: t + ( :; )

2-

2Z] dx dy.

(5)

Buscamos la solucin en la forma


%1

(x, y) = (h' -

y') a (x);

es evidente que la funcin Zl (x. y) satisface las condiciones de frontera


= O en las rectas y = +b.
Intrcducendo esta expresin de Zl en la funcional (5), encontramos

J [zt(x, g)l =

J (:~

b5a'2+

b3a2 ~ b3a) dx;

(6)

-o

La ecuacin de Euler para la funcional (6) es

,,5
Ct -

2bZ

Ct=

-W

(7)

La ecuacin (7) es una ecuacin lineal no homognea con coeficientes


constantes y su solucin general es

Las constantes Ct y Cz se determinan


Ct

(-a)-a

de las condiciones de frontera


(a)=0

16.

LOS

VALORES

lo queda C2=OyC,=

LAS

Ji

2ch

Es decir, obtenemos

2'1

(x , y) =

5 a
2 b

1-

ch

'y/52b 1
JC

(x. y)

(bt

,/5aj

2b

ch
t-

I
~2

167

de modo que

chy

b2_gZ
2

Para obtener una aproximacin


del problema en la forma

PROPIOS

--

l
a(x)- -

FUNCIONES

eh

..V/521) 1
Ji
x

5 a
-2 b

ms exacta se puede buscar la solucin


y'') a} (x)

+ (b' -

y') a, (x).

222. Hallar en el recinto D la solucin aproximada de


la ecuacin de Poisson 6z = -1 que se anule en su frontera
si D es el tringulo equiltero formado por las rectas y =

~3 x y x

b.
223. Hallar en el recinto D la solucin aproximada de
la ecuacin 6.z = -1 que se anule en su frontera si D es el
=

trapecio issceles formado por las rectas y = ~3 x, x =


y x= 3.
15. Mtodos variacionales para la determinacin
de 108 valores y de 188 funciones propios
La ecuacin de Sturm-Liouville
d
- dx (p (x) ti) +q (x) g = AY,
donde p (x)
condiciones

> O tiene

derivada continua

y q (x) es continua,

III
COl1

las

y (a) = O e 11 (b) = O
(2)
tiene la solucin nula (trivial) !I El O cualquiera que sea el valor real
o complejo de A.

CAP. 111. MeTODOS DI ~ECTOS

168

El conjunto de la ecuacin (1) y de las condiciones de frontera (2)


se denomina problema d contorno tU Slurm-Liouoille
(1)-(2).
Losvalores de'" para los cuales el problema de contorno (1)-(2)
tiene soluciones no triviales g == O se denominan va/ores propios
y las soluciones mismas llevan el nombre de [unciones propia, del
probleftlll

de contorno.

La ecuacin (1) es la ecuacin de Euler para el siguiente problema


sobre extremo condicionado:

hallar el mfnlmo de la funcional


b

J lu

(x)!

(3)

(PII'z+ qUa)dx

con las condiciones (2) y la condicin


b

J y2

(4)

dx= 1.

()

= Y (x) es una solucin de este problema variacional, tambin


ser una solucin del problema (1)-(2) distinta del cero Idntico en
virtud de la condicin (4). Por eso, los valores propios y las funciones
propias del problema de contorno de Sturm-Liouville
se denominan
tambin valores propios y unctcnes propias de la funcional (3) con las
condiciones (2) y (4).
La funcin propia 11 = Ij (x) se denomina normada si
Si IJ

EJEMPLO 1. Hallar

los valores propios y las funciones

propia,

de la funcional
3

J (y (~)J =

J 2x +

3)2 y'2 -

y2J

dx

con las condiciones

ti (O) = O,

Y (3)

O,

(5)

y2dx= 1.

O
SOLUCION. La ecuacin

de Sturm-Liouville
tiene la forma
d
-y-7iX
((2x+3)2tll ='J..y,

o sea,
(2x

+ 3)'y" + 4 (2x + 3) g' + (A + 1) Y = O.

(6)

Sil.

LOS VALORES

LAS PUNCIONES

PROPIOS

169

Mediante la sustitucin 2x
3 = tI la ecuacin (6) se reduce a la
ecuacin lineal con coeficientes constantes
d2y
dy
4 dt? +4dT+().+l)y=O.

(7)

Su ecuacin caracterfst ica


4k'
tiene las rafees

+ 4k + )..+ I
I

kt,2=-'22"Y
Consideremos tres casos.
1) A < O. Entonces la solucin

=O

'1r-

-A.

general de la ecuacin (7) es

y = Cle"tl + e.e"tf
donde k. y ka son nmeros reales; en consecuencia. la solucin genera)
de la ecuacin (6) es

y
las

condiciones

Cl (2x

de frontera

3)111

el

(2x

+ 3)"1.

(5) dan

Ct3"'+C23111=O. }
e19~t+C2~=O,
de donde el = O. Ct ==
2) A = O. Entonces

e y
'

5'

O.

y, por consiguiente,

De las condclones de frontera obtenemos


Ct+CJln3=O.
Cl +Caln 9=0,
de donde Ct=O y

e2=o,

o sea, ysO.
1

3) A. > O. Entonces ki
de la ecuacin (7) es

2=

y=e--r (CtCOS

V~ Y
-'2 i -2-

~I

t+C2sen

la solucin general

v:); t)

ClIP. 111. MeraDOS

170

Pasando

DIRECTOS

la variab le x, obtenemos

CIC<>Sr~

In (2x+3)]

+c2sen[ ~ln(2x+3}]

Y2x+3

Las condiciones de frontera t5) dan


e1cos

(v:

el cos

("-fIn 9) +Czsen (-2- In 9) =0. J

(~I

In3)+C2s~n

=0,)

In3)

V-

(9)

El sistema (9) tendr soluciones no triviales si su determinante


igual a cero
cos (

y-f

In 3) sen (

1(-T-

V~In9 ) sen (-2vX

cos (-2por
X

lo

tanto.

~:

sen (Y~ln3-

4nZn2
x, 1:::I'"i'ii2'3

(n=

In 3)
In 9

= O;

In3)=0,

V~ In J) =-_ O, de donde -2-ln


Vi 3=
(-2-

es

m'C,

es

decir,

sen x

Los valores propios sern

1, 2, ... ).

Tomando cualquier ecuacin del sistema (9), por ejemplo, la primera,


e introduciendo en ella /..n en lugar de A, obtenemos
Cl cos
o sea, el (_I}n
AII

4n2n2

= O.

/I.Jt

+ e,

de donde el

sen ns:

= O,

= O. Tomando

en (8) el

o y

= In23 ' obtenemos las funciones prop las del problema conside-

rado
sen

AA
[

In (2x+ 3) ]
In 3

Los coeftcentes en se determinan

de la condicin

) Y~l(x) dx= I
O

(n= 1,2, , .. ).
de normacin

LOS VALORES

IS.

Y LAS PUNCIONES

PROPIOS

17}

lo' que da

c.,. = :i:,Vr-21n3

y, por consiguiente,
2

Un (x)=

V13

sen [

M In (2X+3)J

1n3

V2x+3

(n= 1,2, ... ).

Hallar los valores propios y las funciones propias en los


problemas que siguen:
1

224.

J [y (x) =

1 (y2+

y'2) dx;

y (O)= y (1) = O;

J !ldx=).
o

225. J(y(x)J=

X2y'2dx;

y(1)=y(2)=O;

t
2

J y2dx=

1.

i
e

226. J{y(x)l=

J (6l+X2y'2}dx;
1

y(I)=y(e)=O;

) yadx=

1.

1
2n

227. J (y (x)] =

J (y2_y'2)dx;

y(1C)=y(2n)=O;

JI

2n

J !ldx=
:n

228.

J[y(x)l=

J 13!1-(x+

1)2y'1l)dx;

y(O)=y(I)=O;

J y dx=J.
2

Il

J.

CAP. 11. MaTO DOS 01 R flCTOS

172

Los valores pror!OS y las funciones propias del problema variaconal (3), (2) Y (4 tienen varias propiedades importantes.
1) Si)..,., y t..n son dos valores propios diferentes de la funcional (3)
con las cori(ficiones (2) y (4) Y si 11m (x) Y !/n (x) son las funciones propIas que les corresponden, estas funcIones 11m (.x) e /In (x) son ortogonales. o sea,
b

Ym

(x) IIn (x) dx=

(m

=1<

1\).

"'n

2) Todos los valores propios


de la funcional (3) son reales.
3) Si An es un valor propio de la funcional (3) e IIn (x) es la uncl6n propia normada que le corresponde, se tiene
J [Yn (x)]

= An'

4) El menor de los valores propios coincide con el mlnimo de la


funcional (3) con las condiciones (2) y (4).
BJEMPLO 2. Demostrar la desigualdad

J"o

y'?' (x)

>-

Jo

y (O)=y (n)=U.

yZ (x) ox,
n

Determinemos

:SOLUCIN

el min

g't. (x) dx con las condiciones

'"

) 112(x) dx= 1,

!I (O)=y (n)=O.

La ~w"~lon de Euler para la funcional


n

I (U (x)J =

(!I'a - Aya) d

o
tiene la forma
Y

+ 1.y = O;

y (O)

O,

y (n) = O.

Las funciones propias de este ltimo problema son !In (x) = sen nx
y los valores propios son ~ = fll.
El valor propIo mlnlmo es ~ = 1. Por eso, en virtud de la propiedad 4),
n
mln

J
O

y'2 (x)dx= l.

15.

LOS VALORES

En consecuencia,

Y LAS FUNCIONES

funcin y (x) tal que

para cualquier

y2

(x) x = 1

tenemos

It

~ y';\ (x) dx>

o
Esta

173

PROPIOS

desigualdad

y2 (x) dx.

no se puede

precisar

ya que para

Yl (x)=

sen x

li

se tiene

jo

J y?

Yi9(x)dx=

(x)dx=1.

.1(

OBSERVACIN. Si

anterior

introduciendo

= k2

-+

1, el prob lema se reduce

la funcin z (x)

y ~)

y'J. (x) dx

al

Empleando la definicin extremal de los valores propios. sealemos cmo pueden ser calculados aproximadamente
a partir del mtodo
de Ritz, Debe tenerse en cuenta que el mtodo de Ritz da una aproximacin por exceso del valor propio.
EJEMPLO 3. Hallar aproximadamente
el primer valor propio del
problema

y"
';.."y = 0,
y (-1) = y (1) = O.
SOLUCIN. El problema

sobre el mlnirno

de la funcional

{J'2

dx

-1

con las condiciones


1

Y ( .- 1) = y (1)

=O

y2 dx = I

-t

es un problema isoperimtrico
mo de la funcional

y se reduce al problema
1

JIy (x)) =

-1

(y'2 -

'}..2y2)

dx

sobre el mini-

174

CAP. 111. M~TODOS

DIRECTOS

cuya ecuacin de Euler coincide con la ecuacin dHerencial considerada y"


Aay = O. y (-1) = fI (1) = O.
La solucin general de la ecuaci6n es y = Cl cos'Ax
Ca sen AX,
De las condiclones de frontera encontramos

et cos 'A-Cz

sen A = O, }
C t cos A-\- C2 sen A = O,

(lO)

de modo que la condicin de existencia de una solucin no nula del

sistema (lO) es la condicin de que sen2A=0, o sea, A=n;

Por conslqulente, para el primer valor propio tenemos A~=


= ( ~ )2 Y la primera arm6nica de la cuerda viene dada por la solu.. exacta
t y=cos-y,
1tx. '=Ti
1t
1a segun da arrnonrca
' .
clan
es y=sen.nx,
~

= .n;

I a t ercera armonrca
. . es y

3nx
= cos -2-'

= "23 n, etc.

A ttulo de comparacin, busquemos las soluciones pares (armnicas pares de la cuerda) aproximadas en forma de un polinomio segn
las potencias de x. Tomando las funciones coordenadas en la forma
'PI (x) =x21l-2_x211 (k= 1,2, ... ) minimicemos la funcional
J en las
m

=~

funciones 11m(x)

Ck'P" (x).

Limitndonos

II-t
=c!p (x), tendremos J
el

!~;.,z)

(x)J=-c~ ( : -

({JI

al

trmino

{JI (x)

y para determinar

obtendremos

oJ

IYt (x)]

.:Je1
Puesto que debe ser el

=F O, resulta

A' = 2,5. Tomando para y

Y2 (x) = C'Pl (x)

+ C!Pi

!~AZ) +2c

185-

(x).

encontramos

[Y2('~)]=C:

(~ -

l'2 (

1~5 A2)+

+ei (
y para determinar
iJJ[yz(xH_

Oc!

-Ct

iJJ!yz{x))
OC2

Ct

el

C2

1:- 3\~;'2) ,

obtenemos el sistema

(..!!-~A2)+
3
15
(J2.._~A2)-j.
15 105

Ca

c2

(~-.E_AZ)=O
15
105

(176_.2!.AZ)=O
105 .315

...I
J

S 15.

LOS VALORES

Y LAS FUNCIONES

PROPIOS

175

La condicin de existencia de soluciones no nulas el Y c: de este ltimo


sistema es que su determinante sea igual a cero; esto da A4 _ 28)..2
63 = O. de donde Al = 2,46744 Y 11,1 = 25.53256. Comparemos los
valores aproximados obtenidos para AP y 11,: con sus valores exactos.

El valor exacto de ).: es ( ~ ) z ~ 2,46740 Y el valor exacto de A~es


(~

) ~ 22,20661 de modo que el valor aproximado

obtenido para

Al es

de gran exactitud mientras que para el segundo valor prop o se


Obtiene una aproximacin tosca.
EJEMPLO..
Hallar el primer valor propio del problema
yW
(1 + Xi) !I = 0,
y (-1) = !I (1) = O.
SOLUCION. Tomemos como funciones coordenadas las funciones
CJ>A (x) = 1 - x'Jfl (k = l. 2...
) que satisfacen, obviamente,
las
condiciones de frontera. Poniendo

+ )..

Yi (x) = Cl (1 - x') + c:a(1 - x4).


el problema sobre la mlntmizacln
de la funcional

planteemos

1 lb' (x)) =

ly'2- h (1 +xZ) y2) dx

-1

que tiene la ecuacin dada como ecuacin de Euler. Tendremos

J[yz. (x) =cl ( ~ -

:~~ ~.) +2clc2 (

1: - ~: A) 7
2 (

+c2
Para determinar
oJ[!l2(X)]
OCI

al [Y2 (x)]

32

5&3~

T- 3465 A).

Y el! obtenemos el sistema

CI

=2

CI

(!_
128 'J..)..L2
(...!i_~i.)-O
'\I
3
104
5
45
-,
,C2

(~_~.)

+2

(32 _ 5888,)

-o

5
45 '"
C2
7
3465 '" - .
La condicin de existencia de solucin no nula de este ltimo sistema
lC2

el

da

52)..20- 1068j"
2079,.." O.
tomando la raz menor. encontramos AJ = 2,1715.
PRINCIPIO De RAJLEIGH. Supongamos que se tiene el problema de
valores prop os
de donde.

L(y)=-

:'" [P(,X)

:~J+q(x)y=~"r(x)y.

alY (a) HtY' (a) =0,


cxH-~l > O, }
Ct2Y (o) +~2Y' (b) =O. cx~-Hi > 0,
donde p (x), p' (x), q (x) y r (x) son continuas en [a,
en [a, b).

<11'1

(12)

nI; p

(x)

>

176

CAP 111. MeTODOS DI'REeTOS

Diremos que la funcin y (x) es admisible (y E D) si tiene dos


derivadas continuas y satisface las condiciones de frontera (12).
Supongamos que para toda funcin admisible y (x) se cumple
la condicin
b

yL (y) d:c"> O.

En este caso el problema'de contorno (11) - (12) tiene solamente valores propios reales A.
Podemos poner en correspondencia a este problema de valores
propios el siguiente problema varlaclonal:
en tre todas las r unciones admisib les y (x) tajes que
b

> O.

r (:c) y2 dx

(13)

a
b

S
ha 11ar aque lla para la cua 1

IIL (y) dx

a
b

= m n.
r (x) g2 dx

Sea y = 1I>,'(x) la solucin de este problema.


Si Al es el valor mnimo, o sea, si
ti

S
Al

= mo
lIED

ti
~
a

gL (y) dx

o
ti

"'tL ('1>.)

dx

~~b ---

!r",~dx

r (x) y2 dx

entonces Al es el menor valor propio positivo y "1>1 (x) es la funcin


propia que le corresponde.
Si a las uncones admisibles se impone, a parte de la condicin
(13), una condicin ms
ti

r1j)y dx

=O

C1

(condicin de ortogonalidad), el problema


b

~ yL(y)dx
_0-,-

= mln

~ ry2dx
el

tendr

de nuevo una solucin

"1>, (x).

LOS VALORES

16.

Y LAS FUNCIONES

PROPIOS

177

'Si A,I es el valor mnimo correspondiente,


entonces A, ser el
siguiente, en cuanto a la magnitud (A.l ;;;a: ~). valor propio y 'ila (x)
ser la funcin propia ortogonal a '$1 (x) que le corresponde. En general. si se conocen ya los k primeros valores propios positivos

A. ~
Y el sistema

ortogonal

)'z ~

"PI

~ (x),

el valor

propio

Ah

. . ~

correspondiente

de funciones

'l'k

(x), . . .,

propias

(x).

siguiente. ser igual a


b

S yL(y)
A.Mi

= mn
l'ED

_;;;a...

b-_-

'i f!J2 dx
(1

con la particularidad
de que Se consideran aquellas funciones admisibles y (x) que, a parte de (13), satisfacen las siguientes condiciones
complementarlas
b

r(x) 1j>'II (x) y(x)dx=O

(v=l, 2... k).

>

Si en la ecuacin (11) se tiene que Ia funcin r (x)


O en la, b)
con frecuencia se emplea para estimar por arriba el menor valor
propio positivo Al la s gutente desigualdad (principio de Rayleigh).
b

S
Al

-<

!JL (y) dx

.:.;.1I"'b---

S ry2 dx
a
EJEMPLO 5. Valindose

del principio de Raylegh, estimar ~


problema de contorno
_yW = Ay, 1/' (O) = O, y (1) = O.
SOLUCION. En nuestro caso tenemos L (y) = -!t. o sea, P (x) e
;:; l > 0, q (x) := O y r (x) El> O en [O. 1]. Es obvio que ~ = O,
~1 = 1, a, = I Y fls = O de modo
que al
ff = 1 > O y al
+ ~i = 1 > O. Tomemos como funcin admistble ti (x) = 1 - Xli;
segn el principio de Raylelgh, tendremos
t
t
~ yL (y) dx
~ 2 (1 - x2) dx
4
O
3 = 2.5.
~
c~
....:0:..,= -8-

en el siguiente

IIot """

1
~ yr'l. dx

(1_x2) dx

15

Recordemos que el valor exacto es Al


12-01387

:n2
=4~
2,4674.

CAP. 1lJ. MaTaDOS

178

DIRECTOS

Estimar el menor valor propio en los problemas


siguen:
229. -y" = J.. (10 - X2) y; y (-1) = y (1) = O.
230. -y" = Ay; Y (O) = Y (1) = O.

que

En el problema de la determinacin de los valores y de las funciones propios tambin se puede emplear el mtodo de Kantorvcn
(mtodo de reduccin a ecuaciones diferenciales ordinarias). Supongamos, por ejemplo. que en un recinto D se tiene la ecuacn

+ "-z = O

6.z

y que

zlr=O,
donde r es la frontera del recinto D.
Busquemos la solucin en la forma
m

Zm (x,.

Xn)

X2 ....

= 2j

a.h

(x) IPk (x, y) +q>o (x. y)

k""t

escogiendo las funciones coordenadas q>k (x, y) y las funciones 0'.11 (x),
por ahora incgnitas. de modo que Zm (x, y) se anule en todos los
puntos de
Las funciones al (x), al! (z), ... O'.m (x) deben satisfacer el sistema de ecuaciones

r.

(k=I.2

[Mm+)..zmJIP)I(x,y)dy=O

.....

In)

(14)

D:f

y deben anularse en los valores extremos del argumento. Aqu D.'!(


es la in terseccin del recinto D y de la recta x = consto
.
Aquellos valores de ).. para los cuales el sistema (14) tendr solucin no trivial darn una aproximacin de los valores propios y las
soluclnes correspondientes darn una aproximacin de las funciones
propias.
EJEMPLO 6. Hallar aproximadamente
el primer valor propio
y la primera funcin propia en el problema

6.z + AZ = 0,
donde el recinto
SOLUCIN,

z Ir = O

D es el rectngulo: -a ~ x ~ a, -b '" y ~ b,
Buscamos la solucin del problema en la forma
ZI (x, y)

(y2 -

ba)

a, (x).

La eucacin (14) toma en este caso la lor ma


11

-11

120'.,

+ IY2. -

'Z)

al + , (y2. - b2) al} ly2 - b2)

dy

= O.

15.

LOS VALORES

I.AS FUNCIONES

PROPIOS

179

o sea,

..!ib5ct+

(~b5"
15

15

__

al (-a)=at

!3 ba) al=O '

(0)=0.

la solucin general de (15) es

ai (x) = C I sen

( 15)

+ C2 <:05

'}.,---2":O"~Z-X

A- 2!; x,

Teniendo en cuenta la simetra del problema y tomando una solucin


particular, obtenemos
y

Ct.:...O

Czcos

i..-

2;2

queda claro de aqu que tendremos una solucin

/'

no trivial slo si

,- 2b2 a=t2k-I)T;

hA =
En particular,

a=O;

(2k-I)2 n2

5
-1-

(2a)2

2b2

para k.....,._ 1 encontramos

A, =

n2
(2a)2

10

+ 2b2

siendo el valor exacto

n2
n.2
Al = (2a)2 + 2b2'
El error es menor que 1.3%.
Para la primera funcin propia oh tenemos la aproximacin
zs(x, y) = (g2_

b2)

nx

COS 2Q'

Hallar una aproximacin del primer valor propio en los


problemas que siguen.
231. y"
')..2y = O, Y (O) = y (1) = O.
232. lJ"
j. (2
cos x) y = O, y (O)= y (n) = O.
233. Hallar aproximadamente el primer valor propio en ~I

+
+

problema

~z

AZ = O

z Ir = O,

donde D es el crculo de radio uno con centro en el origen


de coordenadas.

RESPUESTAS E INDICACIONES
lo al 'mlD =0 en el punto (O. O); b) tmx = 1 en el punto (O. O);
e) no hay extremo. 2. No hay extremo. 3. f mili = -8 en los puntos
V2) y (V2); en el punto (O, O) no hay extremo.
4. !mln =O en el punto (O, O); en los puntos de la circunferencia
x2+y2= 1 hay mximo no estricto. 5, 'mx=
en el punto (1,-1).

eV2", -

V:2,

Vs

6. f mtn =4 en

el

a- 3113

R. fmlo--

punto (-},

en el punto

1, I ) . 7. f mln =

(2n
3'

2rc) ;
3""

t en el punto (l, O).

3113

fmx=-g-

enelpunto

n2+n+2

( ~,

;).

9.

fmax:= ( n2;-~+2)

para x,=X2-='''=X",=

:-:;n2 2n+2' 11. No. 13. Los nmeros

a.1t

cientes de Fourier

14. tmln =

de la funcin'

(x).

~h

deben ser los coefi-

-i-

en los puntos

1 ' - y2"1) y (1- V'2' y21) ; fmb.=T I en los puntos


( V2
1 lf2
1) y (1- Vii - Vi1) . 15. fmln=l336 en el punto
( V2'
18
12 )
( 13 ''""i3

16. fm-x=4 en los puntos (2,2,1),

f mx "'"4 27 en los pun 10$

(1,2,2) y (2,1,2);

("3'" 3"4 37) ' (73" "3'


4 3'4)
t

(12

4 3'
7 3"
4) .
(3'

( :' !);

17,

fmx

=(!

18.

fm1n=1

fmllx = I1 en ,1 punto ( - ~ , - ~ ).

en el punto (-1

2. - 2); fmx =9

en el

punto

en

19.

el

punto

i-: = -9

(1, -- 2, 2). 20.

RESPUESTAS

E INDICACIONES

fmx=i;- en el punto (~,

~,

~).

mnimo de la funcin z=+(xn+yn)

4V5

22. c.

23. -5-'

26. El

a=R'V2.
y

1911'2
8

24.

181

21. Indicacin.

el

con la condicin x+y=S.

. 25. El cuadrado

V V5'
2-

de dimensin

r = ~ -,/2+

radio de la base del cilindro

la altura del cilindro h = R

Hallar

V'S

27. Primero. 28. Proximi-

dad de cualquier orden. 29. Proximidad de cualquier orden. 30. p=e-1


31. p=l. 32. p=e-l.

33. p=e-1. 34. P2=

2n+3
6
.36. PIOOl'=e.

36. Continua. 37. Continua. 38. Discontinua (conSiderar

nx) . 39. a) Discontinua;

Yn (x) = senn
b)

continua.

41.

a;

48.

-0,1

49. M

",S. llJ=-2-

O,J

6J

1
-0,1
0,01

0,01002

11J =a+

3(e2-1)
+6 (3-) 2+~.
4
a
e a
5'

0,1
0,01

50.

b) continua. 40. a) Discontinua;


l-e2

Continua.

1,2
-0,098

0,01

la sucesin

f,J

4,7919

6,6821
0,4963

0,4792
0,0479

a2
5; oJ =a.

"J _ 3 (e2_1)
u 4
a

0,0481

Diferencable; 2) diferenciable; 3) diferenclable; 4) no di{jJ2 [g (x)) = 2J (g (x)] ss.


e
63. M=3k+--1 k2; 6J=3k.
1)

[erenciables. 51.

e-

sr

4,582

0,1

0,3158
0,03016

0,3
0,03

0,01

R,ESPUESTAS

182

5
54. tlJ--k+- 3

E INDJCACIONES

8
5
7' k2. tJJ=:rk
AJ

11

6J

2,810
0,1
0,01
4
55. M=Tk?;

0,181
0,0168

()J == O.
k

6J

O
O
O

1,3333

-1
0,3
0,03
b

J ~ydx.

57. 6J=

1,667
0,167
0,0167

0,1200
0,0012

58. 6J=2)

(y6y-y'

Oy'}dx.

a
1

59. 6J = 2y

6y {O) +

(x

~y.+2y' ()y') dx.

O
n

80. 6J =

J (!J' cos y 6y +sen y tJy') cU.


o
b

61.

(JJ

rl ( -aof 6Yl +-(}


al- 6112+'" +r
af,) 6Yn
{Jt
112
!In

d.t.

62. 6ZJ IY, y) = 2) Sy, 6y)


63.

62eF(I/)=eF(lI) {(6F)2

65.

J~
b

(J2J

m
LJ

a /l. 1-0

66. 67-J

= Jr Jr (F~

J'lF
ayO)

6y(lt)

6y(l1 dx.

{&)2+F" ,6~ 6z~+... +F" (6z;")2J dx dy.

m.

i, 11-1

1[ ~

+ 62FJ.

Oy(lI)

67.0'1.)=

Ux

z.trv

F;,lIkOy8YII+

~
t,

F;lIt6Yt{JY+

11-1
11

R.ESPUESTAS

68. Considerar

183

E INDICACIONES

la funcional
J [cp+a.TJJ=11> (a.)

y emplear la segunda definicin


de la variacin.
6J =0, llegamos a la ecuacin integral

Exigiendo

que

JK

(s, t) q (s] ds+cp (1)- f (t)

=0.

69. Procediendo de la misma

forma que en el problema anterior,

encontramos que la -cuacn funcional de Euler, que expresa la anulacin de la primera var iacin, viene dada por
(pq')' - CJ' (x+2)-

q> (x- 2) +cp (x)+ f (x)=0.

Esta ltima es una ecuacin mixta con derivadas y diferencias.


70. -(pq:')'+qq=f(x),

71.

y=_x3,

72. y=

Sh~~~X)

.73.Dos

extremales
1

+ (3 + 2 V2) (2x-

y=
74. Dos extremales

+x)senx,

donde

1)2

1)

e y=)'I'"(3x_1)2. 75. y=(C+


I
Una constan! ... arbitraria.
76. 1I=-rX

1I=1"(X+1)2
es

tt. Y="6"'-6X3,

X[e-x+(l+e)xe-x-IJ.
1
- '"6
",3+ 2.

(V2

79. 11 = In x, 8]. La integra!

78.

13

Y=6"'-

no depende del camino

de integracin; el problema var iacional carece de sentido. 82. y ....O


si a.=0; siendo a. =fr. 0, no existe extrerna l suave. 83. y=cosx.
84. y=cosx+Csenx.
donde C es una constante arbitraria. 86. 1!=
sh x
2
=x + \. 86 .11= ShT'
87. Y =e (I-x). 88. No hay extrernaes; la
ecuacin

de Eu ler

no

90. No hay extreruales.


96. y

.lit sen x
~O~

=(I-x)shx.

tiene

soluciones.

89.

y=Cj
1

93. y=C1e:>:+C2e-x- Txex:.

. 97. y=2x.

98. La circunferencia

x3
100. U=T{x3+6x+f).

-t-C2"'- ~z. .
94. U=2chx

J
r

-=K. 99. 11=

10]. No hay extremo. 102.

184

~ESPUESTAS

E I NDICACION.ES

El problema variaconal carece de sentido porque bajo el signo de la


integral figura una diferencial exacta. 103. y=sh x,
y=sen

104. g=+x2

J06.

105. {

2x,

x2.

2=-2+

Y==x-.{- (x3+5x-6l.

32+n2

8n

x.

107. { !I = sen x,
z=sen x,

{
Z

x2.

lOS.

11='""'2+1,
{

z=

1.

j}z ) 2 iJ2z
( iJz ) 2 iJ2z
110. ( Tx
'aX2+ ay ay2 =f(x,
n

112.

a [ aj(x,.
~""'liX""
1
1-

Xn)

X2

~
ox

y).

+c(x,.

, '.

xu) z=

X2

, Xn)

X2

= f (XI,
113. Solucin. El planteam lento

del problema

es el siguiente.

Entre las superficies z =q> (x. g) que se proyectan en el recinto D del

ptano xOy y que pasan por cierta curva cerrada alabeada cuya proyeccn es la curva frontera r del recinto D.,!ha llar la superficie 'cuyo
lea
s

=j

lVI +

IP!

+ cp; dx dy

!)

sea mfnlmo (problema de Platean). La ecuacin


para este prob lema es

CP,t:

"1X VI +CP~+CP~

+..!..
~g

dHerencial

IPy

de Euler

1f1 +q>!+II'~

o en forma desarrollada
11'_ (1 + II'~)- 2CPzl{q>z<p,,+ CPvv(1 +q>!) =0.
Esta es la ecuacin diferencial buscada de las superficies de rea
mnimo. La realizacin stca de la superficie de rea mlnma se puede
obtener, por ejemplo, con una pelicu la de jabn tendida sobre un lazo
de alambre.
114. Z (x, 11) = y. El problema tiene solucin nica aunque las
condlcones de frontera no se dan CIl toda la frontera.

RESPUEST

115. r cos

IP+ea

AS E I NDICi\CIONF.$

= el In I r sen

185

IP+ V,Z sen? IP- en

117. x2 ces C2- y2 cos Ca-2xy sen ez =C,.


1t8. Campo central, 119. a) Campo propio; b) campo central;
e) no forma campo. 120. Campo propio. 121. a) Campo central: b) no
forma campo; e) campo propio. 122. a) Campo central; b) campo propio; e} no forma campo. 123. No lorma campo porque esta familia
de curvas no cubre lodo el recinto D. 124. y=C eh x forman un campo
propio de extrema les; y = C2 sh x forman un campo centra I de ex tremales. 125. y=Ccos.v: forman un campo propio de extremales: y=
= C sen x forman un campo central de extremales. 126. La extrema I

y= ~ (l-x2)
y=Cx~

puede ser

incluida en el campo central de exlremales

con centro en el punto O (O, O). 127. La extremal

Ij=e>.:

se puede incluir en el campo propio de extremares y=ex+C.


128. Si
a<n, la extremal !I::.~O se puede incluir en el campo central de extrernales y=Csenx con centro en el punto 0(0. O). Si a>n, las curvas de la lami! ia y =C sen x no forman campo. 129. La extrema] y =
=x+ 1 se puede incluir en el campo propio y=x+e. 130. y=

x; .

:c -

131. Y ( ~

-x)

= O. 132. y?-I =0.

0+ (l. O).

133.

134. No hay punto conjugado. 135. Se cumple. 136. Se cumple cualquiera que sea a. 137. La condicin de Jacobi se cumple. La extremal
11=0 se puede incluir tanto en un campo central como propio de extrema les. 138. La condicin de Jacobl se cumple. La extremal y =

!..=.l.. x+ 1
(l

se puede incluir en un campo central

de extremales con

centro en el punto A (O, l). 139. La condicin de Jacobi no se cumple.


142. Se puede. 143. Se puede. 144. Se puede. 145. Se puede aunque la

condicin de Legendre se cumple slo para


< l. 146. Se alcanza
mlnlmo fuerte en la funcin y=t'J!.. 147. Se alcanza mnimo fuerte
en la funcin y =21n (x +'1). 148. Se alcanza mntmo dbil en la Iuncin 11

J;2

.149. Se alcanza

mntmo

dbil

.
160. Se alcanza mnimo uerte en la curva y

en

b
y=- x ,

la recta

In (I+x)
1n 2

. 151. Se al-

canza mximo fuerte en la curva y=cosx+senx.


152. No se alcanza
extremo en curvas continuas. 153. Se alcanza mnimo dbil en la recta
11= 2x
l. No hay extremo fuerte. 154. Se alcanza min rno fuerte en
la extremal 1I=2x.-1. 156. Se alcanza mlnimo fuerte en la extremal
$t=xa. 166. Se alcanza minimo dbil en la ex trema I !I =x-I,
157.

n la extrernai y
y mxmc

=!_ x

a~l>j) ;i

se alcanza

,b, > -V2 .

No

mnimo

db i1 si lb'

hay extremo

si

<~

V2

I I = ';-2 .

RESPUESTAS E INDICACIONES

186

158. Si P =F q,

se alcanza

= {[(lfI2
p3/2'] x+p3/2)2.;
que ofrece mnimo db il.

mnimo dbil en la extremal


y=
si p=q. la extrema! es la recta y=p
sh

159. a) Para 1;:> O la extremal

x
-v=;-

el

realiza

Vs
1
le I > nz'

rnntmo

sh

fuerte de la funcional.
sen--=-

b) Si

E,

<O Y

la

extrernal

U=

x.

V~I

real iza ('1 mximo ruerte de la funcional.

t;

sen V le I
no existe solucrn

di!! problema

e) S e =O.

en la clase de Iunclones

extremal

x-I

cont inuas. Consideremos la funcin


Ur, (x) =f! Ve
(e> O) que es
solucin de la ecuacin de Euler ell"-II=O
de la funcional constderada. La funcin lIe (xl satisface la condicin de frontera y (1) = I
y no satisface la segunda condicin de frontera y (O)= O. No obstante,
lm !Ir. (O) = O. Para e -+- O obtenemos de !le (x) la esolucin lmite
e-()

(x)

={

160. La extremal 11= -

O~x<l.

O,
l.

2 In (1

x=L
.i,

In 2'

x)

161. Hay mnimo fuerte en la extrernal


b
I
.
d b . f
g ( x ) =ax se a canza mrmmo e I SI

V3
a> -2-;

. a=-2-'
b
V3

. b

51

dbil.

SI

163. En 111 recia

realiza

g (x)

= 1.

el mnimo

fuerte.

162. En la extremal

b
V3
a<-2-

,-

y rnaxrmo

no se a Icanza rn'..
siquiera

d eb ilI

e I ex t remo

se alcanza mnimo dbil si b ;a


a
y mximo db il si b> a si b.;? a
hay mximo fuerte mientras
que para b < a
no hay ni mnimo ni mximo fuertes. 164. Se
alcanza mnimo dbil en la extremar U=2x, z=4x. 165. La exlremal

y3

es la parbola

{ ~:.:~

!J"~x

'Va.

.-x que se puede incluir en el campo

cen-

tral de extremales
( 1)

RESPUESTAS

187

E 1NDICACTONES

a y ~ son unos parmetros) con centro en el punto (O, O, O). Es


obvio que se cumplen las condiciones reforzadas de Legendre. Demostremos que en el segmento O ~x ~ 1 no hay punto x conjugado del
punto x = O. Para ello bastar persuad irse de que las ex trema les de la
familia (I) no cortan la extrernal dada si xE [O, IJ' Supongamos que
en un punto x E (O. 1) se cortan dos extremales
e la familia (1).
Tendremos entonces

asx = (%2X,
x2 + ~tx= x"+ ~"x.

De aqu resulta que <x, =(%2 Y que ~. = ~z. Luego, no hay dos extremales distintas que se corten. Por consiguiente, la condicin reforzada
de Jacob i se cumple en el segmento rO. 1) e incluso en cualquier segmento de lonttud finita. 166. La familia de extremales es y (x) =

=e,

ch

X~l

el

A. Las constantes arbitrarias

y Cz y el parme-

de las condiciones

tro ). se determinan

xo-Ca

el

!lo= ,eh

A.,

lCt

) 1f1"'+7i dx= el

(Sh Xl"Z. C

xo-c" )-1

el

Sh

- .

XI)

168. y(x}=2sennn,x. donde n es un


I
,rnmero entero 169. y(x)=T(2x-xZ).
170. V 6. 17t. r=R, z=c1
167. Y (x)

+ezCJ)

=3X2+2x+J.

V4S

172.

V'Ti
--2-.179.

,/"173. V 20.114.2

Ir
r 17+4

,r
V 2-1. rrs,

v- (5"2-'1/"-)
6

V 6

el extremo se puede alcanzar solamente


cambio, si cos x =0,
se tiene

o sea

y=e,senx,
z= -

J (A, B) = 4 cth 1.

e,senx,

Xl

= ;

siendo

183. J (A. B) =

. 180. 1. 181. Si
en la recta

+ me. donde
e"
~ .

VIO
----ro'

t1

Z=O

En

es un nmero entero.

una constante arh itraria.


184.!J

=F O,

COS Xi

y=O,
{

178.

= 2x2/3

182.

185. Las lneas

quebradas formadas por los segmentos de las rectas y = x e y=;:; I o por


los segmentos de las rectas y=O e g=x-I
realizan el mnimo absoluto. La recta

Y=TX

realiza el mximo

dbil.

186. y=

-)C

para

188

Rr;SPUESTAS

INDICACIONES

0<:1;
y=x-2para
1<x-<4cy=xparaO~x~3;
y=-x+6
para 3 <x
4. En ambas quebradas la funcional alcanza su mnimo
O, :<-<0,
a b soIuta. 187. No existen. 188. y = {
189. Las extremales

<

sao lneas rectas. Si

YZ-Y1
;(2-Xl

1<

x, x>O.

1, existen dos soluciones dlsconti-

nuas que son lneas quebradas paralelas a las bisectrices de los ngulos coordenados. 190. La recta y =.dg (j) Que pasa por los puntos fijos
realiza mximo dbil si O<tgcp<n,
realiza mnimo dbil si n<
< tg!jl < 2.n, etc, El mnimo Iuerte se alcanza en la lnea quebrada
formada por segmentos de rectas tales que la tangente de los ngulos
de su inclinacin es igual a 4n;- I n (n es un nmero entero).
3

191-

t ~V9.-

(xl"

'" x""5'

16 ,_

(x- 5)2.

,/ 34

5 ",-x.o:::,5'

_ 3(x-10)

16

O "

Tx,

34
5<x<10.

192. Las exlremales son las elipses


()I
con centros en el eje Ox, La frontera del recinto admisible se determina por I11S ecuaciones y=O e y2= 2 (x-ea) (la ltima es [a
solucin de la ecuacin 1_.y2y'2=0).
Los parmetros e, y e'l, se escogen de modo que la elipse (1) pase por los puntos fijos A y B. La funcional
alcanza mximo en el arco de la elipse. Si el camino del punto A al
punto B se escoge segn los arcos de dos parbolas (Y. posiblemente,
segn un segmento de la recta y =O), se obtiene una solucin con
puntos angulares en la que la funcional alcanza su mnimo (mn J =O).
dy

193.

195

.!!!L

= P21
P2
~ 2!h '

= 4p2

11x2"'

dx

dPl =0
dx

e/PI

'""dT =

-11,0;,

dp

)',2y2

dX

dx

x'l.y

P
, dp =
y2_ p?
dx

'dx

=..E!.
2'

dY2

...EL
2yf

'

dy

='2;)' 194.

dp2

dx

!J

11",2+ y2_

P
2xy'

dp

J96.

p2

p2

dx = 4xy'l.

dY1 =
dx

dYI =...f!!_ ddYx2=


dx
2Yt'
d!1I
PI
dyz
"'(IX = - T' Tx =

dP2 =2y . 197.


dx

p~

-;- --- 4y~

:1 ~ 2x, ':!;2 -O:

198.

'J'xf

.-

~t.

~~2 =11P'l..

~ESPUESTAS

:~I

=2x,

=C,

~~ =0.

199. yJ=C1x3+C2.

dy

11G2(y)_Q

200. y3=ln2x.

+C2 202. En la extremal

ruer t e:

'
a 1canza e 1 mmimc

189

E INDICACIONES

J = --5
mm

. 203'.

201. x""

x2_x_1

11=

p (x, y)

s~

1122xyXl

Las extremales son las semicircunferencias


Y=Vq-(X-C2)2
con
centros en el eje Ox; JI = 2Ctx _x2 son las exlremales que pasan
por el origen 0(0. O); el campo es el semiplano superior. 204. El arco
de la circunferencia que pasa por el punto MI (Xi. JI,) Y que tiene el

-v

centro en el punto O (O. O) realiza el mnimo uerte. 205. xF (~) =C.


206. Las elipses 3x2-8xy+6y2=C. 207. x3+2g3-3xy2_2x2y=C.
208. I =
209. I =xy
y'. 210. l =xyy'2.
21 t.

*+

V-:::yt2.

==v(

;2) (X211'2.+1I2).

La integral de la accin es J =
trayectorias

XZ

son las el ipses -C

212. Una catenaria.

J JI'

213. Indicacin.

V p2_f-p'2dcp.

p~ +2h

t=

2 ces fi
+ -2h2y2
C - V
. C (2h-C)

214. Las

sen2 ~
xy =- ..
- .
1<

215. La solucin exacta es 11= :~ ~ - x, 216. Las soluciones exactas


217. La solucin exacta es JI =.

son a) y c= O, b) y=x.

La solucin exacta es y= 2s~h; -x.

es y = cos

coordenada; entonces,
=X2+y2,

n: .

= (l_x2)

rt

altx211.

La

50-

xl/. 221. Indicacin: tomar. IPo(x,y)=

cpdx, y)=xy(l-x-y),

cin: hallar la primera


a(x);

11n (x)

220. Indicacin: tomar xy como funcin

b2+a2

.. cp" (x, g) =xny(l-x-y}


l3 (x, y) = xz+ 1I2+xy (l-x-

_ ~2)

218.

11... 0

b2-a2

Zt

(x2-x).

219. Indicacin: la solucin apto-

xmada debe buscarse en la forma


lucin exacta

lP2,(x,y}=x2y(l-x-y)
...
como (unciones coordenadas; entonces,
11)[3,0401-0.0562 (x+x2)]. 222. Indica-

aproximacin

entonces

en la forma

4dx. y) = _ ~

zl

(x, y) = ( y2 _

(1- ~) (112-

;2 ).

190
223.
=1

RESPUESTAS

2'1

3 ( yZ-'31)
(x, y)='4
x2

+ n .n

Un (x) =

Z 2

_lnz2+4nZrZ
-

4I

z2

fin (X )-_ +
_

V!

(I._
35
2.35
1_36X-I_3&X-5-1

V'2 sen nnx

V'2 senVi(nn In x)

_
Ylny2Vx
000

(n -

(n = 1, 2, " .).

l= sen (To;-Inx)

' !In (x -

sen sx

E INDICACIONES

I , 2

225.

226'

"'n =

_.

) 2"7'
lO.

"'/1

. 224.

"-n=
h.n =

25+2n2.nZ
4

Z
= 1-I\,!/n

(X

'
)

(1\= 1, 2, ... ).

228.
131n22+4n2n2
41n22

' !In (x) =


(n=I,2,

nn In (I+x) ]

sen [

In 2

,r----'=,r
V

In

1 +x

... ).

229. Tomando !I = 1- x~, encontramos Al

-<

:S.

13

El valor exacto

-<

es Al = T' 230. Tomando y = x (1- x), encontramos Al 10. El valor


exacto es Al =:n:2 23'. Al = 10 el valor exacto es i..f = n2 232. Af=
= 0,493. 233. Al= 6;

Zt

(x, JI)

= IX (x2 -1- y2 -

1).

A NUESTROS LECTORES:

MIR-RUBIOS 1860 edita libros soviticos traducidos al espaol. Entre ellos figuran las mejores obras
de las distintas ramas de la ciencia y la tcnica: manuales para los centros de enseanza superor y escuelas tecnolgicas; literatura sobre ciencias naturales y
mdicas. Tambin se incluyen monografas, libros de
divulgacin cientfica y ciencia ficcin.
Dirijan sus opiniones a MIR-RUBIOS

el. Alcal, 98 - 28009 Madrid.

1860.

Al principio de cada capitulo


se resumen los resultados principales. se exponen los conocimientos tericos necesarios, las
rrmulas requeridas y se estu-

dian con gran detalle ejemplos

tpicos ilustrativos.
Este manual contiene ms de
100 ejemplos analizados y 230
problemas destinados para resol-

verse independientemente. Unos

problemas se acompaan
con
las respuestas, otros, con las
reereneiss de cmo deben resolverse.
La obra est destinada para los
estudiantes de centros de enseanza tcnica superior que se
especializan en los clculos matcmtees.

ISBN 84-604-1605-4

Vous aimerez peut-être aussi